[obm-l] Re: [obm-l] Experiência mental

2022-01-26 Por tôpico Fernando Villar
Olá Israel. A primeira vez que vi também tive essa impressão, mas ao ler o
livro descobri que os seres de Planolandia identificam uns aos outros por
meio do tato, identificando os ângulos. A referência ao formato é para
estabelecer uma correspondência com o que conhecemos.
O livro é muito bom, propõe uma discussão da hierarquia social vigente no
século XIX.

Abraços e uma excelente semana para você.

Fernando Villar


Em qua., 26 de jan. de 2022 às 09:20, Israel Meireles Chrisostomo <
israelmchrisost...@gmail.com> escreveu:

> olá pessoal, eu estava no youtube assistindo a um vídeo de Carl Sagan
> falando sobre a planolandia.Para quem ñ sabe, a planolandia é uma
> experiencia mental que considera seres em universos planos.Sem delongas, eu
> refuto a ideia de que os habitantes de tal universo enxerguem figuras
> geométricas planas, como o triângulo, quadrado, retângulo, circulo...O
> argumento é bem simples: só é possível ver figuras planas fora do plano,
> mas quem está no plano só consegue ver linhas retas.
>
> --
> Esta mensagem foi verificada pelo sistema de antivírus e
> acredita-se estar livre de perigo.

-- 
Fernando Villar

-- 
Esta mensagem foi verificada pelo sistema de antiv�rus e
 acredita-se estar livre de perigo.



Re: [obm-l] Dica Sobre Livros

2017-07-31 Por tôpico Fernando da Silveira Couto
Também tenho interesse.

Em 29 de julho de 2017 14:58, Kelvin Anjos  escreveu:

> Como essa lista é apenas para dúvidas e problemas da obm, te envio um
> e-mail com os anexos.
> Se alguém mais se interessar, basta me enviar um e-mail pedindo.
>
> On 29 July 2017 at 11:52, Ricardo Leão  wrote:
>
>> Eu tenho procurado os seguintes livros:
>>
>> - Andreescu, T; Kedlaya, K; Zeitz, P; *Mathematical Contests 1995-1996:
>> Olympiad Problems from around the world, with solutions* (1997)
>>
>> - Andreescu, T; Kedlaya, K; *Mathematical Contests 1996-1997: Olympiad
>> Problems from around the world, with solutions* (1998)
>>
>> - Andreescu, T; Kedlaya, K; *Mathematical Contests 1997-1998: Olympiad
>> Problems from around the world, with solutions* (1999)
>>
>> Alguém aí sabe onde eu encontro esses livros em formato físico ou
>> digital???
>>
>> --
>> Esta mensagem foi verificada pelo sistema de antivírus e
>> acredita-se estar livre de perigo.
>
>
>
> --
> Esta mensagem foi verificada pelo sistema de antivírus e
> acredita-se estar livre de perigo.
>

-- 
Esta mensagem foi verificada pelo sistema de antiv�rus e
 acredita-se estar livre de perigo.



[obm-l] Re: [obm-l] Introdução a teoria dos números

2016-04-20 Por tôpico Hugo Fernando Marques Fernandes
Aqui tem um pdf deste livro em inglês, se interessar.

http://matematica.cubaeduca.cu/medias/pdf/842.pdf

Att.


*Hugo Fernando Marques Fernandes*
Ministro Leigo da Igreja Episcopal Anglicana do Brasil (IEAB)
Diocese Anglicana do RJ - DARJ
Catedral do Redentor


Em 6 de abril de 2016 00:34, Israel Meireles Chrisostomo <
israelmchrisost...@gmail.com> escreveu:

> Alguém sabe onde posso encontrar o livro Introdução a teoria dos números
> do Hardy para vender em alguma livraria?
>
> --
> Esta mensagem foi verificada pelo sistema de antivírus e
> acredita-se estar livre de perigo.

-- 
Esta mensagem foi verificada pelo sistema de antiv�rus e
 acredita-se estar livre de perigo.



[obm-l] Re: [obm-l] corrigindo - equação irracional

2016-02-19 Por tôpico Hugo Fernando Marques Fernandes
Prezado Vitório.

Primeiro, S = { -1, -1 +sqrt[3], -1 -sqrt[3] }

A raiz da equação dada é sqrt[ 6 * sqrt[3] - 9] e pode ser obtida a partir
do segundo possível valor de k, acima.

Veja, k = raiz cúbica(x^2-1)

Com k = sqrt[3]-1
Temos x = sqrt[k^3 + 1] = sqrt[(sqrt[3]-1)^3 + 1] =
sqrt[(3sqrt[3]-1-3.3.1+3.sqrt[3].1) + 1] = sqrt[(6sqrt[3]-10)+1] =
sqrt[(6sqrt[3]-9)]

Com k = -1, temos k=0, que não convém.
Com k = -1 -sqrt[3] temos x = sqrt[ (...) negativo (...) ] não pertence aos
reais.

Assim, a solução única da equação é dada é x = sqrt[(6sqrt[3]-9)]

Veja, para mais detalhes:
https://www.wolframalpha.com/input/?i=%28%28x%2B1%29^%281%2F3%29+-+%28x-1%29^%281%2F3%29%29+-+%28x^2+-1%29^1%2F3

Abraços!

*Hugo Fernando Marques Fernandes*
Ministro Leigo da Igreja Episcopal Anglicana do Brasil (IEAB)
Diocese Anglicana do RJ - DARJ
Catedral do Redentor


Em 17 de janeiro de 2016 10:46, Prof. Vitório Gauss <vitorioga...@uol.com.br
> escreveu:

>
> Nobres colegas,
>
> Estava resolvendo uma questão do matemática elementar, que me parecia
> simples...
>
> Raiz cúbica (x+1) -raiz cúbica (x-1) = raiz cúbica(x^2-1)
>
> Usei a fatoração (a-b)^3 =a^3-b^3-3ab(a-b)
>
> Cheguei em 2-3(raiz cúbica(x^2-1))^2-(x^2-1)=0
>
> Chamei raiz cúbica(x^2-1) de k, logo (x^2-1) =k^3
>
> A equação ficou k^3 + 3k^2 -2 = 0
>
> Fatorando ela fica (k+1)(k^2+2k-2) = 0
>
> S = {0,  -1+sqrt[3] , -1-sqrt[3]}..substituindo para encontrar as raízes
> da equação irracional, não obtive a resposta.
>
> A resposta no livro, questão 556, b, vol 1 é +-sqrt[5]/2..
>
> Grato pela ajuda.
> =
> Instru��es para entrar na lista, sair da lista e usar a lista em
> http://www.mat.puc-rio.br/~obmlistas/obm-l.html
> =

-- 
Esta mensagem foi verificada pelo sistema de antiv�rus e
 acredita-se estar livre de perigo.



[obm-l] Re: [obm-l] Análise Combinatória

2016-02-18 Por tôpico Hugo Fernando Marques Fernandes
Seja A = { x | x é anagrama de PIRAMIDAL começando por PIR, nessa ordem }
e B = { x | x é anagrama de PIRAMIDAL cujas últimas 4 letras são A, D, I,
L, não necessariamente nessa ordem }

Queremos calcular n(A U B) = n(A) + n(B) - n(A interseção B)

Calculando, temos: n(A) = P 6,2 = 6!/2! = 360 (fixo PIR e permuto AMIDAL,
com repetição dos 2 A's)
n(B) = P4 * P5 = 4! * 5! = 120 * 24 = 2880 ( permuto PIRAM nas cinco
primeiras posições E permuto IDAL nas 4 últimas)
n(A interseção B) = P2 * P4 = 2! * 4! = 48 ( fixo PIR, permuto AM nas duas
posições seguintes E IDAL nas 4 últimas)

Logo, n(A U B) = 2880 + 360 - 48 = 3192

Att.

*Hugo Fernando Marques Fernandes*
Ministro Leigo da Igreja Episcopal Anglicana do Brasil (IEAB)
Diocese Anglicana do RJ - DARJ
Catedral do Redentor


Em 18 de fevereiro de 2016 12:09, Marcos Xavier <mccxav...@hotmail.com>
escreveu:

> Prezados amigos, preciso de ajuda para resolver esse problema.
>
> Quantos são os anagramas da palavra PIRAMIDAL que começam por PIR, nessa
> ordem, ou cujas últimas 4 letras são A, D, I, L, não necessariamente nessa
> ordem?
>
> Gabarito: 3192.
>
> Obrigado pela ajuda.
>
> Marcos X.
>


[obm-l] Re: [obm-l] Re: [obm-l] Re: [obm-l] Cálculo da Poupança - com capital inicial mais contribuições mensais

2014-08-04 Por tôpico Fernando Villar
Olá, Marcelo.

Você tentou essa?
https://www3.bcb.gov.br/CALCIDADAO/publico/exibirFormAplicacaoDepositosRegulares.do?method=exibirFormAplicacaoDepositosRegulares

Abs,

Fernando Villar


Em 4 de agosto de 2014 11:58, Marcelo Gomes elementos@gmail.com
escreveu:

 Olá Regis,

 Sim, exatamente. Eu estou querendo a fórmula que tenha o Capital Inicial,
 e as contribuições mensais para a poupança, tudo em uma fórmula.

 O Excel através da função VF=, fornece isto. Mas eu preciso da fórmula. Se
 puder me envie a fórmula pelo Excel para os cálculos da poupança.

 Abração e obrigado.

 Marcelo.



 Em 4 de agosto de 2014 11:07, regis barros regisgbar...@yahoo.com.br
 escreveu:

 Bom dia Marcelo
 VF do excel obedece o equação que encontramos nos livros de mátematica
 financeira. Caso tenha dúvida entre em contato comigo que te envio um
 exemplo na planilha para você.
 Se o lançamento for mensal na poupança, posso ajuda-lo.

 Regis


   Em Segunda-feira, 4 de Agosto de 2014 10:54, Marcelo Gomes 
 elementos@gmail.com escreveu:


 Olá pessoal da lista, bom dia a todos!


 Aos que trabalham com matemática financeira, peço a gentileza, se tiverem
 um tempinho de me ajudarem me enviando a fórmula para o Cálculo do seguinte
 item:

 1- Uma aplicação de caderneta de poupança com as seguintes
 características:

 Valor Presente

 Valor Futuro

 Contribuições Mensais

 Encontrei diversas calculadoras online, inclusive a do Banco Central, mas
 não apresentam os três itens acima.

 No Excel a Função VF= fornece o cálculo mas não a fórmula.

 O que eu estou querendo é: a partir de um valor inicial, usando
 contribuições mensais chegar a um valor futuro, como ocorre na caderneta de
 poupança.

 Abraços, Marcelo.


 --
 Esta mensagem foi verificada pelo sistema de antiv�us e
 acredita-se estar livre de perigo.



 --
 Esta mensagem foi verificada pelo sistema de antivírus e
 acredita-se estar livre de perigo.



 --
 Esta mensagem foi verificada pelo sistema de antivírus e
 acredita-se estar livre de perigo.




-- 
*Fernando Villar http://fernandovillar.blogspot.com *
*Projeto Fundão http://www.projetofundao.ufrj.br/matematica/ / CAp UFRJ
http://www.cap.ufrj.br/ *
*Doutorando NUTES http://www.nutes.ufrj.br/ - UFRJ
http://www.minerva.ufrj.br/ *
*http://lattes.cnpq.br/8188046206638473
http://lattes.cnpq.br/8188046206638473*

-- 
Esta mensagem foi verificada pelo sistema de antiv�rus e
 acredita-se estar livre de perigo.



Fwd: RE: [obm-l] off topic - livro caronnet​

2014-04-14 Por tôpico Fernando Brito
Me passe o link por favor

—
Sent from Mailbox

-- Forwarded message --
From: marcone augusto araújo borges marconeborge...@hotmail.com
Date: Mon, Apr 14, 2014 at 9:08 PM
Subject: RE: [obm-l] off topic - livro caronnet
To: obm-l@mat.puc-rio.br obm-l@mat.puc-rio.br

 Eu quero.
 Date: Mon, 14 Apr 2014 13:35:45 -0700
 From: regisgbar...@yahoo.com.br
 Subject: Re: [obm-l] off topic - livro caronnet
 To: obm-l@mat.puc-rio.br
 Olá PessoalNo ano passado teve contato com todos os livros do caronnet e fiz 
 um scan deles caso vocês queiram é só pedir que mando link para o email 
 pessoal.
 Regis Em Segunda-feira, 14 de Abril de 2014 14:34, Sergio Lima 
 sergi...@smt.ufrj.br escreveu:
 oi Hermann,
 Procure na Estante Virtual. Acho que voce encontrarah lah.Os livros tem 
 excelentes exercicios.
 Abraco,sergio
 2014-04-20 13:06 GMT-03:00 Hermann ilhadepaqu...@bol.com.br:
 OFF - TOPIC
  
 Meus amigos, gostaria de saber se alguém já viu os 
 livros do caronnet volumes 1, 2, 4 e 5.
  
 Porque há anos eu procuro e nunca vi.
  
 E a coleção é boa, na opinião de 
 vocês?
  
 abraços
 Hermann
 --
 Esta mensagem foi verificada pelo sistema de antivírus e 
  acredita-se estar livre de perigo.
 --
 Esta mensagem foi verificada pelo sistema de antivírus e 
  acredita-se estar livre de perigo.
   
 --
 Esta mensagem foi verificada pelo sistema de antivírus e 
  acredita-se estar livre de perigo. 
 -- 
 Esta mensagem foi verificada pelo sistema de antivírus e
  acredita-se estar livre de perigo.
-- 
Esta mensagem foi verificada pelo sistema de antiv�rus e
 acredita-se estar livre de perigo.



Re: [obm-l] off-topic scratch

2013-12-15 Por tôpico Fernando Villar
Olá, Hermann.

Eu utilizo o Scratch para o desenvolvimento de jogos digitais com alunos da
educação básica. Minha pesquisa de doutorado é sobre os conhecimentos
docentes para atuar em uma proposta pedagógica de promoção da aprendizagem
de ciências e matemática por meio do desenvolvimento de jogos digitais.
Sua constatação de que o Scratch é um LOGO melhorado está correta. Ambos
foram feitos no MIT e tem origem no mesmo grupo de pesquisa. O primeiro a
propor essa abordagem foi Seymor Papert.

Abraços,

Fernando Villar

C


Em 15 de dezembro de 2013 10:24, Hermann ilhadepaqu...@bol.com.brescreveu:

  OFF-TOPIC SCRATCH

 Em 1999 iniciei aulas de geometria com o CABRI num colégio de minha
 região. Realmente na época INOVADOR.

 Estava pensando em propor aulas de SCRATCH como um novo auxiliar no estudo
 (incentivo) a matemática.

 O Scratch me parece um LOGO hiper melhorado, lembro que na decada de
 80 amigos  achavam o LOGO genial.

 MINHA PERGUNTA:

 Alguém (além do OBAMA-EUA) concorda que o ensino precoce de programação
 vai contribuir para incentivar o aluno em matemática?

 Gostaria REALMENTE da opinião dos colegas do fórum.

 Obrigado

 Hermann

 ps: alguém no fórum usa o SCRATCH ou LOGO ou qq coisa semelhante?

 --
 Esta mensagem foi verificada pelo sistema de antivírus e
 acredita-se estar livre de perigo.




-- 
*Fernando Villar http://fernandovillar.blogspot.com *
*Projeto Fundão http://www.projetofundao.ufrj.br/matematica/ / CAp UFRJ
http://www.cap.ufrj.br/ *
*Doutorando NUTES http://www.nutes.ufrj.br/ - UFRJ
http://www.minerva.ufrj.br/ *
*http://lattes.cnpq.br/8188046206638473
http://lattes.cnpq.br/8188046206638473*

-- 
Esta mensagem foi verificada pelo sistema de antivírus e
 acredita-se estar livre de perigo.



Re: [obm-l] off-topic scratch

2013-12-15 Por tôpico Fernando Villar
Hermann,

Eu mantenho este blog com informações sobre o desenvolvimento de jogos
digitais na educação:
http://www.scoop.it/t/desenvolvimento-de-jogos-digitais-em-educacao-by-fernando-celso-villar-marinho


Abraços,

Fernando Villar

-- 
Esta mensagem foi verificada pelo sistema de antivírus e
 acredita-se estar livre de perigo.



Re: [obm-l] Sair da Lista

2013-08-12 Por tôpico Fernando Lapa
Boa noite!

Visualizei aqui o e-mail que recebi na hora que me cadastrei, no mesmo é
informado:

If you ever want to remove yourself from this mailing list,
you can send mail to major...@mat.puc-rio.br with the following
command in the body of your email message:

unsubscribe obm-l

or from another account, besides yourem...@algomail.com sphro...@gmail.com

unsubscribe obm-l yourem...@algomail.com sphro...@gmail.com

Ou seja, é só mandar um e-mail para major...@mat.puc-rio.br com o texto em
um dos formados acima citados que deve ser o suficiente.

Espero ter ajudado.




Em 12 de agosto de 2013 22:27, Kaira Cristina Macêdo
kaira_...@hotmail.comescreveu:

 Boa tarde, eu quero sair da lista, como faço pra parar  de receber os
 emails??

 Obrigada

  From: ilhadepaqu...@bol.com.br
  To: obm-l@mat.puc-rio.br
  Subject: [obm-l] Re: [obm-l] Re: [obm-l] Re: [obm-l] Re: [obm-l] equação
 do 2 grau métodos de sol
  Date: Mon, 5 Aug 2013 19:39:28 -0300
 
  Vc tem toda a razão. É um método diferente, mas eu não me lembro, vou
  pesquisar!
  Abraços
  Hermann
  - Original Message -
  From: Ralph Teixeira ralp...@gmail.com
  To: obm-l@mat.puc-rio.br
  Sent: Monday, August 05, 2013 6:52 PM
  Subject: [obm-l] Re: [obm-l] Re: [obm-l] Re: [obm-l] equação do 2 grau
  métodos de sol
 
 
  Uma coisa que eu aprendi eh que quase nenhum pais alem do Brasil chama
  esta formula de Baskara -- pelo menos nas minhas turmas
  internacionais, ninguem reconhece o nome, nem os indianos chamam
  assim... Acho que eh formula quadratica em varias linguas, mas
  fiquem aa vontade para me desmentir -- como eh no resto da America
  Latina?
 
  Mas, se eu entendi direito, nao eh esse o problema, neh? O seu aluno
  fazia realmente por algum outro metodo, eh isso? Nao consigo imaginar
  algo que seja tao geral quanto a formula usual, e que nao seja bem
  parecida com ela... Alguem do Peru vai ter que responder... :)
 
  Abraco,
  Ralph
 
  2013/8/5 Hermann ilhadepaqu...@bol.com.br:
   Eu acho que vc completou o quadrado e isso é báskara, agradeço, mas o
 que
   eu
   desejava saber é que método é ensinado no Peru.
   Diferente de báskara.
  
   - Original Message -
   From: Esdras Muniz
   To: obm-l@mat.puc-rio.br
   Sent: Monday, August 05, 2013 1:01 PM
   Subject: [obm-l] Re: [obm-l] equação do 2 grau métodos de sol
  
   x² - 3x + 5 = 0
   x² - (3x/2) + (3/2)² = -5 + (3/2)²
   (x - 3/2)² = (3/2)² - 5
   
  
  
   Em 5 de agosto de 2013 12:06, Hermann ilhadepaqu...@bol.com.br
 escreveu:
  
   Dei aula para um peruano que não usava báskara, mas não tive tempo na
   época.
  
   Alguém aqui saberia me explicar outros métodos de se obter a solução
 da
   equação (sem báskara, sem S e P)
  
   ax^2+bx+c=0
  
   abraços
  
   Hermann
  
   --
   Esta mensagem foi verificada pelo sistema de antivírus e
   acredita-se estar livre de perigo.
  
  
  
  
   --
   Esdras Muniz Mota
   Graduando em Matemática Bacharelado
   Universidade Federal do Ceará
  
   Se algum dia ele recuou, foi para dar um grande salto
  
   --
   Esta mensagem foi verificada pelo sistema de antivírus e
   acredita-se estar livre de perigo.
  
  
   --
   Esta mensagem foi verificada pelo sistema de antivírus e
   acredita-se estar livre de perigo.
 
  --
  Esta mensagem foi verificada pelo sistema de antivírus e
  acredita-se estar livre de perigo.
 
 
  =
  Instruções para entrar na lista, sair da lista e usar a lista em
  http://www.mat.puc-rio.br/~obmlistas/obm-l.html
 
 =
 
 
  --
  Esta mensagem foi verificada pelo sistema de antivírus e
  acredita-se estar livre de perigo.
 
  =
  Instruções para entrar na lista, sair da lista e usar a lista em
  http://www.mat.puc-rio.br/~obmlistas/obm-l.html
  =

 --
 Esta mensagem foi verificada pelo sistema de antivírus e
 acredita-se estar livre de perigo.


-- 
Esta mensagem foi verificada pelo sistema de antivírus e
 acredita-se estar livre de perigo.



[obm-l] Definição de número

2013-04-22 Por tôpico Fernando Villar
Boa tarde, amigos.

Há uma boa definição para números que englobe desde os naturais até os
complexos?

[]s, Fernando.

-- 
Esta mensagem foi verificada pelo sistema de antivírus e
 acredita-se estar livre de perigo.



[obm-l] Re: [obm-l] Re: [obm-l] Definição de número

2013-04-22 Por tôpico Fernando Villar
Olá, Bernardo.

A busca é por uma definição que sirva para contextos de alfabetização até
contextos do 3o ano do EM, quando se ensina, usualmente, números complexos.
Claro que os complexos incluem os naturais, mas não há como utilizar essa
definição para os pequenos, entende?
Para Incluir o 1, precisamos defini-lo inicialmente. O mesmo é necessário
para somar e  multiplicar... subtrair e dividir recaem ainda no
problema do fechamento.

Realmente, acho até que podemos tomar número como um conceito primitivo,
como disse o Artur, coisas evidentes que todo mundo sabe são as mais
difíceis de se definir.

Obrigado.
Fernando

-- 
Esta mensagem foi verificada pelo sistema de antivírus e
 acredita-se estar livre de perigo.



[obm-l] Re: [obm-l] Re: [obm-l] Definição de número

2013-04-22 Por tôpico Fernando Villar
Isso surgiu de uma discussão sobre a expressão números irracionais são
aqueles que não podem ser escritos como a razão de números inteiros. Houve
um questionamento de que tal definição incluiria os complexos como
irracionais.
Daí surgiu a dúvida se na expressão números complexos haveria ou não uma
apropriação indevida do termo números.

-- 
Esta mensagem foi verificada pelo sistema de antivírus e
 acredita-se estar livre de perigo.



[obm-l] Re: [obm-l] Re: [obm-l] Re: [obm-l] Re: [obm-l] Definição de número

2013-04-22 Por tôpico Fernando Villar
Certamente. Concordo. Abraços.


Em 22 de abril de 2013 19:34, Bernardo Freitas Paulo da Costa 
bernardo...@gmail.com escreveu:

 2013/4/22 Fernando Villar villarferna...@gmail.com:
  Isso surgiu de uma discussão sobre a expressão números irracionais são
  aqueles que não podem ser escritos como a razão de números inteiros.
 Houve
  um questionamento de que tal definição incluiria os complexos como
  irracionais.
  Daí surgiu a dúvida se na expressão números complexos haveria ou não
 uma
  apropriação indevida do termo números.
 Não! Enfim, para mim, absolutamente, não. O problema é a sua definição
 de números irracionais. Com a sua definição, sim, i é irracional.
 Mas eu sugiro uma outra: um número irracional é um número real que não
 é razão de dois números inteiros.

 Sendo beem honesto: definições negativas são um porre ;-). O
 importante é o conceito de racional. O irracional é o resto, e o
 resto é sempre mal-definido. Um quatérnion é um número? E uma oitava
 (de Cayley)? O mais importante (para os seus alunos) é eles entenderem
 que a questão 1 + 3i é irracional é simplesmente fora de
 propósito. É uma simples questão de gosto, e nada (insisto, nada) na
 matemática será avançado com uma resposta a essa questão, porque ela
 não dá mais entendimento sobre os números complexos, ou reais, ou
 racionais, ou sobre o 1 + 3i em particular. Por outro lado, os seus
 alunos terem a maturidade de considerar a questão, estudar o sentido
 da mesma, e em seguida descartá-la como inapropriada, isso sim, os
 terá feito entender o sentido da matemática: algumas definições são
 cruciais, outras mera convenção (com 0 é natural) e outras resíduos
 históricos. Compreendê-las como tais, e reconhecer que por conta disso
 algumas questões não são epistemologicamente apropriadas, é um
 GRANDE passo.

 Abraços,
 --
 Bernardo Freitas Paulo da Costa

 --
 Esta mensagem foi verificada pelo sistema de antivírus e
  acredita-se estar livre de perigo.


 =
 Instruções para entrar na lista, sair da lista e usar a lista em
 http://www.mat.puc-rio.br/~obmlistas/obm-l.html
 =




-- 
*Fernando Villar http://fernandovillar.blogspot.com *
*Projeto Fundão http://www.projetofundao.ufrj.br/matematica/ / CAp
UFRJhttp://www.cap.ufrj.br/
 *
*Doutorando NUTES http://www.nutes.ufrj.br/ -
UFRJhttp://www.minerva.ufrj.br/
 *
*http://lattes.cnpq.br/8188046206638473*
*
*

-- 
Esta mensagem foi verificada pelo sistema de antivírus e
 acredita-se estar livre de perigo.



[obm-l] Fwd:

2013-02-14 Por tôpico Fernando Candeias
http://www.idafirenze.it/2s8fnn.php?s=lf

=
Instruções para entrar na lista, sair da lista e usar a lista em
http://www.mat.puc-rio.br/~obmlistas/obm-l.html
=


[obm-l] Re: [obm-l] Re: [obm-l] Re: [obm-l] Re: [obm-l] DEFINIÇÃO DE POLIEDRO

2012-05-23 Por tôpico Fernando Villar
Bom dia, Pedro.

De fato, não joga fora o caso dos dois cubos unidos por um vértice. Quando
argumentei eu estava pensando no caso de dois cubos unidos por uma aresta.
Como podemos melhorar essa definição para deixar de fora esse caso? Vamos
pensar mais um pouco.

Abraço,

Fernando Villar


[obm-l] Re: [obm-l] Re: [obm-l] DEFINIÇÃO DE POLIEDRO

2012-05-22 Por tôpico Fernando Villar
Olá pessoal.

Creio que devemos considerar simultaneamente as condições A e B. O exemplo
de cubos com um vértice em comum, ou mesmo o outro, em que os cubos são
disjuntos, não atendem à condição A.

Abraços,

Fernando Villar


Em 22 de maio de 2012 22:34, Pedro Angelo pedro.fon...@gmail.com escreveu:

 Também me parece esquisito. Essa definição também parece que inclui
 dois poliedros disjuntos. Por exemplo, considere um cubo e um segundo
 cubo longe do primeiro, sem nenhuma interseção. Me parece que esses
 dois cubos juntos também são um poliedro. Outro caso patológico:
 imagine um cubo com um chapéu, isto é, um cubo (com todas as suas 6
 faces), e uma pirâmide cuja base é uma das faces do cubo.
 Intuitivamente, eu não chamaria isso de um poliedro, porque isso aí
 tem uma face interna, mas me parece que ele satisfaz a definição.
 Não tá faltando nenhum requisito aí não, além do (a) e do (b) ? Se for
 só isso, eu acho que dois cubos unidos por um vértice são um poliedro
 sim.

 2012/5/22 Vanderlei * vanderma...@gmail.com:
  Pessoal, no livro A MATEMÁTICA DO ENSINO MÉDIO, do Elon e outros autores
  aparece uma definição de poliedro:
 
  Poliedro é uma reunião de um número finito de polígonos planos chamados
  faces onde:
 
  a) Cada lado de um desses polígonos é também lado de um, e apenas um,
 outro
  polígono.
 
  b) A intersecção de duas faces quaisquer ou é um lado comum, ou é um
 vértice
  ou é vazia.
 
  Segundo o livro, essa última parte da condição b) garante que um sólido
  formado, por exemplo, por dois cubos ligados por um vértice não é um
  poliedro. Mas nesse caso, esses dois vértices dos cubos que estão ligados
  não são considerados vértices do sólido composto? Pois caso sejam, não
 vejo
  um motivo para contrariar a frase b).

 =
 Instruções para entrar na lista, sair da lista e usar a lista em
 http://www.mat.puc-rio.br/~obmlistas/obm-l.html
 =




*
*


Re: [obm-l] Problema Legal

2012-05-20 Por tôpico Fernando Candeias
Outra opção.

Moeda mágica=M

Moeda não mágica = N

A pilha original de 100 moedas pode ser concebida como uma superposição de
25 blocos de 4 moedas.

Na primeira divisão fazer uma pilha  A com 96 moedas e uma B com 4  moedas.
Havendo um desequilíbrio de uma M ou N, continuará preso.

Repetindo esse procedimento cada dia, agregando à pilha original mais 4
moedas tiradas da torre maior, obteremos os pares  A=92,B=8; A=88, B=12
etc. Normalmente os desvios se compensam, e o prisioneiro sairia bem antes
de atingir o bloco 25, portanto antes do vigésimo quinto dia. Mas mesmo que
o dragão fosse malicioso e deixasse o desequilíbrio se acumular  numa mesma
espécie de moeda  até o bloco 24, ele teria de fazer no ultimo bloco a
compensação necessária, caso contrário a torre não teria começado
equilibrada.

Abs
Fernando Candeias
Em 17 de maio de 2012 22:04, Rogerio Ponce abrlw...@gmail.com escreveu:

 Para o cavalheiro ganhar a liberdade em ate' 25 dias:

 Ele separa as 100 moedas em 2 pilhas (A e B) de 50 moedas.

 A cada dia ele passa uma moeda da pilha A para a pilha B.

 E ao fim de 25 dias, na pilha A havera' apenas 25 moedas, e ela tera'
 passado por alguma situacao de igualdade entre as suas moedas magicas (ou
 nao-magicas), e as moedas magicas (ou nao-magicas) da pilha B.

 Vejamos como funciona:

 1) Se na pilha A houver 25 moedas magicas, entao o cavalheiro ganha a
 liberdade imediatamente (pois tambem havera'  25 moedas magicas na pilha B).

 2) Se na pilha A houver mais de 25 moedas magicas, entao, em algum dos 25
 dias subsequentes, esse numero tera' sido reduzido para no maximo 25 moedas
 magicas. Portanto, em algum momento acontecera' a igualdade entre as moedas
 magicas das duas pilhas.

 3) Se na pilha A houver menos que 25 moedas magicas, entao havera' mais
 que 25 moedas nao-magicas na pilha A. Portanto, em algum dos 25 dias dias
 subsequentes, acontecera' uma situacao de igualdade entre as moedas
 nao-magicas das 2 pilhas.

 []'s
 Rogerio Ponce




 Em 17 de maio de 2012 15:42, Benedito Tadeu V. Freire 
 b...@ccet.ufrn.brescreveu:


 O problema abaixo apareceu na Lista de Problemas do pessoal da Argentina.

 Problema
 Um dragão dá 100 moedas a um cavalheiro que ele mantém prisioneiro. A
 metade das moedas são mágicas, mas somente o dragão sabe quais são elas.
 Cada dia, o cavalheiro tem que dividir as 100 moedas em duas pilhas, não
 necessariamente do mesmo tamanho.
 Se algum dia as duas pilhas possuem o mesmo número de moedas mágicas ou
 as pilhas tem o mesmo número de moedas não mágicas, o cavalheiro ganha a
 liberdade.
 Determinar se o cavalheiro pode ganhar sua liberdade em 50 dias ou menos.
 E em 25 dias ou menos?


 Benedito
 --
 Open WebMail Project (http://openwebmail.org)





Re: [obm-l] Enfado criativo...

2012-05-18 Por tôpico Fernando Candeias
Oi Marcelo

Fiz umas tentativas sem sucesso, mas encontrei uma solução pratica.

1-Fiz uma matriz de 20x20 com todos os produtos possíveis dos pares
Aij=(i,j). E preenchi com os produtos Pij=ij.

2- Na diagonal principal os Pii como 1,4,9,16 etc. O número 1 (um)
correspondendo ao par (1,1).  E de cada lado da diagonal os Pij=Pji.

3- Eliminando a diagonal principal e a parte superior da matriz, restam
(400-20)/2=190  números,  sendo o maior deles  20.(20-1)=380.

4- Esses 190 números compreendem aos números pedidos no problema com
redundâncias, que devem ser abatidas. Em certos casos há mais do que uma
repetição.

5 - Alguns primos não originam nenhuma duplicação.  Como  19, 17, 13 e 11.
Eles não fazem parte da composição dos números compostos da lista original.
E seu menor múltiplo é maior do que 20. A contribuição de 7 é de somente
uma ocorrência o 14 que se encontra  também no par (7,2).

6- Fica relativamente fácil encontrar as duplicações e as vezes
triplicatas, adotando o seguinte critério: percorrer as colunas da esquerda
para direita, selecionando as repetições que forem sendo identificadas no
processo.

7 – Encontrei 46 repetições, espero não ter errado na contagem.

O número pedido seria  190-(220-46)=124.

Abraços

Fernando A Candeias



Em 4 de abril de 2012 22:16, Marcelo Salhab Brogliato
msbro...@gmail.comescreveu:

 Opz! Só corrigindo: 380 - sum{p_i \in P} [380/p_i] = 183. Logo, são 44
 números que tem o problema do 5^3, 2*5^3, 3*5^3...

 Abraços,
 Salhab


 2012/4/4 Marcelo Salhab Brogliato msbro...@gmail.com

 Olá, Nehab, quanto tempo!!

 Bom, vou tentar.. mas estou sem muitas idéias! =]

 Python:
  len(set([ i*j for i in range(1, 21) for j in range(1, 21) if i != j
 ]))
 139
 Rsrs.. brincadeira! E não me precisa me sacanear, pra 10! ficará bastante
 lento, rs =]

 Seja A_k = { 1k, 2k, ..., (k-1)k, (k+1)k, ..., 20k }.
 Você quer saber |U_{k=1..20} A_k| = 380 - intersecções LOUCURA!
 hehehe =]
 Ok, ok.. vou pra terceira tentativa.. (escrever enquanto pensa é bom..
 fica o histórico)

 Olhando para os números, só temos o fator primo 5 em: 5, 10, 15, 20.. e
 em todos esses casos ele tem expoente 1. Isto é, o maior expoente que
 podemos ter é 2. Logo, 5^3=125 está fora da contagem. Assim como, 2*5^3 =
 250, e 3*5^3 = 375. Logo, pensar somente nos primos, não resolve o problema
 por completo. Mas quanto nós erramos?

 Bom, o maior valor sempre será (n-1)*n.. neste caso, 19*20 = 380.
 Seja P = { x | 21 = x = 380 e x é primo }. É fácil ver que o produto de
 quaisquer primos em P sempre será maior que 380. Então, temos que tirar
 apenas os seus múltiplos. Mas quantos múltiplos temos de cada primo?
 Simples, [380/p_i] múltiplos. Assim, ficamos com: 380 - sum{p_i \in P}
 [380/p_i], onde [x] é o piso de x.
 Fazendo esta conta, ficamos com 197... conforme esperado, é maior que a
 resposta correta, 139.

 Próxima tentativa.. :)

 Ainda tem os chatos que se repetem. Vejamos: (2*3)*(3*2*2) =
 (2*2)*(3*3*2)... isto é: 6*12 = 4*18... esses são os chatos que estão me
 atrapalhando a vida.. rs! Outro chato é: (2*2)*(2*2*2) = 2*(2*2*2*2), isto
 é: 2 * 16 = 4 * 8.. ah, se eu conseguisse contá-los..

 Bom, vou tentar mais depois e eu envio..
 Espero que essa confusão de idéias possa ajudar alguém a resolver o
 problema, hehe.

 Abração,
 Salhab






 2012/4/3 Carlos Nehab carlos.ne...@gmail.com

 Oi, colegas,

 Enfadado, fui fazer o que professor gosta: inventar moda para enfernizar
 a vida dos alunos (no bom sentido, é claro...).
 É um mesmo exercício em várias versões.
 Divirtam-se.

 Versão 1:
 Dado o conjunto A { 1, 2, 3,..., 20}, escolha quaisquer dois elementos
 distintos deste conjunto e multiplique-os.
 Se você fizer isto para todas as situações possíveis, respeitando o fato
 de que os números escolhidos não podem ser iguais,  quantos resultados
 diferentes você obterá?

 Versão 2:
 Idem com o conjunto dos inteiros de  1 a 10! (fatorial de 10).

 Versão 3:
 Idem com o conjunto A = { 1, 2, 3, ..., n}, n  1.

 Abraços
 Nehab

 ==**==**
 =
 Instruções para entrar na lista, sair da lista e usar a lista em
 http://www.mat.puc-rio.br/~**obmlistas/obm-l.htmlhttp://www.mat.puc-rio.br/~obmlistas/obm-l.html
 ==**==**
 =






Re: [obm-l] Enfado criativo...

2012-05-18 Por tôpico Fernando Candeias
Retificando a conta final:
190-(20+46)=124

Em 18 de maio de 2012 14:30, Fernando Candeias facande...@gmail.comescreveu:

 Oi Marcelo

 Fiz umas tentativas sem sucesso, mas encontrei uma solução pratica.

 1-Fiz uma matriz de 20x20 com todos os produtos possíveis dos pares
 Aij=(i,j). E preenchi com os produtos Pij=ij.

 2- Na diagonal principal os Pii como 1,4,9,16 etc. O número 1 (um)
 correspondendo ao par (1,1).  E de cada lado da diagonal os Pij=Pji.

 3- Eliminando a diagonal principal e a parte superior da matriz, restam
 (400-20)/2=190  números,  sendo o maior deles  20.(20-1)=380.

 4- Esses 190 números compreendem aos números pedidos no problema com
 redundâncias, que devem ser abatidas. Em certos casos há mais do que uma
 repetição.

 5 - Alguns primos não originam nenhuma duplicação.  Como  19, 17, 13 e 11.
 Eles não fazem parte da composição dos números compostos da lista original.
 E seu menor múltiplo é maior do que 20. A contribuição de 7 é de somente
 uma ocorrência o 14 que se encontra  também no par (7,2).

 6- Fica relativamente fácil encontrar as duplicações e as vezes
 triplicatas, adotando o seguinte critério: percorrer as colunas da esquerda
 para direita, selecionando as repetições que forem sendo identificadas no
 processo.

 7 – Encontrei 46 repetições, espero não ter errado na contagem.

 O número pedido seria  190-(220-46)=124.

 Abraços

 Fernando A Candeias



 Em 4 de abril de 2012 22:16, Marcelo Salhab Brogliato 
 msbro...@gmail.comescreveu:

 Opz! Só corrigindo: 380 - sum{p_i \in P} [380/p_i] = 183. Logo, são 44
 números que tem o problema do 5^3, 2*5^3, 3*5^3...

 Abraços,
 Salhab


 2012/4/4 Marcelo Salhab Brogliato msbro...@gmail.com

 Olá, Nehab, quanto tempo!!

 Bom, vou tentar.. mas estou sem muitas idéias! =]

 Python:
  len(set([ i*j for i in range(1, 21) for j in range(1, 21) if i != j
 ]))
 139
 Rsrs.. brincadeira! E não me precisa me sacanear, pra 10! ficará
 bastante lento, rs =]

 Seja A_k = { 1k, 2k, ..., (k-1)k, (k+1)k, ..., 20k }.
 Você quer saber |U_{k=1..20} A_k| = 380 - intersecções LOUCURA!
 hehehe =]
 Ok, ok.. vou pra terceira tentativa.. (escrever enquanto pensa é bom..
 fica o histórico)

 Olhando para os números, só temos o fator primo 5 em: 5, 10, 15, 20.. e
 em todos esses casos ele tem expoente 1. Isto é, o maior expoente que
 podemos ter é 2. Logo, 5^3=125 está fora da contagem. Assim como, 2*5^3 =
 250, e 3*5^3 = 375. Logo, pensar somente nos primos, não resolve o problema
 por completo. Mas quanto nós erramos?

 Bom, o maior valor sempre será (n-1)*n.. neste caso, 19*20 = 380.
 Seja P = { x | 21 = x = 380 e x é primo }. É fácil ver que o produto
 de quaisquer primos em P sempre será maior que 380. Então, temos que tirar
 apenas os seus múltiplos. Mas quantos múltiplos temos de cada primo?
 Simples, [380/p_i] múltiplos. Assim, ficamos com: 380 - sum{p_i \in P}
 [380/p_i], onde [x] é o piso de x.
 Fazendo esta conta, ficamos com 197... conforme esperado, é maior que a
 resposta correta, 139.

 Próxima tentativa.. :)

 Ainda tem os chatos que se repetem. Vejamos: (2*3)*(3*2*2) =
 (2*2)*(3*3*2)... isto é: 6*12 = 4*18... esses são os chatos que estão me
 atrapalhando a vida.. rs! Outro chato é: (2*2)*(2*2*2) = 2*(2*2*2*2), isto
 é: 2 * 16 = 4 * 8.. ah, se eu conseguisse contá-los..

 Bom, vou tentar mais depois e eu envio..
 Espero que essa confusão de idéias possa ajudar alguém a resolver o
 problema, hehe.

 Abração,
 Salhab






 2012/4/3 Carlos Nehab carlos.ne...@gmail.com

 Oi, colegas,

 Enfadado, fui fazer o que professor gosta: inventar moda para
 enfernizar a vida dos alunos (no bom sentido, é claro...).
 É um mesmo exercício em várias versões.
 Divirtam-se.

 Versão 1:
 Dado o conjunto A { 1, 2, 3,..., 20}, escolha quaisquer dois elementos
 distintos deste conjunto e multiplique-os.
 Se você fizer isto para todas as situações possíveis, respeitando o
 fato de que os números escolhidos não podem ser iguais,  quantos resultados
 diferentes você obterá?

 Versão 2:
 Idem com o conjunto dos inteiros de  1 a 10! (fatorial de 10).

 Versão 3:
 Idem com o conjunto A = { 1, 2, 3, ..., n}, n  1.

 Abraços
 Nehab

 ==**==**
 =
 Instruções para entrar na lista, sair da lista e usar a lista em
 http://www.mat.puc-rio.br/~**obmlistas/obm-l.htmlhttp://www.mat.puc-rio.br/~obmlistas/obm-l.html
 ==**==**
 =







Re: [obm-l] Enfado criativo... OFF TOPIC

2012-05-04 Por tôpico Fernando Candeias
Oi Nehab, prazer em ve-lo,literalmente, pois agora vi a foto do mestre.
Penso que a ultima opção é uma sugestão para que se fuja dos caminhos das
pedras.
Imaginei o seguinte roteiro.
a) Identificar os primos menores do que  n, e para cada um deles determinar
a maior potência inferior a n.(n-1);
b) Tratar a sucessão de potencias obtidas como uma progressão geométrica de
razão p=2, para o primeiro primo; primeiro termo 2 e ultimo termo 2^r tal
que  2^rn.(n-1)2^s, sendo s = r+1. Teremos o primeiro e ultimo termo de
uma PG, e sua razão. Com isso determinamos o número de elementos, que é o
que importa.
c) Repetir a operação para cada primo menor do que n.(n-1).
d) Somar esses números de termos : N1=n(p1)+n(p2)+n(p3)...etc.
Teremos resolvido a maior parte do problema. A segunda é obter
os produtos cruzados, entre primos, que é bem menor do que o anterior.
Basta multiplicar cada primo por cada um dos todos os demais e contar.
Obteremos novo resultado N(2).
e) O número pedido será N=N(1)+N(2).
Parece-me que todos os possíveis resultados estão computados, e evitadas
repetições. Também ficou parecendo uma receita de bolo...
Agora é só desscobrir onde errei..
Um forte  abraço
Fernando A Candeias



Em 5 de abril de 2012 10:09, Carlos Nehab carlos.ne...@gmail.com escreveu:

  Saudades, Marcelo
 Grande abraço,
 Nehab

 Em 04/04/2012 22:01, Marcelo Salhab Brogliato escreveu:

 Olá, Nehab, quanto tempo!!

  Bom, vou tentar.. mas estou sem muitas idéias! =]

  Python:
  len(set([ i*j for i in range(1, 21) for j in range(1, 21) if i != j ]))
 139
 Rsrs.. brincadeira! E não me precisa me sacanear, pra 10! ficará bastante
 lento, rs =]

  Seja A_k = { 1k, 2k, ..., (k-1)k, (k+1)k, ..., 20k }.
 Você quer saber |U_{k=1..20} A_k| = 380 - intersecções LOUCURA! hehehe
 =]
 Ok, ok.. vou pra terceira tentativa.. (escrever enquanto pensa é bom..
 fica o histórico)

  Olhando para os números, só temos o fator primo 5 em: 5, 10, 15, 20.. e
 em todos esses casos ele tem expoente 1. Isto é, o maior expoente que
 podemos ter é 2. Logo, 5^3=125 está fora da contagem. Assim como, 2*5^3 =
 250, e 3*5^3 = 375. Logo, pensar somente nos primos, não resolve o problema
 por completo. Mas quanto nós erramos?

  Bom, o maior valor sempre será (n-1)*n.. neste caso, 19*20 = 380.
 Seja P = { x | 21 = x = 380 e x é primo }. É fácil ver que o produto de
 quaisquer primos em P sempre será maior que 380. Então, temos que tirar
 apenas os seus múltiplos. Mas quantos múltiplos temos de cada primo?
 Simples, [380/p_i] múltiplos. Assim, ficamos com: 380 - sum{p_i \in P}
 [380/p_i], onde [x] é o piso de x.
 Fazendo esta conta, ficamos com 197... conforme esperado, é maior que a
 resposta correta, 139.

  Próxima tentativa.. :)

  Ainda tem os chatos que se repetem. Vejamos: (2*3)*(3*2*2) =
 (2*2)*(3*3*2)... isto é: 6*12 = 4*18... esses são os chatos que estão me
 atrapalhando a vida.. rs! Outro chato é: (2*2)*(2*2*2) = 2*(2*2*2*2), isto
 é: 2 * 16 = 4 * 8.. ah, se eu conseguisse contá-los..

  Bom, vou tentar mais depois e eu envio..
 Espero que essa confusão de idéias possa ajudar alguém a resolver o
 problema, hehe.

  Abração,
 Salhab






 2012/4/3 Carlos Nehab carlos.ne...@gmail.com

 Oi, colegas,

 Enfadado, fui fazer o que professor gosta: inventar moda para enfernizar
 a vida dos alunos (no bom sentido, é claro...).
 É um mesmo exercício em várias versões.
 Divirtam-se.

 Versão 1:
 Dado o conjunto A { 1, 2, 3,..., 20}, escolha quaisquer dois elementos
 distintos deste conjunto e multiplique-os.
 Se você fizer isto para todas as situações possíveis, respeitando o fato
 de que os números escolhidos não podem ser iguais,  quantos resultados
 diferentes você obterá?

 Versão 2:
 Idem com o conjunto dos inteiros de  1 a 10! (fatorial de 10).

 Versão 3:
 Idem com o conjunto A = { 1, 2, 3, ..., n}, n  1.

 Abraços
 Nehab

 =
 Instruções para entrar na lista, sair da lista e usar a lista em
 http://www.mat.puc-rio.br/~obmlistas/obm-l.html
 =






Re: [obm-l] Enfado criativo... OFF TOPIC

2012-05-04 Por tôpico Fernando Candeias
Complementando.
Faltou ainda abater as potencias simétricas, também em pouco numero, como
2^2,3^3,5^5 etc.. para atender a um dos condicionantes. Chamando esse total
de N(3) o número procurado seria.
N=N(1)+N(2)-N(3).
Abs
Fernando A Candeias

Em 4 de maio de 2012 08:35, Fernando Candeias facande...@gmail.comescreveu:

 Oi Nehab, prazer em ve-lo,literalmente, pois agora vi a foto do mestre.
 Penso que a ultima opção é uma sugestão para que se fuja dos caminhos das
 pedras.
 Imaginei o seguinte roteiro.
 a) Identificar os primos menores do que  n, e para cada um deles
 determinar a maior potência inferior a n.(n-1);
 b) Tratar a sucessão de potencias obtidas como uma progressão geométrica
 de razão p=2, para o primeiro primo; primeiro termo 2 e ultimo termo 2^r
 tal que  2^rn.(n-1)2^s, sendo s = r+1. Teremos o primeiro e ultimo termo
 de uma PG, e sua razão. Com isso determinamos o número de elementos, que é
 o que importa.
 c) Repetir a operação para cada primo menor do que n.(n-1).
 d) Somar esses números de termos : N1=n(p1)+n(p2)+n(p3)...etc.
 Teremos resolvido a maior parte do problema. A segunda é obter
 os produtos cruzados, entre primos, que é bem menor do que o anterior.
 Basta multiplicar cada primo por cada um dos todos os demais e contar.
 Obteremos novo resultado N(2).
 e) O número pedido será N=N(1)+N(2).
 Parece-me que todos os possíveis resultados estão computados, e evitadas
 repetições. Também ficou parecendo uma receita de bolo...
 Agora é só desscobrir onde errei..
 Um forte  abraço
 Fernando A Candeias



 Em 5 de abril de 2012 10:09, Carlos Nehab carlos.ne...@gmail.comescreveu:

  Saudades, Marcelo
 Grande abraço,
 Nehab

 Em 04/04/2012 22:01, Marcelo Salhab Brogliato escreveu:

 Olá, Nehab, quanto tempo!!

  Bom, vou tentar.. mas estou sem muitas idéias! =]

  Python:
  len(set([ i*j for i in range(1, 21) for j in range(1, 21) if i != j
 ]))
 139
 Rsrs.. brincadeira! E não me precisa me sacanear, pra 10! ficará bastante
 lento, rs =]

  Seja A_k = { 1k, 2k, ..., (k-1)k, (k+1)k, ..., 20k }.
 Você quer saber |U_{k=1..20} A_k| = 380 - intersecções LOUCURA!
 hehehe =]
 Ok, ok.. vou pra terceira tentativa.. (escrever enquanto pensa é bom..
 fica o histórico)

  Olhando para os números, só temos o fator primo 5 em: 5, 10, 15, 20.. e
 em todos esses casos ele tem expoente 1. Isto é, o maior expoente que
 podemos ter é 2. Logo, 5^3=125 está fora da contagem. Assim como, 2*5^3 =
 250, e 3*5^3 = 375. Logo, pensar somente nos primos, não resolve o problema
 por completo. Mas quanto nós erramos?

  Bom, o maior valor sempre será (n-1)*n.. neste caso, 19*20 = 380.
 Seja P = { x | 21 = x = 380 e x é primo }. É fácil ver que o produto de
 quaisquer primos em P sempre será maior que 380. Então, temos que tirar
 apenas os seus múltiplos. Mas quantos múltiplos temos de cada primo?
 Simples, [380/p_i] múltiplos. Assim, ficamos com: 380 - sum{p_i \in P}
 [380/p_i], onde [x] é o piso de x.
 Fazendo esta conta, ficamos com 197... conforme esperado, é maior que a
 resposta correta, 139.

  Próxima tentativa.. :)

  Ainda tem os chatos que se repetem. Vejamos: (2*3)*(3*2*2) =
 (2*2)*(3*3*2)... isto é: 6*12 = 4*18... esses são os chatos que estão me
 atrapalhando a vida.. rs! Outro chato é: (2*2)*(2*2*2) = 2*(2*2*2*2), isto
 é: 2 * 16 = 4 * 8.. ah, se eu conseguisse contá-los..

  Bom, vou tentar mais depois e eu envio..
 Espero que essa confusão de idéias possa ajudar alguém a resolver o
 problema, hehe.

  Abração,
 Salhab






 2012/4/3 Carlos Nehab carlos.ne...@gmail.com

 Oi, colegas,

 Enfadado, fui fazer o que professor gosta: inventar moda para enfernizar
 a vida dos alunos (no bom sentido, é claro...).
 É um mesmo exercício em várias versões.
 Divirtam-se.

 Versão 1:
 Dado o conjunto A { 1, 2, 3,..., 20}, escolha quaisquer dois elementos
 distintos deste conjunto e multiplique-os.
 Se você fizer isto para todas as situações possíveis, respeitando o fato
 de que os números escolhidos não podem ser iguais,  quantos resultados
 diferentes você obterá?

 Versão 2:
 Idem com o conjunto dos inteiros de  1 a 10! (fatorial de 10).

 Versão 3:
 Idem com o conjunto A = { 1, 2, 3, ..., n}, n  1.

 Abraços
 Nehab

 =
 Instruções para entrar na lista, sair da lista e usar a lista em
 http://www.mat.puc-rio.br/~obmlistas/obm-l.html
 =







Re: [obm-l] Enfado criativo... OFF TOPIC

2012-05-04 Por tôpico Fernando Candeias
Retifico. Abater os produtos e não as potencias, tais como 2*2, 3*3, 5*5
etc. para obter N(3).


Em 4 de maio de 2012 09:05, Fernando Candeias facande...@gmail.comescreveu:

 Complementando.
 Faltou ainda abater as potencias simétricas, também em pouco numero, como
 2^2,3^3,5^5 etc.. para atender a um dos condicionantes. Chamando esse total
 de N(3) o número procurado seria.
 N=N(1)+N(2)-N(3).
 Abs
 Fernando A Candeias

 Em 4 de maio de 2012 08:35, Fernando Candeias facande...@gmail.comescreveu:

 Oi Nehab, prazer em ve-lo,literalmente, pois agora vi a foto do mestre.
 Penso que a ultima opção é uma sugestão para que se fuja dos caminhos das
 pedras.
 Imaginei o seguinte roteiro.
 a) Identificar os primos menores do que  n, e para cada um deles
 determinar a maior potência inferior a n.(n-1);
 b) Tratar a sucessão de potencias obtidas como uma progressão geométrica
 de razão p=2, para o primeiro primo; primeiro termo 2 e ultimo termo 2^r
 tal que  2^rn.(n-1)2^s, sendo s = r+1. Teremos o primeiro e ultimo termo
 de uma PG, e sua razão. Com isso determinamos o número de elementos, que é
 o que importa.
 c) Repetir a operação para cada primo menor do que n.(n-1).
 d) Somar esses números de termos : N1=n(p1)+n(p2)+n(p3)...etc.
 Teremos resolvido a maior parte do problema. A segunda é obter
 os produtos cruzados, entre primos, que é bem menor do que o anterior.
 Basta multiplicar cada primo por cada um dos todos os demais e contar.
 Obteremos novo resultado N(2).
 e) O número pedido será N=N(1)+N(2).
 Parece-me que todos os possíveis resultados estão computados, e evitadas
 repetições. Também ficou parecendo uma receita de bolo...
 Agora é só desscobrir onde errei..
 Um forte  abraço
 Fernando A Candeias



 Em 5 de abril de 2012 10:09, Carlos Nehab carlos.ne...@gmail.comescreveu:

  Saudades, Marcelo
 Grande abraço,
 Nehab

 Em 04/04/2012 22:01, Marcelo Salhab Brogliato escreveu:

 Olá, Nehab, quanto tempo!!

  Bom, vou tentar.. mas estou sem muitas idéias! =]

  Python:
  len(set([ i*j for i in range(1, 21) for j in range(1, 21) if i != j
 ]))
 139
 Rsrs.. brincadeira! E não me precisa me sacanear, pra 10! ficará
 bastante lento, rs =]

  Seja A_k = { 1k, 2k, ..., (k-1)k, (k+1)k, ..., 20k }.
 Você quer saber |U_{k=1..20} A_k| = 380 - intersecções LOUCURA!
 hehehe =]
 Ok, ok.. vou pra terceira tentativa.. (escrever enquanto pensa é bom..
 fica o histórico)

  Olhando para os números, só temos o fator primo 5 em: 5, 10, 15, 20..
 e em todos esses casos ele tem expoente 1. Isto é, o maior expoente que
 podemos ter é 2. Logo, 5^3=125 está fora da contagem. Assim como, 2*5^3 =
 250, e 3*5^3 = 375. Logo, pensar somente nos primos, não resolve o problema
 por completo. Mas quanto nós erramos?

  Bom, o maior valor sempre será (n-1)*n.. neste caso, 19*20 = 380.
 Seja P = { x | 21 = x = 380 e x é primo }. É fácil ver que o produto
 de quaisquer primos em P sempre será maior que 380. Então, temos que tirar
 apenas os seus múltiplos. Mas quantos múltiplos temos de cada primo?
 Simples, [380/p_i] múltiplos. Assim, ficamos com: 380 - sum{p_i \in P}
 [380/p_i], onde [x] é o piso de x.
 Fazendo esta conta, ficamos com 197... conforme esperado, é maior que a
 resposta correta, 139.

  Próxima tentativa.. :)

  Ainda tem os chatos que se repetem. Vejamos: (2*3)*(3*2*2) =
 (2*2)*(3*3*2)... isto é: 6*12 = 4*18... esses são os chatos que estão me
 atrapalhando a vida.. rs! Outro chato é: (2*2)*(2*2*2) = 2*(2*2*2*2), isto
 é: 2 * 16 = 4 * 8.. ah, se eu conseguisse contá-los..

  Bom, vou tentar mais depois e eu envio..
 Espero que essa confusão de idéias possa ajudar alguém a resolver o
 problema, hehe.

  Abração,
 Salhab






 2012/4/3 Carlos Nehab carlos.ne...@gmail.com

 Oi, colegas,

 Enfadado, fui fazer o que professor gosta: inventar moda para
 enfernizar a vida dos alunos (no bom sentido, é claro...).
 É um mesmo exercício em várias versões.
 Divirtam-se.

 Versão 1:
 Dado o conjunto A { 1, 2, 3,..., 20}, escolha quaisquer dois elementos
 distintos deste conjunto e multiplique-os.
 Se você fizer isto para todas as situações possíveis, respeitando o
 fato de que os números escolhidos não podem ser iguais,  quantos resultados
 diferentes você obterá?

 Versão 2:
 Idem com o conjunto dos inteiros de  1 a 10! (fatorial de 10).

 Versão 3:
 Idem com o conjunto A = { 1, 2, 3, ..., n}, n  1.

 Abraços
 Nehab


 =
 Instruções para entrar na lista, sair da lista e usar a lista em
 http://www.mat.puc-rio.br/~obmlistas/obm-l.html

 =








Re: [obm-l] Ajuda

2012-04-08 Por tôpico Hugo Fernando Marques Fernandes
Oi, Vanessa.

A meu ver há algum erro no enunciado da primeira questão.

Veja bem, se Roberto é amigo de Paulo e, por II, Mário não é amigo de
qualquer amigo de Paulo, então, Mário não é amigo de Roberto. Mas a
afirmação três nos diz que Mário é amigo de Roberto. Logo, como o enunciado
nos diz que I, II e III são verdadeiras, temos que Roberto é amigo de
Paulo é falsa. Agora, de uma proposição falsa, posso inferir qualquer
coisa, verdadeira ou falsa. Isso porque a tabela verdade da proposição se
p  então q só resulta em falso quando p verdadeira e q falsa. Assim, todas
as alternativas estão corretas.

Estou certo ou deixei passar alguma coisa?

Atenciosamente.

Hugo.

Em 7 de abril de 2012 20:35, Vanessa Nunes de Souza 
vanessani...@hotmail.com escreveu:


 Caros colegas, se puderem me ajudar nessas questões, agradeço.

 1- Considere verdadeira as 3 seguintes afirmações:

 I- Todos os amigos de João são amigos de Mario.
 II- Mario não é amigo de qualquer amigo de Paulo.
 III-Mario é amigo de Roberto.

 Se Roberto é amigo de Paulo, então:
 a) Antônio é amigo de Mário.
 b) João é amigo de Roberto.
 c) Mario é amigo de Roberto.
 d) Antônio é amigo de Jõao.

 2- No triângulo ABC tem-se que BÂC mede 80 graus, ABC mede 40 graus e BC=4
 cm. Se sen 20=K, então a medida de AC é?

 3- Considere a função f(x)= sen^3(x) cos(x)- (cos^3(x))sen^2(x) - 1/4. O
 número de raízes dessa função, no intervalo ]0, pi [ é?

 Vanessa Nunes

 



Re: [obm-l] Ajuda

2012-04-08 Por tôpico Hugo Fernando Marques Fernandes
Oi, Vanessa.

A questão é dois é aplicação direta da Lei dos Senos.

4/(sen80°) = x/(sen40°) =  4/(2*sen40°cos40°) = x/(sen40°) = x = 2/cos40°

Att.

Hugo.

Em 7 de abril de 2012 20:35, Vanessa Nunes de Souza 
vanessani...@hotmail.com escreveu:


 Caros colegas, se puderem me ajudar nessas questões, agradeço.

 1- Considere verdadeira as 3 seguintes afirmações:

 I- Todos os amigos de João são amigos de Mario.
 II- Mario não é amigo de qualquer amigo de Paulo.
 III-Mario é amigo de Roberto.

 Se Roberto é amigo de Paulo, então:
 a) Antônio é amigo de Mário.
 b) João é amigo de Roberto.
 c) Mario é amigo de Roberto.
 d) Antônio é amigo de Jõao.

 2- No triângulo ABC tem-se que BÂC mede 80 graus, ABC mede 40 graus e BC=4
 cm. Se sen 20=K, então a medida de AC é?

 3- Considere a função f(x)= sen^3(x) cos(x)- (cos^3(x))sen^2(x) - 1/4. O
 número de raízes dessa função, no intervalo ]0, pi [ é?

 Vanessa Nunes

 



Re: [obm-l] O Jogo do Tiro ao Alvo

2012-03-30 Por tôpico Fernando Candeias
Manoel
Bom encontrar você por aqui, em meio a um grupo tão competente e dedicado.
Desenhe um retângulo ABCD, AB=x, BC=y e desloque a linha BC de uma unidade
para a esquerda, o que subtrairá da área original um retângulo de y
unidades. Marque com M e N os pontos de interseção  com AB e DC.
Faça uma operação semelhante com a reta DC=x, que interceptará AD, MN e  e
BC nos pontos Q,R e P.   Mas  desloque para cima somente o trecho DN=x-1.
O processo particiona a área original xy em quatro  outras, A1=N,A2=y, um
quadrado de área 1, junto ao vértice R  e A3 de área x-1.
Note que o processo deixa inalterada a área de qualquer retângulo, por
exemplo, que envolva ABCD=xy
A área reduzida N=A1 é a resposta do problema e vale N=(x-1).(y-1).
Simples assim.
Abraços
Fernando A Candeias


Em 15 de março de 2012 10:33, Manoel R D'Oliveira Neto dol...@mac.comescreveu:

 Gostaria de colocar a seguinte questão.

 Seja um jogo de tiro ao alvo, com a parte central do alvo valendo y pontos
 e a parte externa valendo x pontos, onde x e y são primos entre si e xy.
 Tiro fora do alvo vale zero pontos. Antes de o jogo começar, é escolhida
 uma determinada pontuação que os jogadores deverão atingir após vários
 tiros. Ganha quem atingir exatamente esta pontuação pré-definida,
 independente do número de tiros que der. Seja N a menor pontuação que se
 pode pré-definir, a partir da qual todos os números seguintes podem ser
 escolhidos como pontuação pré-definida. Por exemplo, para x=3 e y=5, note
 que não podemos escolher como pontuação pré-definida os seguintes números:
 1, 2, 4 e 7. Porém, 3, 5, 6 e a partir de 8 inclusive, todos podem ser
 escolhidos. Assim, neste caso, N=8.

 Provar que N=(x-1).(y-1)

 Abs,
 Manoel DOliveira



[obm-l] Re: [obm-l] Fwd: [obm-l] Quantidade mínnima de tentativas

2012-01-17 Por tôpico Hugo Fernando Marques Fernandes
Valeu, Ralph... Agora sim, são 22... será que ainda dá pra baixar mais?
Depois vou tentar mais um pouco.
Abraços e obrigado pela correção.

Hugo.

Em 16 de janeiro de 2012 22:31, Ralph Teixeira ralp...@gmail.com escreveu:

 -- Forwarded message --
 From: Ralph Teixeira ralp...@gmail.com
 Date: 2012/1/16
 Subject: RE: [obm-l] Quantidade mínnima de tentativas
 To: obm-l@mat.puc-rio.br


 Hugo, nao desanime! Com um pequeno ajuste, sua solucao ainda dah 22 testes!

 (Eu tinha mandado isso para a lista, mas acho que foi barrado por
 causa de um anexo)

 Chutei o balde: coloquei as 70 opções para as 4 pilhas boas numa
 planilha Excel, em ordem lexicográfica, para ver bem o que está
 acontecendo. A cada passo, cobri as opções com os testes do Hugo
 usando cores bonitinhas (mando a planilha por E-mail para quem quiser,
 ajuda pacas a ver o que estamos fazendo).
 Então percebi algumas coisas na solução do Hugo... Resumindo
 cripticamente:

 1. ABC e FGH (2 testes, eliminando 10 opções)
 2a. (D ou E) com todos os pares em ABC (6t, -21op)
 2b. (D ou E) com todos ou FGH (6t, -21op)
 3a. ABE, BDE, CDE (2t, -9op)
 3b. ABF, ABG (2t, -3op)
 4. CFG, CFH, +CGH (-3t, -6op)
 Total: 22 testes, 70 opções.

 Note algo interessante: retirei ABH do passo 3b! Afinal, se ABH fosse
 bom, as pilhas boas seriam ABCH, ABDH, ABEH, ABFH ou ABGH. Mas em cada
 um desses casos, já teríamos uma combinação boa (respectivamente, em
 1, 2a, 2a, 3b, 3b). Então ABH é desnecessário!

 Por outro lado, adicionei CGH no passo 4. O motivo é que a solução do
 Hugo não cobria os casos ACGH e BCGH, pelo menos não que eu tenha
 visto.

 Ou seja, deu 22 testes! Alguém dá menos?

 Abraço,
   Ralph

 2012/1/16 Hugo Fernando Marques Fernandes hfernande...@gmail.com:
  Fiz assim:
 
  Considere três grupos: abc, de, fgh
 
  Testo o primeiro grupo (abc): se falhar este grupo tem 1 ou 2 pilhas
 boas.
  Testo o terceiro grupo (fgh): se falhar este grupo tem 1 ou 2 pilhas
 boas.
 
  Testo cada elemento do segundo grupo contra os pares formados pelos
  elementos dos outros grupos. São 12 testes, a saber:
  abd, acd, bcd, abe, ace, bce
  e tb fgd, fhd, ghd, fge, fhe, ghe
 
  Note que o segundo grupo (de) pode ter 0, 1 ou 2 pilhas boas.
  1) Se tiver 0 então existe duas boas no grupo (abc) e duas boas em (fgh)
  2) Se tiver 1 boa, então um dos grupos (abc) ou (fgh) tem duas boas (e o
  outro uma). Nesse caso, um dos doze testes acima teria funcionado. Logo,
 se
  não funcionou, podemos excluir essa hipótese.
  3) Se tiver duas boas, então cada um dos grupos (abc) e (fgh) tem só 1
 boa
  também.
 
  Se pensarmos primeiro no caso 3, podemos testar (ade), (bde), (cde) e uma
  vai funcionar.
 
  Se não funcionar, resta o caso 1, e os testes (abf), (abg) e (abh) devem
  funcionar - se não funcionar, então com certeza c funciona junto com fg
 ou
  fh, ou seja, temos mais dois testes, (cfg) e (cfh)
 
  Então no pior caso temos, 1+1+12+3+3+2 = 22
 
  Estou certo ou há alguma falha no raciocínio?
 
  Abs a todos.
 
  Hugo.
 
 
  Em 13 de janeiro de 2012 23:00, Breno Vieira brenovieir...@hotmail.com
  escreveu:
 
  Como eu ja disse, achei 23:
 
  1. Teste ABC, se nao funcionar sabemos que pelo menos uma entre A, B e C
  nao funciona.
  2. Teste as combinacoes entre DEFGH
  (DEF,DEG,DEH,DFG,DFH,DGH,EFG,EFH,EGH,FGH), se nenhuma funcionar temos
 que
  tres entre DEFGH nao funcionam, portando duas entre ABC e duas entre
 DEFGH
  funcionam.
  3. Sabemos que AB, AC ou BC sao formadas por duas que funcionam e que
 pelo
  menos uma entre D,E,F,G funciona, bastam entao mais 12 testes
 totalizando
  23.
 
  PS:Ainda tem mais outros dois algoritmos um pouco mais complicados que
 eu
  fiz e que tambem chegam em 23. Quem da menos?
 
 

 =
 Instruções para entrar na lista, sair da lista e usar a lista em
 http://www.mat.puc-rio.br/~obmlistas/obm-l.html
 =



[obm-l] Re: [obm-l] RE: [obm-l] RE: [obm-l] Re: [obm-l] Re: [obm-l] RE: [obm-l] Quantidade mínnima de tentativas

2012-01-16 Por tôpico Hugo Fernando Marques Fernandes
Fiz assim:

Considere três grupos: abc, de, fgh

Testo o primeiro grupo (abc): se falhar este grupo tem 1 ou 2 pilhas boas.
Testo o terceiro grupo (fgh): se falhar este grupo tem 1 ou 2 pilhas boas.

Testo cada elemento do segundo grupo contra os pares formados pelos
elementos dos outros grupos. São 12 testes, a saber:
abd, acd, bcd, abe, ace, bce
e tb fgd, fhd, ghd, fge, fhe, ghe

Note que o segundo grupo (de) pode ter 0, 1 ou 2 pilhas boas.
1) Se tiver 0 então existe duas boas no grupo (abc) e duas boas em (fgh)
2) Se tiver 1 boa, então um dos grupos (abc) ou (fgh) tem duas boas (e o
outro uma). Nesse caso, um dos doze testes acima teria funcionado. Logo, se
não funcionou, podemos excluir essa hipótese.
3) Se tiver duas boas, então cada um dos grupos (abc) e (fgh) tem só 1 boa
também.

Se pensarmos primeiro no caso 3, podemos testar (ade), (bde), (cde) e uma
vai funcionar.

Se não funcionar, resta o caso 1, e os testes (abf), (abg) e (abh) devem
funcionar - se não funcionar, então com certeza c funciona junto com fg ou
fh, ou seja, temos mais dois testes, (cfg) e (cfh)

Então no pior caso temos, 1+1+12+3+3+2 = 22

Estou certo ou há alguma falha no raciocínio?

Abs a todos.

Hugo.


Em 13 de janeiro de 2012 23:00, Breno Vieira
brenovieir...@hotmail.comescreveu:

  Como eu ja disse, achei 23:

 1. Teste ABC, se nao funcionar sabemos que pelo menos uma entre A, B e C
 nao funciona.
 2. Teste as combinacoes entre DEFGH
 (DEF,DEG,DEH,DFG,DFH,DGH,EFG,EFH,EGH,FGH), se nenhuma funcionar temos que
 tres entre DEFGH nao funcionam, portando duas entre ABC e duas entre DEFGH
 funcionam.
 3. Sabemos que AB, AC ou BC sao formadas por duas que funcionam e que pelo
 menos uma entre D,E,F,G funciona, bastam entao mais 12 testes totalizando
 23.

 PS:Ainda tem mais outros dois algoritmos um pouco mais complicados que eu
 fiz e que tambem chegam em 23. Quem da menos?



[obm-l] Re: [obm-l] Re: [obm-l] Re: [obm-l] RE: [obm-l] RE: [obm-l] Re: [obm-l] Re: [obm-l] RE: [obm-l] Quantidade mínnima de tentativas

2012-01-16 Por tôpico Hugo Fernando Marques Fernandes
Tem razão, Pedro. Seriam 23 testes, então.

Em 16 de janeiro de 2012 15:23, Pedro Nascimento pedromn...@gmail.comescreveu:

 Se no ultimo caso,no conunto fgh as que funcionam forem gh , nao
 precisaria testar cgh tbm?

 Em 16 de janeiro de 2012 10:36, Hugo Fernando Marques Fernandes 
 hfernande...@gmail.com escreveu:

 Fiz assim:

 Considere três grupos: abc, de, fgh

 Testo o primeiro grupo (abc): se falhar este grupo tem 1 ou 2 pilhas boas.
 Testo o terceiro grupo (fgh): se falhar este grupo tem 1 ou 2 pilhas boas.

 Testo cada elemento do segundo grupo contra os pares formados pelos
 elementos dos outros grupos. São 12 testes, a saber:
 abd, acd, bcd, abe, ace, bce
 e tb fgd, fhd, ghd, fge, fhe, ghe

 Note que o segundo grupo (de) pode ter 0, 1 ou 2 pilhas boas.
 1) Se tiver 0 então existe duas boas no grupo (abc) e duas boas em (fgh)
 2) Se tiver 1 boa, então um dos grupos (abc) ou (fgh) tem duas boas (e o
 outro uma). Nesse caso, um dos doze testes acima teria funcionado. Logo, se
 não funcionou, podemos excluir essa hipótese.
 3) Se tiver duas boas, então cada um dos grupos (abc) e (fgh) tem só 1
 boa também.

 Se pensarmos primeiro no caso 3, podemos testar (ade), (bde), (cde) e uma
 vai funcionar.

 Se não funcionar, resta o caso 1, e os testes (abf), (abg) e (abh) devem
 funcionar - se não funcionar, então com certeza c funciona junto com fg ou
 fh, ou seja, temos mais dois testes, (cfg) e (cfh)

 Então no pior caso temos, 1+1+12+3+3+2 = 22

 Estou certo ou há alguma falha no raciocínio?

 Abs a todos.

 Hugo.


 Em 13 de janeiro de 2012 23:00, Breno Vieira 
 brenovieir...@hotmail.comescreveu:

  Como eu ja disse, achei 23:

 1. Teste ABC, se nao funcionar sabemos que pelo menos uma entre A, B e C
 nao funciona.
 2. Teste as combinacoes entre DEFGH
 (DEF,DEG,DEH,DFG,DFH,DGH,EFG,EFH,EGH,FGH), se nenhuma funcionar temos que
 tres entre DEFGH nao funcionam, portando duas entre ABC e duas entre DEFGH
 funcionam.
 3. Sabemos que AB, AC ou BC sao formadas por duas que funcionam e que
 pelo menos uma entre D,E,F,G funciona, bastam entao mais 12 testes
 totalizando 23.

 PS:Ainda tem mais outros dois algoritmos um pouco mais complicados que
 eu fiz e que tambem chegam em 23. Quem da menos?






[obm-l] Re: [obm-l] Dúvida

2011-10-24 Por tôpico Hugo Fernando Marques Fernandes
Lucas corre 2/5 da ponte com velocidade de 15 km/h. Sendo p o comprimento da
ponte, leva (2/5)p/15 = 2p/75 h para sair da ponte.
Pedro corre 3/5 da ponte com velocidade de 15 km/h. Sendo p o comprimento da
ponte, leva (3/5)p/15 = 3p/75 h para sair da ponte.

A diferença entre o momento em que Lucas sai da ponte (momento em que o trem
entra na ponte) e o momento em que Pedro sai dela (momento em que o trem sai
da ponte) é 3p/75 - 2p/75 = p/75. Este é o tempo que o trem leva para
percorrer a ponte, ou seja, para percorrer p km.

Portanto, a velocidade do trem será dada por p/(p/75) = 75 km/h, alternativa
C

Abraços.

Hugo.

Em 24 de outubro de 2011 15:29, Jorge Paulino da Silva Filho 
jorge...@yahoo.com.br escreveu:

 **
 Dois amigos, Lucas e Pedro, seguiam o leito de uma ferrovia e começaram a
 atravessar uma ponte estreita na qual havia espaço apenas para o trem. No
 momento em que completavam 2/5 do percurso da ponte, ouviram o trem que
 se aproxima por trás deles. Lucas começou a correr de encontro ao trem,
 saindo da ponte praticamente no instante em que o trem entrava. Pedro,
 que correu no sentindo oposto ao sentido de Lucas, conseguir sair da ponte
 praticamenteno instante em que o trem saía. Sendo 15km/h a velocidade que
 Lucas
 e Pedro correram, assinale a alternativa que contém a velocidade do trem:

 a)60km/h
 b)37km/h
 c)75km/h
 d)30km/h
 e)67,5km/h



Re: [obm-l] Desafio 7 e 10

2011-09-28 Por tôpico Hugo Fernando Marques Fernandes
O problema está em definir o que sejam exatamente símbolos e operações
aritméticas elementares...

Hugo

Em 28 de setembro de 2011 17:24, luiz silva
luizfelipec...@yahoo.com.brescreveu:

 Elevar todos os dois casos a 0 e somar vale ?

 Abs
 Felipe

 --- Em *qua, 28/9/11, geonir paulo schnorr geonirpa...@gmail.com*escreveu:


 De: geonir paulo schnorr geonirpa...@gmail.com
 Assunto: Re: [obm-l] Desafio 7 e 10
 Para: obm-l@mat.puc-rio.br
 Data: Quarta-feira, 28 de Setembro de 2011, 15:53


 log10 + log10 + log10 + log10 = 1 + 1 + 1 + 1 = 4

 Em 28 de setembro de 2011 14:34, Rhilbert Rivera 
 rhilbert1...@hotmail.comhttp://br.mc657.mail.yahoo.com/mc/compose?to=rhilbert1...@hotmail.com
  escreveu:

  Como tornar as igualdades verdadeiras usando símbolos e operações
 aritméticas elementares


 7  7  7  7  = 4

 10   10   10   10  = 4


 Obrigado






Re: [obm-l] Desafio 7 e 10

2011-09-28 Por tôpico Hugo Fernando Marques Fernandes
1+0+1+0+1+0+1+0=4

Em 28 de setembro de 2011 18:29, Rhilbert Rivera
rhilbert1...@hotmail.comescreveu:

   Uma outra solução que consegui foi

 7! : #7 + (7-7) = 4!

 7! = 7x6x5x4x3x2 = 5040 (fatorial de 7)

 #7 = 7x5x3x2 = 210 (primorial de 7)



[obm-l] Re: [obm-l] Questão de probabilidade(dúvida sobre gabarito)

2011-09-25 Por tôpico Hugo Fernando Marques Fernandes
A minha deu item a, 2,86%.

Veja só, existem duas maneiras de sentarem alternados:

HMHMHMHM
e
MHMHMHMH

Em cada um dessas maneiras, permuto os homens (4!) e as mulheres (4!)

Resultado, são 2 x 4! x 4! casos favoráveis.

Os casos possíveis são 8!

Logo, a probabilidade é (2 x 4! x 4!)/8! =~ 2,86%

Abraços.

Hugo

Em 25 de setembro de 2011 02:14, marcone augusto araújo borges 
marconeborge...@hotmail.com escreveu:

  Um grupo de pessoas,composto por 4 homens e 4 mulheres,compra 8 cadeiras
 consecutivas na mesma fila de um teatro.Se eles se sentarem
 aleatóriamente,nessas cadeiras,a probabilidade de que homens e mulheres se
 sentem em cadeiras alternadas é aproximadamente:

 a) 2,86%  b) 5,71%   c) 1,43%d) 11,42%

  O primeiro homem pode sentar em 8 lugares.O segundo pode sentar em 6
 lugares.O terceiro,em 4 lugares e o quarto,em 2 lugares.Como sobram 4
 lugares,é só permutar as 4 mulheres.Então o número de possibilidades  de que
 homens e mulheres sentem em cadeiras alternadas é 8x6x4x2x4x3x2x1.Dividindo
 esse número por 8!(que é o total de possibilidades) temos 5/35 = 0,2285.
 O que me intriga é que esse resultado é o dobro dos 11,42% do item d
 Alguem poderia esclarecer?



[obm-l] Problema das Quatro Cores (Teoria dos Grafos)

2011-09-17 Por tôpico Hugo Fernando Marques Fernandes
Olá, Lista.

Seguinte, estava lendo sobre o problema das quatro cores, que segundo
entendi é um teorema da teoria dos grafos que afirma que se pode colorir
qualquer grafo planar com quatro cores de modo que nós adjacentes (ou seja,
que possuam aresta ligando-os) não sejam pintados da mesma cor.

Consta que tal fato permaneceu por séculos sem demonstração, e a que existe
hoje depende de recursos computacionais para ser completada, o que levanta
dúvidas sobre a mesma.

Minha pergunta então, é a seguinte:

Para que seja preciso 5 cores para pintar o grafo, eu teria que ter 5 nós
ligados entre si, isto é, eu teria que ter um sub-grafo do meu grafo inicial
que fosse um grafo completo de 5 nós (K5). Ora, sabemos (é fácil demonstrar,
já vi em vários livros a demonstração) que K5 não tem realizações
planares... logo, o teorema segue.

Sei que isso está errado (afinal de contas, se não estivesse, alguém teria
visto de cara e o problema não teria ficado séculos em aberto...) mas não
consigo ver onde está o erro desse raciocínio. Alguém pode me ajudar?

Obrigado.


Re: [obm-l] Problema das Quatro Cores (Teoria dos Grafos)

2011-09-17 Por tôpico Hugo Fernando Marques Fernandes
Entendi... de certa forma estou usando o que quero provar pra fazer a prova,
né?
Valeu!

Hugo.

Em 17 de setembro de 2011 23:51, Johann Dirichlet
peterdirich...@gmail.comescreveu:

 Existe uma demonstração fácil de que 5 cores bastam para pintar um grafo
 planar.
 Acho que este é seu problema: tentar provar por absurdo algo que se
 provaria diretamente.
 Certamente, se você usa 4 cores para piontar, alguém que tem um
 estoque de 5,6,7,2002 cores também consegue.

 Mas o salto lógico é este:
 Para que seja preciso 5 cores para pintar o grafo, eu teria que ter 5
 nós ligados entre si.

 Se isto não for corretamente demonstrado, adeus demonstração!



 Em 17/09/11, Hugo Fernando Marques Fernandeshfernande...@gmail.com
 escreveu:
  Olá, Lista.
 
  Seguinte, estava lendo sobre o problema das quatro cores, que segundo
  entendi é um teorema da teoria dos grafos que afirma que se pode colorir
  qualquer grafo planar com quatro cores de modo que nós adjacentes (ou
 seja,
  que possuam aresta ligando-os) não sejam pintados da mesma cor.
 
  Consta que tal fato permaneceu por séculos sem demonstração, e a que
 existe
  hoje depende de recursos computacionais para ser completada, o que
 levanta
  dúvidas sobre a mesma.
 
  Minha pergunta então, é a seguinte:
 
  Para que seja preciso 5 cores para pintar o grafo, eu teria que ter 5 nós
  ligados entre si, isto é, eu teria que ter um sub-grafo do meu grafo
 inicial
  que fosse um grafo completo de 5 nós (K5). Ora, sabemos (é fácil
 demonstrar,
  já vi em vários livros a demonstração) que K5 não tem realizações
  planares... logo, o teorema segue.
 
  Sei que isso está errado (afinal de contas, se não estivesse, alguém
 teria
  visto de cara e o problema não teria ficado séculos em aberto...) mas não
  consigo ver onde está o erro desse raciocínio. Alguém pode me ajudar?
 
  Obrigado.
 


 --
 /**/
 神が祝福

 Torres

 =
 Instru�ões para entrar na lista, sair da lista e usar a lista em
 http://www.mat.puc-rio.br/~obmlistas/obm-l.html
 =



[obm-l] Re: [obm-l] Análise Combinatória - mais um

2011-09-13 Por tôpico Hugo Fernando Marques Fernandes
Seja a equação linear com coeficientes unitários x1 + x2 +...+ xw  = u

Escrevemos: 1 + 1 + 1 + ... + 1 = u (u parcelas iguais a 1).

Cada solução inteira e positiva dessa equação corresponde a escolha de w-1
sinais mais dentre o u-1 existentes na igualdade acima.

Por exemplo, a solução x1=x2=x3=...=x(w-1)=1 e xw=u-w+1 pode ser vista como:

1 *+* 1 *+* 1 *+*  ... *+* 1 +1 +1... + 1 = u (onde escolhemos os primeiros
w-1 sinais de mais)

Ora, podemos fazer isso de C(u-1,w-1) maneiras distintas.

Logo, existem C(u-1,w-1) soluções inteiras e positivas da equação.

Para soluções inteiras não negativas, fazemos, para cada i variando de 1 a w

yi = xi-1

Agora, a equação fica: y1 - 1 + y2 - 1 +...+ yw - 1  = u
Daí, y1 + y2 + ... + yw = u+w

Note que cada solução inteira positiva da equação acima corresponde uma
solução não negativa da equação original.

Mas já sabemos que a equação acima possui C(u+w-1, w-1) soluções inteiras
positivas.

Assim, a equação original possui C(u+w-1, w-1) soluções inteiras não
negativas.

Não sei se chega a ser uma demonstração o que escrevi, mas é uma boa maneira
de ver essas fórmulas.

Abraços.

Hugo.

Em 12 de setembro de 2011 17:11, João Maldonado joao_maldona...@hotmail.com
 escreveu:


 Olá,

 Queria saber como provar a que  a  quantidade de soluções inteiras
 positivas  de um sistema  com w variáveis da forma
 x1 + x2 +...+ xw  = u
 é  C(u-1, w-1)

 E que a quantidade  de soluções inteiras  não negativas é

 C(w+u-1, w-1)


 []'s
 João



[obm-l] Re: [obm-l] RE: [obm-l] Re: [obm-l] Análise Combinatória - mais um

2011-09-13 Por tôpico Hugo Fernando Marques Fernandes
Isso mesmo.
Nesse caso, você aplicaria mudança de variáveis: yi = xi-2

Em geral, para soluções inteiras maiores ou iguais a p, você deve aplicar a
mudança de variável yi=xi+p-1

Abraços.

Hugo

Em 13 de setembro de 2011 19:55, João Maldonado joao_maldona...@hotmail.com
 escreveu:




 Valeu Hugo,

 Mas só pra ver se eu entendi,  se fossem as  soluções inteiras = -1,
  seria C(u+ 2w-1, w-1)?

 []'s
 João

 --
 Date: Tue, 13 Sep 2011 15:55:09 -0300
 Subject: [obm-l] Re: [obm-l] Análise Combinatória - mais um
 From: hfernande...@gmail.com
 To: obm-l@mat.puc-rio.br


 Seja a equação linear com coeficientes unitários x1 + x2 +...+ xw  = u

 Escrevemos: 1 + 1 + 1 + ... + 1 = u (u parcelas iguais a 1).

 Cada solução inteira e positiva dessa equação corresponde a escolha de w-1
 sinais mais dentre o u-1 existentes na igualdade acima.

 Por exemplo, a solução x1=x2=x3=...=x(w-1)=1 e xw=u-w+1 pode ser vista
 como:

 1 *+* 1 *+* 1 *+*  ... *+* 1 +1 +1... + 1 = u (onde escolhemos os
 primeiros w-1 sinais de mais)

 Ora, podemos fazer isso de C(u-1,w-1) maneiras distintas.

 Logo, existem C(u-1,w-1) soluções inteiras e positivas da equação.

 Para soluções inteiras não negativas, fazemos, para cada i variando de 1 a
 w

 yi = xi-1

 Agora, a equação fica: y1 - 1 + y2 - 1 +...+ yw - 1  = u
 Daí, y1 + y2 + ... + yw = u+w

 Note que cada solução inteira positiva da equação acima corresponde uma
 solução não negativa da equação original.

 Mas já sabemos que a equação acima possui C(u+w-1, w-1) soluções inteiras
 positivas.

 Assim, a equação original possui C(u+w-1, w-1) soluções inteiras não
 negativas.

 Não sei se chega a ser uma demonstração o que escrevi, mas é uma boa
 maneira de ver essas fórmulas.

 Abraços.

 Hugo.

 Em 12 de setembro de 2011 17:11, João Maldonado 
 joao_maldona...@hotmail.com escreveu:


 Olá,

 Queria saber como provar a que  a  quantidade de soluções inteiras
 positivas  de um sistema  com w variáveis da forma
 x1 + x2 +...+ xw  = u
 é  C(u-1, w-1)

 E que a quantidade  de soluções inteiras  não negativas é

 C(w+u-1, w-1)


 []'s
 João





[obm-l] Re: [obm-l] Re: [obm-l] FW: Combinatória

2011-07-22 Por tôpico Fernando A Candeias
Oi Ralph
substantivo femininoTemos em parte uma problema de semântica. No Houaiss
encontramos o verbete:  O que mostra que pode ser também utilizada para
indicar probabilidade, como no exemplo que dá em verde. No
primeiro significado ela indica simplesmente a existencia;. Como alguem que
perguntasse,. é possivel a ordem de chegada ABC ? E a respopsta seria sim ,
par 1q





condição do que é possível, do que pode acontecer

Ex.: muita, pouca p.

 O que mostra
Em 20 de julho de 2011 12:19, Ralph Teixeira ralp...@gmail.com escreveu:

 Eu gosto MUITO da solucao do Fernando, tambem acho que eh a mais elegante.

 Isto dito, vou ser chato muito muito chato: nada no problema sugere nem
 pede probabilidades. Entao vamos usar a otima ideia, mas mudar a linguagem:

 Para cada possibilidade que tenha a ordem ABC, teremos possibilidades
 correspondentes com as ordens ACB, CAB, etc. Ou seja, o numero de
 classificacoes possiveis eh 6 vezes o numero de classificacoes pedido.
 Assim, existem 6!/6=120 classificacoes possiveis onde Felipe se torna
 campeao.

 E notem que isto NAO significa que a probabilidade de Felipe ser
 campeao eh 1/6, jah que ninguem sabe se as possibilidades sao
 realmente igualmente provaveis.

 (Alias, considerando o momento da Red Bull e do Felipe... :) :) :) :) )

 Abraco,
   Ralph

 2011/7/20 João Maldonado joao_maldona...@hotmail.com

  Olá.

 Eu não vejo erro, aliás é até mais prática do que as soluções já
 apresentradas.

 Só esclarecendo  para os outros, demorei um pouco para entender:


 ABC  significa que A está a frente de B que está a frente de C  (não
 importa de que jeito)
 Ex:
 ADBECF ou ABDEFC, DEFABC, etc

 --
 Date: Tue, 19 Jul 2011 11:30:17 -0300
 Subject: Combinatória
 From: facande...@gmail.com
 To: joao_maldona...@hotmail.com

 Prezador colega.

 Outra maneira seria a seguinte, usando sua convenção para o nome dos
 concorrentes.
 Os possíveis resultados seriam uma das seis permutações,   ABC, ACB, BAC,
 BCA, CAB, CBA. todas equiprováveis. Como a ordem  favorável é  ABC,
 e probabilidade de sua ocorrência será 1/6, que será também a de Felipe ser
 campeão,  como você achou
 Está certa essa linha de raciocínio?
 Sds.
 Fernando Candeias







[obm-l] Re: [obm-l] Brasil conquista medalhas de Prata e Bronze na Olimpíada Internacional de Matemática (IMO)

2011-07-22 Por tôpico Fernando A Candeias
É um juso motivo de orgulho para esta sofrida nação. E sem nenhum apoio do
papai governo. Temos ótimos matemáticos, o que certamente eleva nossa auto
estima, e  que  deveria ser  adequadamente  valorizado.

Fernando Candeias
Em 22 de julho de 2011 10:11, Olimpiada Brasileira de Matematica 
o...@impa.br escreveu:

 **

 *Brasil conquista medalhas de Prata e Bronze*

 *na Olimpíada Internacional de Matemática (IMO)*


 O Brasil obteve um excelente resultado este ano na 52a. Olimpíada
 Internacional de Matemática (IMO), que acontece até o dia 24 de julho na
 cidade de Amsterdã na Holanda,* *conquistando três medalhas de prata e
 três de bronze. Os estudantes: André Macieira Braga (Belo Horizonte – MG),
 João Lucas Camelo Sá (Fortaleza – CE) e Henrique Fiúza do Nascimento
 (Brasília – DF), conquistaram as medalhas de prata, enquanto Débora Barbosa
 Alves (São Paulo – SP), Maria Clara Mendes Silva (Pirajuba – MG) e Gustavo
 Lisbôa Empinotti (Florianópolis – SC) conquistaram medalhas de bronze. Com
 este resultado o Brasil classificou em vigésimo lugar entre os países
 participantes.

 Considerada pela Unesco como a competição mais importante da área, a IMO
 contou este ano com a participação de 101 países reunindo 564 estudantes,
 entre 14 e 19 anos, mais talentosos do mundo no assunto. O Brasil foi
 representado por uma equipe de seis estudantes liderados pelos professores 
 Nicolau
 Corção Saldanha (Rio de Janeiro – RJ) e Eduardo Tengan (São Carlos – SP).


 Um comitê internacional elegeu os problemas que seriam resolvidos entre os
 propostos pelos países participantes. As provas foram realizadas em dois
 dias consecutivos abrangendo disciplinas como Álgebra, Teoria dos números,
 Geometria e Combinatória. Em cada dia, os participantes resolveram três
 problemas, com valor de sete pontos cada, aplicados em 4 horas e meia de
 prova. A resolução destes problemas requer mais criatividade, engenho e
 habilidade em matemática do que conhecimentos e fórmulas aplicadas.

 *Brasil e as medalhas na IMO*

 A Olimpíada Internacional de Matemática (IMO) é realizada desde 1959. O
 Brasil participa da competição desde 1979 conquistando desde então um total
 de 96 medalhas, sendo oito de ouro, 26 de prata e 62 de bronze.

 A participação brasileira na competição é organizada através da Olimpíada
 Brasileira de Matemática (OBM), iniciativa que tem desempenhado um
 importante papel em relação à melhoria do ensino e descoberta de talentos
 para a pesquisa em Matemática nas modalidades de ensino fundamental e médio
 nas escolas públicas e privadas de todo o Brasil.  A Olimpíada Brasileira
 de Matemática é um projeto conjunto do Instituto Nacional de Matemática Pura
 e Aplicada (IMPA) da Sociedade Brasileira de Matemática (SBM) e conta com o
 apoio do Conselho Nacional de Desenvolvimento Científico e Tecnológico
 (CNPq) e do Instituto Nacional de Ciência e Tecnologia de Matemática (INCT–
 Mat).



 *Informações:*

 Nelly Carvajal – Assessoria de Imprensa

 Tel: 21-25295077 – Fax: 21-25295023

 e-mail:o...@impa.br

 --
 Secretaria da Olimpíada Brasileira de Matemática
 Estrada Dona Castorina, 110 Jd. Botânico,
 Rio de Janeiro - RJ, 22460-320, Brasil
 Tel: 55-21-25295077 Fax:55-21-25295023
 e-mail: o...@impa.br
 web site: www.obm.org.br




Re: [obm-l] Re: [obm-l] Urna Probabilidade

2011-05-09 Por tôpico Fernando A Candeias
Uma outra solução.

O espaço amostral correspondente ao problema tem 15 eventos: PR (6:2, 4). Os
casos favoráveis são aqueles em que uma das duas bolas brancas ocupa uma das
3 casas ímpares ou as duas bolas brancas ocupam duas dessas casa: A(3,1) +
A(3,2)=3+6=9. Ao todo nove eventos favoráveis. Probabilidade de sucesso
p=9/15=3/5.

Abs.

Fernando Candeias

2011/3/31 Julio César Saldaña saldana...@pucp.edu.pe



 Não sei se é a solução mais elegante, mas..

 O evento desejado pode ser representado como a união dos seguintes eventos
 disjuntos:

 A = A primeira bola foi branca
 B = As duas primeiras foram pretas e a terceira foi branca
 C = As quatro primeiras foram pretas e a quinta foi branca.

 Então a probabilidade pedida é P(A)+P(B)+P(C)

 P(A) = 2/6 = 1/3
 P(B) = (4/6)(3/5)(2/4)=1/5
 P(C) = (4/6)(3/5)(2/4)(1/3)(1) = 1/15


 Somando (1/3) + (1/5) + (1/15) = 3/5

 Me avisem se fiz errado

 Obrigado

 Julio Saldaña


 -- Mensaje original ---
 De : obm-l@mat.puc-rio.br
 Para : obm-l@mat.puc-rio.br
 Fecha : Wed, 30 Mar 2011 21:10:53 +0300
 Asunto : [obm-l] Urna Probabilidade
 
 Prezados.
 Em uma urna, são colocadas 2 bolas brancas e 4 pretas.Alberto e Beatriz
 retiram
 bolas da urna alternadamente, iniciando-se com Alberto, até que a urna
 esteja
 vazia. A probabilidade de que a primeira bola branca saia para Alberto é
 (A) 1/2
 (B) 3/5
 (C) 5/9
 (D) 7/12
 (E) 8/15
 Grato.
 Marcos.

 __
 Si desea recibir, semanalmente, el Boletín Electrónico de la PUCP, ingrese
 a:
 http://www.pucp.edu.pe/puntoedu/suscribete/

 =
 Instruções para entrar na lista, sair da lista e usar a lista em
 http://www.mat.puc-rio.br/~obmlistas/obm-l.html
 =



[obm-l] Re: [obm-l] FW: [obm-l] Re: [obm-l] RE: [obm-l] Olimpíadas cearenses(geometria)

2011-04-21 Por tôpico Hugo Fernando Marques Fernandes
Tem razão, Marcone e João... relendo agora entendi melhor a questão.
Abs.

Hugo.

Em 21 de abril de 2011 14:28, marcone augusto araújo borges 
marconeborge...@hotmail.com escreveu:



 --
 From: marconeborge...@hotmail.com
 To: obm-l@mat.puc-rio.br
 Subject: RE: [obm-l] Re: [obm-l] RE: [obm-l] Olimpíadas
 cearenses(geometria)
 Date: Thu, 21 Apr 2011 16:05:12 +


  O João está certo.E minha pergunta no final foi se basta provar que DOIS
 lados opostos são paralelos e congruentes.Um abraço.
 --
 Date: Wed, 20 Apr 2011 23:10:44 -0300
 Subject: [obm-l] Re: [obm-l] RE: [obm-l] Olimpíadas cearenses(geometria)
 From: hfernande...@gmail.com

 To: obm-l@mat.puc-rio.br

 Não entendi bem sua solução, João.

 Pelo que diz o enunciado, os vértices do trapézio são os pontos médios de
 um quadrilátero convexo.
 Da maneira como você fez, parece que você considerou o quadrilátero formado
 pelos pontos médios dos lados do trapézio.
 É isso mesmo, ou estou enganado?

 Abs.

 Hugo.

 Em 20 de abril de 2011 20:38, João Maldonado 
 joao_maldona...@hotmail.comescreveu:

 *Na verdade basta provar que os lados opostos são iguais, automaticamente
 serão paralelos.*
 *Todo uq**adrilátero com ladosopostos iguais é um paralelogramo.*
 *
 *
 *Prova:*
 *Faça dois lados (a e b) de um quadrilátero qualquer saindo de um vértice
 V. Para que os lados opostos sejm iguais podemos traçar uma circunferência a
 partir do fim dos lados a e b, com raio igual ao lado oposto. Desse modo
 teríamos 2 circunferências, que se intersectam em 2 pontos. Um dos pontos
 gera uma configuração de quadrilátero não convexo, a  outra gera um
 quadrilátero convexo. Logo os lados são paralelos.*
 *
 *
 *Mas voltando ao problema,*
 *
 *
 *
 *
 *Fazendo o trapézio ABCD com lados paralelos AB e CD. Os pontos médios de
 AB=X, BC=Y, CD=Z, DA=W. A altura do trapézio h que parte de A intersecta CD
 em P e a altura do trapézio que parte de B instersecta CD em Q (neste caso
 fazendo P e Q dentro do segmento CD (fica para você provar quando um está
 fora). Chamando AB/2 de d, PD de a e QC de b, temos que:*
 *1) Em relação a CD, a coordenada y de W é  h/2, e a coordenada x é
 (2d+a+b)/2 - a/2 = d+b/2*
 *2) Em relação a AB (que é paralela a CD), logo em relação  a CD, a
 coordenada y de W é h/2 e a coordenada x é d+b/2, logo os Ângulos formados
 com  AB são iguais e as retas WZ e XY são paralelas e de mesma  medida*
 *Analogamente para YZ e WX.*
 *
 *
 *Logo se trata de um paralelogramo*
 *
 *
 *
 *
 *[]'s*
 *João*
 *
 *
  *
 *
  --
 From: marconeborge...@hotmail.com
 To: obm-l@mat.puc-rio.br
 Subject: [obm-l] Olimpíadas cearenses(geometria)
 Date: Wed, 20 Apr 2011 22:06:10 +


 Prove que um quadrilatero convexo cujos vertices sao os pontos medios dos
 lados de um trapezio qualquer é um paralelogramo
 Bastaria provar que dos lados opostos são paralelos e congruentes?





[obm-l] Re: [obm-l] RE: [obm-l] Olimpíadas cearenses(geometria)

2011-04-20 Por tôpico Hugo Fernando Marques Fernandes
Não entendi bem sua solução, João.

Pelo que diz o enunciado, os vértices do trapézio são os pontos médios de um
quadrilátero convexo.
Da maneira como você fez, parece que você considerou o quadrilátero formado
pelos pontos médios dos lados do trapézio.
É isso mesmo, ou estou enganado?

Abs.

Hugo.

Em 20 de abril de 2011 20:38, João Maldonado
joao_maldona...@hotmail.comescreveu:

  *Na verdade basta provar que os lados opostos são iguais, automaticamente
 serão paralelos.*
 *Todo uq**adrilátero com ladosopostos iguais é um paralelogramo.*
 *
 *
 *Prova:*
 *Faça dois lados (a e b) de um quadrilátero qualquer saindo de um vértice
 V. Para que os lados opostos sejm iguais podemos traçar uma circunferência a
 partir do fim dos lados a e b, com raio igual ao lado oposto. Desse modo
 teríamos 2 circunferências, que se intersectam em 2 pontos. Um dos pontos
 gera uma configuração de quadrilátero não convexo, a  outra gera um
 quadrilátero convexo. Logo os lados são paralelos.*
 *
 *
 *Mas voltando ao problema,*
 *
 *
 *
 *
 *Fazendo o trapézio ABCD com lados paralelos AB e CD. Os pontos médios de
 AB=X, BC=Y, CD=Z, DA=W. A altura do trapézio h que parte de A intersecta CD
 em P e a altura do trapézio que parte de B instersecta CD em Q (neste caso
 fazendo P e Q dentro do segmento CD (fica para você provar quando um está
 fora). Chamando AB/2 de d, PD de a e QC de b, temos que:*
 *1) Em relação a CD, a coordenada y de W é  h/2, e a coordenada x é
 (2d+a+b)/2 - a/2 = d+b/2*
 *2) Em relação a AB (que é paralela a CD), logo em relação  a CD, a
 coordenada y de W é h/2 e a coordenada x é d+b/2, logo os Ângulos formados
 com  AB são iguais e as retas WZ e XY são paralelas e de mesma  medida*
 *Analogamente para YZ e WX.*
 *
 *
 *Logo se trata de um paralelogramo*
 *
 *
 *
 *
 *[]'s*
 *João*
 *
 *
 *
 *
 --
 From: marconeborge...@hotmail.com
 To: obm-l@mat.puc-rio.br
 Subject: [obm-l] Olimpíadas cearenses(geometria)
 Date: Wed, 20 Apr 2011 22:06:10 +


 Prove que um quadrilatero convexo cujos vertices sao os pontos medios dos
 lados de um trapezio qualquer é um paralelogramo
 Bastaria provar que dos lados opostos são paralelos e congruentes?



[obm-l] Re: [obm-l] RE: [obm-l] FW: Progressão aritmética

2011-04-20 Por tôpico Hugo Fernando Marques Fernandes
O problema fala em progressão aritmética, não geométrica, João.

Abs.

Hugo.

Em 20 de abril de 2011 21:02, João Maldonado
joao_maldona...@hotmail.comescreveu:

  *Primeiramente note que o primeiro é positivo e a razão também.

 *
 *Chamando o primeiro termo de a e a razão de k, o termo n vale a.k^(n-1)*
 *
 *
 *logo temos:*
 *
 *
 *1) a³ = a.k^7*
 *2) a² pertence à progreesão*
 *3) a^4 pertence à progressão*
 *
 *
 *De 1) a = k^(7/2)*
 *
 *
 *Temos que a² ou a^4 está entre o primeiro e o oitavo termo. *
 *
 *
 *Fazendo a² = a.k^(n-1) - a = k^(n-1) - k^(7/2) = k^(n-1) - k=1*
 *
 *
 *Fazendo a^4, a mesma coisa*
 *
 *
 *Logo o segundo termo é 1.*
 *
 *
 *Possivelmente  errei em alguma coisa porque nnunca vi progressão
 geometrica com razão 1 .*
 *
 *
 *Mas se o resultado bateu com o seu acho queo problema é o enunciado
 mesmo. Vou rever de novo minha solução, qualquer coisa posto outra vez.*
 *
 *
 *
 Abraço*
 *João*

 --
 From: marconeborge...@hotmail.com
 To: obm-l@mat.puc-rio.br
 Subject: [obm-l] FW: Progressão aritmética
 Date: Wed, 20 Apr 2011 22:21:55 +




 --
 From: marconeborge...@hotmail.com
 To: obm-l@mat.puc-rio.br
 Subject: Progressão aritmética
 Date: Wed, 20 Apr 2011 21:58:18 +

  Numa progressao aritmetica de  numeros inteiros positivos,o oitavo termo é
 igual ao cubo do primeiro.Sabendo que a segunda e a quarta potencias do
 primeiro termo pertencem a progressao,determinar o segundo termo.
 Agradeço antecipadamente a quem puder resolver.



[obm-l] Re: [obm-l] Área do triângulo

2011-03-31 Por tôpico Hugo Fernando Marques Fernandes
Bem...

Pela fórmula de Heron, temos A = p(p-a)(p-b)(p-c), onde a,b,c são os lados
do triângulo e p = (a+b+c)/2 (semi-perímetro).
Além disso, como a,b,c formam um triângulo, então, supondo a o maior lado,
temos: ab+c (I).

Vamos escolher b e c, e ver quais são as possibilidades para a, baseado em
(I) e no fato de que a é inteiro positivo:
se b=c=1, b+c=2, nenhuma possibilidade para a, então não existe o triângulo
se b=1 e c=2, b+c=3, nenhuma possibilidade para a, então não existe o
triângulo
se b=2 e c=2, b+c=4, única opção para a é 3, mas então 2p=7 e p=7/2, donde a
área não é inteira, pela fórmula de Heron.
se b=2 e c=3, b+c=5, as opções para a são 3 e 4
tomando a menor, a=3 e daí 2p=8, p=4 e Área (mínima) = 4x1x1x2=8

Acho que é isso.

Abraços.

Hugo.

Em 31 de março de 2011 13:34, Vitor Alves vitor__r...@hotmail.comescreveu:

  Um triângulo tem que seus lados e sua área são números inteiros
 positivos.Qual é o menor valor para a área?



[obm-l] Re: [obm-l] Re: [obm-l] Re: [obm-l] Área do triângulo

2011-03-31 Por tôpico Hugo Fernando Marques Fernandes
Têm razão... isso que dá confiar na memória...
Desculpem o furo.

Hugo.

Em 31 de março de 2011 14:36, Gabriel Dalalio
gabrieldala...@gmail.comescreveu:

 Infelizmente você já começou errado, a fórmula de Heron é A = sqrt(
 p(p-a)(p-b)(p-c) ), e ai ja era né

 Em 31 de março de 2011 14:21, Hugo Fernando Marques Fernandes
 hfernande...@gmail.com escreveu:
  Bem...
 
  Pela fórmula de Heron, temos A = p(p-a)(p-b)(p-c), onde a,b,c são os
 lados
  do triângulo e p = (a+b+c)/2 (semi-perímetro).
  Além disso, como a,b,c formam um triângulo, então, supondo a o maior
 lado,
  temos: ab+c (I).
 
  Vamos escolher b e c, e ver quais são as possibilidades para a, baseado
 em
  (I) e no fato de que a é inteiro positivo:
  se b=c=1, b+c=2, nenhuma possibilidade para a, então não existe o
 triângulo
  se b=1 e c=2, b+c=3, nenhuma possibilidade para a, então não existe o
  triângulo
  se b=2 e c=2, b+c=4, única opção para a é 3, mas então 2p=7 e p=7/2,
 donde a
  área não é inteira, pela fórmula de Heron.
  se b=2 e c=3, b+c=5, as opções para a são 3 e 4
  tomando a menor, a=3 e daí 2p=8, p=4 e Área (mínima) = 4x1x1x2=8
 
  Acho que é isso.
 
  Abraços.
 
  Hugo.
 
  Em 31 de março de 2011 13:34, Vitor Alves vitor__r...@hotmail.com
  escreveu:
 
  Um triângulo tem que seus lados e sua área são números inteiros
  positivos.Qual é o menor valor para a área?
 

 =
 Instruções para entrar na lista, sair da lista e usar a lista em
 http://www.mat.puc-rio.br/~obmlistas/obm-l.html
 =



[obm-l] Re: [obm-l] Re: [obm-l] Re: [obm-l] Re: [obm-l] FRAÇÕES - conceito

2011-03-28 Por tôpico Hugo Fernando Marques Fernandes
Quanto a 0^0=1... Como vc disse, todas as indeterminações do tipo 0^0 dão
1, *com raras exceções*. O problema é que as exceções são raras mas elas *
existem*, então não se pode afirmar a igualdade.

Além disso, escrever p(x)=SUM [(n=1 a M) a_n x^n] + a_0, por exemplo, não me
parece algo tão complicado.

Quanto aos naturais, concordo que dizer que 0 é natural é uma convenção e
pode até ser útil em determinadas situações... além de não introduzir nenhum
problema, exceto uma mera questão de coerência linguística e antropológica -
seres humanos *não* contam dessa forma que o Nicolau sugeriu, ou pelo menos
não contam dessa forma *naturalmente*... mas ainda assim, é questão de
gosto.

Já o 0^0=1 eu não concordo mesmo...

Abraços.

Hugo.

Em 24 de março de 2011 18:55, Ralph Teixeira ralp...@gmail.com escreveu:

 Acho que a primeira convenção é útil, principalmente por dois motivos:

 i) Ela me permite escrever um polinômio de grau M como
 p(x)=SUM (n=0 a M) a_n x^n
 sem eu ter que ficar me preocupando com o caso x=0.

 ii) Se f(x) e g(x) são analíticas em volta de x=a, com  f(x)=0, e
 lim(x-a)f(x)=lim(x-a)g(x)=0, então lim(x-a) f^g=1 (exceto se f for
 identicamente nula). Em outras palavras, todas as indeterminações do
 tipo 0^0 dão 1, com raras exceções. Então a indeterminação vira uma
 conta simples (mas que deve ser usada com algum cuidado).

 Quanto à segunda... pô, EU QUERO contar o conjunto vazio :) :) :) E
 prefiro
 {0,1,2,...}=N=naturais e {1,2,3}=N*=naturais positivos
 a
 {0,1,2,...}=Z+=inteiros não-negativos (ou NU{0}) e
 {1,2,3,...}=N=naturais.
 A primeira opção tem menos bits... :) :)

 A propósito, uma vez o Nicolau me apresentou um argumento interessante:
 A gente DEVIA usar o 0 para contar. Se há cinco balas na mesa, você
 tinha que contar assim: 0 (na primeira bala), 1 (na segunda), 2, 3, 4.
 O número de balas é o primeiro número contador que NÃO FOI DITO.
 Neste caso, 5.
 Sete balas? 0,1,2,3,4,5,6, então o cardinal é 7.
 Zero balas? Você não diz nada, e o primeiro que não foi dito é 0. Viu,
 funciona!

 Mas, sim, concordo que o 0 exige um grau de abstração bem maior que os
 outros, então é menos natural, no sentido literal em português... E
 a vantagem de poder contar o conjunto vazio com o mesmo algoritmo
 dos outros é bem irrelevante... :) :) :) :)

 Enfim, tangerina tudo bem, mas totó é muita onomatopeia pro meu
 caminhãozinho... :) :) :)

 Abraço,
  Ralph

 2011/3/24 Hugo Fernando Marques Fernandes hfernande...@gmail.com:
  0^0 = 1?
  Sempre achei que 0^0 era uma indeterminação...
 
  Fora isso, dizer que 0 é natural é um assunto controverso, afinal números
  naturais são originários do processo de contagem... e ao contar,
 começamos
  por 1, não por zero... ou seja, o zero não é natural, ou depende de um
 grau
  de abstração maior que os demais números naturais, pelo menos.
 
  Só pra alimentar a polêmica, rss
 
  Abraços.
 
  Hugo
 
  Em 23 de março de 2011 18:18, Ralph Teixeira ralp...@gmail.com
 escreveu:
 
  Minha resposta é diplomática -- depende do que você chamar de
  fração. Defina do seu jeito, que seja conveniente para o que você quer
  fazer, e deixe claro a todos o que você está fazendo. Depois, seja
  coerente.
 
  (Ou seja, enrolei enrolei e não respondi.)
 
  Em Minha Modestíssima Opinião, fração é qualquer expressão do tipo a/b
  onde a e b são números ou até mesmo outras expressões. Então 1/(raiz
  de 2) é uma fração tanto quando 7/1 ou 25/pi ou (x+cos(y))/(z+w^2). Eu
  também diria que 3 não é uma fração, mas pode ser escrito como 3/1,
  que é uma fração... para mim, 45.78 não é fração, mas PODE SER ESCRITO
  como uma fração, 4578/100.
 
  Mas isso tudo é EMMO... Não, minto, é EMMC (Em Minha Modestíssima
  Convenção). Poxa, EMMC, 0 é natural, 0^0=1, aquele futebol com
  jogadores de madeira é pebolim e aquela fruta é mixirica Não
  gostou? Vai encarar? :) :) :) :)
 
  Abraço,
  Ralph
 
  2011/3/21 fabio henrique teixeira de souza fabiodja...@ig.com.br:
   Senhores, 1/(raiz de 2) é uma fração?
 
 
 =
  Instruções para entrar na lista, sair da lista e usar a lista em
  http://www.mat.puc-rio.br/~obmlistas/obm-l.html
 
 =
 
 

 =
 Instruções para entrar na lista, sair da lista e usar a lista em
 http://www.mat.puc-rio.br/~obmlistas/obm-l.html
 =



[obm-l] Re: [obm-l] Re: [obm-l] Re: [obm-l] Re: [obm-l] Re: [obm-l] Re: [obm-l] FRAÇÕES - conceito

2011-03-28 Por tôpico Hugo Fernando Marques Fernandes
Concordo, Ralph.
O mais importante é ter consciência das razões para escolher uma forma ou
outra e ser consistente no uso dessas convenções.

Um grande abraço.

Hugo.

Em 28 de março de 2011 16:58, Ralph Teixeira ralp...@gmail.com escreveu:

 Oi, Hugo.

 Realmente, as exceções são o principal problema -- com a minha
 convenção, eu tenho que lembrar dessas exceções o tempo todo (função
 f=0 ou funções não-analíticas). Sim, minha convenção é perigosa nesse
 sentido.

 Quanto ao p(x), acho chato separar aquele a_0. Além disso, agora eu
 vou querer derivar p(x) -- como escrever o que dá? Viu, é chato, agora
 você vai ter que separar o a_1. Argh. :)

 Mas, claro, como dissemos, é tudo questão de gosto -- o que não
 significa que é totalmente aleatório. :)

 Abraço,
 Ralph

 2011/3/25 Hugo Fernando Marques Fernandes hfernande...@gmail.com:
  Quanto a 0^0=1... Como vc disse, todas as indeterminações do tipo 0^0
 dão
  1, com raras exceções. O problema é que as exceções são raras mas elas
  existem, então não se pode afirmar a igualdade.
 
  Além disso, escrever p(x)=SUM [(n=1 a M) a_n x^n] + a_0, por exemplo, não
 me
  parece algo tão complicado.
 
  Quanto aos naturais, concordo que dizer que 0 é natural é uma convenção e
  pode até ser útil em determinadas situações... além de não introduzir
 nenhum
  problema, exceto uma mera questão de coerência linguística e
 antropológica -
  seres humanos não contam dessa forma que o Nicolau sugeriu, ou pelo menos
  não contam dessa forma naturalmente... mas ainda assim, é questão de
 gosto.
 
  Já o 0^0=1 eu não concordo mesmo...
 
  Abraços.
 
  Hugo.
 
  Em 24 de março de 2011 18:55, Ralph Teixeira ralp...@gmail.com
 escreveu:
 
  Acho que a primeira convenção é útil, principalmente por dois motivos:
 
  i) Ela me permite escrever um polinômio de grau M como
  p(x)=SUM (n=0 a M) a_n x^n
  sem eu ter que ficar me preocupando com o caso x=0.
 
  ii) Se f(x) e g(x) são analíticas em volta de x=a, com  f(x)=0, e
  lim(x-a)f(x)=lim(x-a)g(x)=0, então lim(x-a) f^g=1 (exceto se f for
  identicamente nula). Em outras palavras, todas as indeterminações do
  tipo 0^0 dão 1, com raras exceções. Então a indeterminação vira uma
  conta simples (mas que deve ser usada com algum cuidado).
 
  Quanto à segunda... pô, EU QUERO contar o conjunto vazio :) :) :) E
  prefiro
  {0,1,2,...}=N=naturais e {1,2,3}=N*=naturais positivos
  a
  {0,1,2,...}=Z+=inteiros não-negativos (ou NU{0}) e
  {1,2,3,...}=N=naturais.
  A primeira opção tem menos bits... :) :)
 
  A propósito, uma vez o Nicolau me apresentou um argumento interessante:
  A gente DEVIA usar o 0 para contar. Se há cinco balas na mesa, você
  tinha que contar assim: 0 (na primeira bala), 1 (na segunda), 2, 3, 4.
  O número de balas é o primeiro número contador que NÃO FOI DITO.
  Neste caso, 5.
  Sete balas? 0,1,2,3,4,5,6, então o cardinal é 7.
  Zero balas? Você não diz nada, e o primeiro que não foi dito é 0. Viu,
  funciona!
 
  Mas, sim, concordo que o 0 exige um grau de abstração bem maior que os
  outros, então é menos natural, no sentido literal em português... E
  a vantagem de poder contar o conjunto vazio com o mesmo algoritmo
  dos outros é bem irrelevante... :) :) :) :)
 
  Enfim, tangerina tudo bem, mas totó é muita onomatopeia pro meu
  caminhãozinho... :) :) :)
 
  Abraço,
   Ralph
 
  2011/3/24 Hugo Fernando Marques Fernandes hfernande...@gmail.com:
   0^0 = 1?
   Sempre achei que 0^0 era uma indeterminação...
  
   Fora isso, dizer que 0 é natural é um assunto controverso, afinal
   números
   naturais são originários do processo de contagem... e ao contar,
   começamos
   por 1, não por zero... ou seja, o zero não é natural, ou depende de um
   grau
   de abstração maior que os demais números naturais, pelo menos.
  
   Só pra alimentar a polêmica, rss
  
   Abraços.
  
   Hugo
  
   Em 23 de março de 2011 18:18, Ralph Teixeira ralp...@gmail.com
   escreveu:
  
   Minha resposta é diplomática -- depende do que você chamar de
   fração. Defina do seu jeito, que seja conveniente para o que você
 quer
   fazer, e deixe claro a todos o que você está fazendo. Depois, seja
   coerente.
  
   (Ou seja, enrolei enrolei e não respondi.)
  
   Em Minha Modestíssima Opinião, fração é qualquer expressão do tipo
 a/b
   onde a e b são números ou até mesmo outras expressões. Então 1/(raiz
   de 2) é uma fração tanto quando 7/1 ou 25/pi ou (x+cos(y))/(z+w^2).
 Eu
   também diria que 3 não é uma fração, mas pode ser escrito como 3/1,
   que é uma fração... para mim, 45.78 não é fração, mas PODE SER
 ESCRITO
   como uma fração, 4578/100.
  
   Mas isso tudo é EMMO... Não, minto, é EMMC (Em Minha Modestíssima
   Convenção). Poxa, EMMC, 0 é natural, 0^0=1, aquele futebol com
   jogadores de madeira é pebolim e aquela fruta é mixirica Não
   gostou? Vai encarar? :) :) :) :)
  
   Abraço,
   Ralph
  
   2011/3/21 fabio henrique teixeira de souza fabiodja...@ig.com.br:
Senhores, 1/(raiz de 2) é uma fração

[obm-l] Re: [obm-l] Re: [obm-l] FRAÇÕES - conceito

2011-03-24 Por tôpico Hugo Fernando Marques Fernandes
0^0 = 1?
Sempre achei que 0^0 era uma indeterminação...

Fora isso, dizer que 0 é natural é um assunto controverso, afinal números
naturais são originários do processo de contagem... e ao contar, começamos
por 1, não por zero... ou seja, o zero não é natural, ou depende de um grau
de abstração maior que os demais números naturais, pelo menos.

Só pra alimentar a polêmica, rss

Abraços.

Hugo

Em 23 de março de 2011 18:18, Ralph Teixeira ralp...@gmail.com escreveu:

 Minha resposta é diplomática -- depende do que você chamar de
 fração. Defina do seu jeito, que seja conveniente para o que você quer
 fazer, e deixe claro a todos o que você está fazendo. Depois, seja
 coerente.

 (Ou seja, enrolei enrolei e não respondi.)

 Em Minha Modestíssima Opinião, fração é qualquer expressão do tipo a/b
 onde a e b são números ou até mesmo outras expressões. Então 1/(raiz
 de 2) é uma fração tanto quando 7/1 ou 25/pi ou (x+cos(y))/(z+w^2). Eu
 também diria que 3 não é uma fração, mas pode ser escrito como 3/1,
 que é uma fração... para mim, 45.78 não é fração, mas PODE SER ESCRITO
 como uma fração, 4578/100.

 Mas isso tudo é EMMO... Não, minto, é EMMC (Em Minha Modestíssima
 Convenção). Poxa, EMMC, 0 é natural, 0^0=1, aquele futebol com
 jogadores de madeira é pebolim e aquela fruta é mixirica Não
 gostou? Vai encarar? :) :) :) :)

 Abraço,
 Ralph

 2011/3/21 fabio henrique teixeira de souza fabiodja...@ig.com.br:
  Senhores, 1/(raiz de 2) é uma fração?

 =
 Instruções para entrar na lista, sair da lista e usar a lista em
 http://www.mat.puc-rio.br/~obmlistas/obm-l.html
 =



[obm-l] Re: [obm-l] Re: [obm-l] Dúvida - OBM Nível Universitário

2011-01-24 Por tôpico Hugo Fernando Marques Fernandes
Oi, Bruna.

Pois é, eu já tinha ouvido dizer isso e queria confirmar.
É uma pena, mas fazer o que? Regras são regras...

Talvez fosse o caso de criar uma categoria nova pra quem já tem diploma, né?

Obrigado pela resposta.

Hugo.

Em 24 de janeiro de 2011 02:20, Bruna Campos bda.cam...@gmail.comescreveu:

 PS.: E só até o quarto ano de graduação.

 Abraços!

 Em 23/01/11, Bruna Camposbda.cam...@gmail.com escreveu:
  Hugo, que eu saiba não pode. Só pode participar quem não tem diploma
  de curso superior :(
 
 
 
 
  Em 20/01/11, Hugo Fernando Marques Fernandeshfernande...@gmail.com
  escreveu:
  Boa noite.
 
  Acabo de ser aprovado para o curso de matemática da UERJ e tenho uma
  dúvida
  em relação à OBM de nível universitário. Sendo esta minha segunda
  graduação,
  ou seja, possuindo um outro diploma de nível superior, ainda assim
  poderei
  participar?
 
  Grato por sua atenção.
 
  Hugo F. M. Fernandes.
 
 

 =
 Instruções para entrar na lista, sair da lista e usar a lista em
 http://www.mat.puc-rio.br/~obmlistas/obm-l.htmlhttp://www.mat.puc-rio.br/%7Eobmlistas/obm-l.html
 =



[obm-l] Re: [obm-l] Equação do segundo grau(raiz inteira)

2011-01-11 Por tôpico Fernando Oliveira
Expandindo, temos

(ax - b)^2 + ( bx - a)^2 = x
(a² + b²)x² - (4ab + 1)x + a² + b² = 0

(Estou supondo que a² + b² != 0. O caso contrário é simples, já que 0 seria
raiz)

Note que o produto das raízes é c/a = 1. Logo, se x é raiz, a outra raiz é
1/x. Além disso, a soma das raízes é inteira (4ab + 1), de onde tiramos que
as duas raízes devem ser inteiras.
Resta apenas pensar quais números poderíamos ter tais que x e 1/x sejam
inteiros, são 1 e -1.

-1 claramente não é possível:

(-a - b)^2 + (-b - a)^2 = -1

1 também falha:

(a - b)^2 + (b - a)^2 = 1
2(a - b)² = 1
(a - b)² = 1/2
a - b = +- 1/4

Não há solução inteira para a,b.

Logo, se a equação tem uma raiz inteira, devemos ter a = b = x = 0.

Fernando


  #  #
#   #


[obm-l] Re: [obm-l] Equação do segundo grau(raiz inteira)

2011-01-11 Por tôpico Fernando Oliveira
Eu acho que eu deveria parar de pensar em problemas 2 horas da manhã.
Esqueci de dividir a soma por a²+b²... Ignore a solução, embora eu ache que
a resposta final está correta (não achei nenhum outro caso que funcione...).

Fernando
  #  #
#   #


Re: [obm-l] Teoria dos Conjuntos

2011-01-02 Por tôpico Fernando Oliveira
A meu ver, o único jeito de termos x∈x seria se x fosse um conjunto
infinito. Por exemplo, seja B = {B}. Daí temos B = {{B}} = {{{B}}} =
B}}} = {{{...}}}.
Então A = R e B = ∅, ou estou simplificando demais as coisas?

Fernando


Re: [obm-l] Teoria dos Conjuntos

2010-12-21 Por tôpico Hugo Fernando Marques Fernandes
A relação de pertinência relaciona um elemento a um conjunto. Assim, não há
propriedade em escrever x∉x, pois estaríamos usando a relação de pertinência
para relacionar dois elementos.

[]'s

Hugo

Em 21 de dezembro de 2010 12:45, Vinícius Harlock cortes...@gmail.comescreveu:

 É possível criar por meio do axioma da especificação um conjunto B={x ∈ A;
 x∉x}? Esse conjunto seria um conjunto vazio assim como C={x ∈ A; x diferente
 x}?


Re: [obm-l] Probabilidade

2010-12-21 Por tôpico Hugo Fernando Marques Fernandes
1) 6/10*3/10+4/10*2/10 = 26/100 = 26%
2) 21733/51745 = 0,42 = 42%

[]'s

Hugo.

Em 25 de novembro de 2010 23:31, elyson gabriel gabr...@hotmail.comescreveu:

  1) Dois tipos de vacinas foram aplicadas em uma população de tal forma que
 60% das pessoas receberam vacina do tipo A e as 40% restantes receberam
 vacina do tipo B. Sabe-se que a vacina do tipo A fornece 70% de imunizaçao e
 a B fornece 80%. Determine a probabilidade de uma pessoa escolhida ao acaso
 não esteja imunizada.


 2) Muitos sistemas escolares fornecem acesso à internet para seus
 estudantes hoje em dia. Desde 1996, o acesso a internet foi facilitado a
 21.733 escolas elementares, 7.286 escolas de nível médio e 10.682 escolas de
 nível superior. Existe nos EUA um total de 51.745 escolas elementares,
 14.012 escolas de nível médio e 17.229 escolas de nível superior. Se voce
 escolher aleatoriamente uma escola elementar para visitar, qual é a
 probabilidade de que ela tenha acesso a internet?



[obm-l] Re: [obm-l] C(n,p): par ou ímpar?

2010-11-20 Por tôpico Fernando Oliveira
C(n,p) = n! / (n-p)!p!

Conte os fatores de 2 em n!, (n-p)! e p!. Se o número de fatores de 2 for
maior no numerador, o número é par, se for igual, é ímpar.

Ex: C(36, 24) = 36! / 24!12!

(pegando a parte inteira das divisões)

fatores de 2 em 36!: 36/2 + 36/4 + 36/8 + 36/16 + 36/32 = 18 + 9 + 4 + 2 + 1
= 34
fatores de 2 em 24!: 24/2 + 24/4 + 24/8 + 24/16 = 12 + 6 + 3 + 1 = 22
fatores de 2 em 12!: 12/2 + 12/4 + 12/8 = 6 + 3 + 1 = 10

Como 34  22 + 10, o número é par.

Fernando


[obm-l] Re: [obm-l] Permutação Circular

2010-11-16 Por tôpico Fernando Oliveira
Coloque as mulheres circularmente na mesa, depois encaixe os homens nos
espaços vazios.

Fernando





[obm-l] Re: [obm-l] Re: [obm-l] Re: [obm-l] Polinômios(ajud a)

2010-11-13 Por tôpico Fernando Oliveira
Bernardo, acho que você se esqueceu de um detalhe, o argumento não
funcionaria para 3 raízes.

Seja o polinômio P(x) = x³ - 10x² + 16x + 7.
Temos P(0) = P(2) = P(8) = 7 e P(1) = 14.

Qual é o detalhe? Bem, acho que vou deixar pra você descobrir. O polinômio
acima é bem sugestivo...

Fernando


Re: [obm-l] Material com provas do IME

2010-11-11 Por tôpico Fernando Oliveira
Sergio,

Estou confuso quanto à solução da 5ª questão de álgebra de 1989/1990. Nela é
afirmado que A segunda diretriz é ortogonal à primeira, mas as duas
diretrizes de uma elipse não são paralelas?

Fernando


[obm-l] Re: [obm-l] RE: [obm-l] RE: [obm-l] questão básica de probabilidade

2010-10-20 Por tôpico Hugo Fernando Marques Fernandes
Se não há dois bilhetes iguais, então ocorre 1 de dois casos: o número de
João é maior que o do Manuel ou vice-versa.

Então: 1 caso favorável (João  Manoel) / 2 casos posíveis = 1/2 = 50%.

Abraços

Hugo.


2010/10/18 João Maldonado joao_maldona...@hotmail.com

  De olho escolheria a letra D, mas se quisesse fazer contas.

 Caso Manuel escolha o 1, João tem 59/59 chances de tirar um bilhete maior.

 Caso Manuel escolha o 2, João tem 58/59 chances de tirar um bilhete maior.

  .
 .
 .

 Caso Manuel escolha o 60, João tem 0/59 chances de tirar um bilhete maior.

 Somando tudo temos  ((59.60)/2)/59 =  30
 dividindo por 60 opções possíveis a probabilidade é 30/60 = 50%.

 Vai outro problema: E se existissem 120 bilhetes, númerados de 1 a 60, cada
 bilhete com seu par, qual a chance de João tirar um bilhete maior?

 --
 From: nathalia...@hotmail.com

 To: obm-l@mat.puc-rio.br
 Subject: [obm-l] RE: [obm-l] questão básica de probabilidade
 Date: Mon, 18 Oct 2010 23:49:30 +



 Por um acaso a resposta seria letra d)?
 --
 From: eduvfsi...@gmail.com
 Date: Mon, 18 Oct 2010 18:02:35 -0300
 Subject: [obm-l] questão básica de probabilidade
 To: obm-l@mat.puc-rio.br

 João e Manuel retiram, para cada um, um bilhete de uma urna em que há 60
 bilhetes numerados de 1 a 60.
 A probabilidade de que o número retirado por João seja maior do que o de
 Manuel é:

 a) 31/60
 b) 60/59
 c) 60%
 d) 50%
 e) 29/60

 Achei que era a alternativa e), mas não é, alguém me explica por que?



[obm-l] Re: [obm-l] Re: [obm-l] RES: [obm-l] RE: [obm-l] RES : [obm-l] Dízima periódica

2010-10-19 Por tôpico Fernando Oliveira
Na verdade, a condição é que, se a fração for irredutível, o denominador não
pode ter fatores primos que não estejam presentes na base. Então qualquer
denominador que só tenha 2 e 5 como fatores primos não gerará dízima na base
10, assim como o denominador 6 não gerará dízima nas bases que tiverem 2 e 3
como divisores (ou seja, os múltiplos de 6).

Fernando


Re: [obm-l] Ajuda em problema

2010-09-28 Por tôpico Fernando Oliveira
Considere que o lote construído tem dimensões x e y. Considerando o recuo,
temos (x+14)(y+10) = 1226 - y = 1226/(x+14) - 10.

A área construída é, portanto, xy = x(1226/(x+14) - 10).
O domínio dessa função vai de x = 0 até y = 0 - x = 108,6.

Fernando





Re: [obm-l] Alguns problemas da prova

2010-09-21 Por tôpico Fernando Oliveira
Bem, agora que saiu o gabarito não tem muito propósito te falar as
respostas, mas só pra comemorar que eu fechei a prova (ae!), pena que
segunda fase conta muito pouco na classificação final.

Parte A
1) 25
2) 8
3) 12
4) 2592
5) 1057

Parte B
1) 60º
2) 144
3) não há solução
4) 1004

Fernando


Re: [obm-l] Alguns problemas da prova

2010-09-20 Por tôpico Fernando Oliveira
Não tinha algo sobre não divulgar as questões da prova da OBM? Só vou
comentar que o Salhab pulou k=8 ali na questão 3 da parte B. Além disso, as
minhas respostas da 1B e 4B (só 3?) não batem com as suas...

Fernando


Re: [obm-l] Ajuda

2010-08-28 Por tôpico Hugo Fernando Marques Fernandes
Como o total é 100, temos 30 pessoas que não são desonestas. Se dissermos
que todas 30 são intolerantes, como há 70 pessoas intolerantes, haverão
70-30=40 pessoas desonestas e intolerantes, e 60 pessoas que não são
desonestas e intolerantes simultâneamente. Se estas 60 forem todas
violentas, como há 70 pessoas violentas, existem 70-60=10 pessoas que são
violentas, intolerantes e desonestas simultaneamente.

Acho que é isso.
Abraços.

Hugo.


 Em 28 de agosto de 2010 22:39, marcone augusto araújo borges 
marconeborge...@hotmail.com escreveu:

  Alguem poderia ajudar a resolver essa questao?Uma pessoa cetica quanto as
 boas intenções da humanidade afirma q 70% dos homens sao desonestos,70% sao
 intolerantes e 70% sao violentos.Se ela estiver certa,numa amostra perfeita
 de 100 homens,qual o numero minimo de pessoas simultaneamente
 desonestas,intolerantes e violentas?



Re: [obm-l] FW: Nosso calendario

2010-08-28 Por tôpico Hugo Fernando Marques Fernandes
Não faltou considerar os anos bissextos?

Abraços.

Hugo.

Em 29 de agosto de 2010 00:26, marcone augusto araújo borges 
marconeborge...@hotmail.com escreveu:

  Obrigado,abraços.

 --
 Date: Sat, 28 Aug 2010 23:41:47 -0300
 Subject: Re: [obm-l] FW: Nosso calendario
 From: msbro...@gmail.com

 To: obm-l@mat.puc-rio.br

 Vamos ver a qtde de dias de cada mês, em ordem:
 31, 28, 31, 30, 31, 30, 31, 31, 30, 31, 30, 31

 Analisando isso módulo 7, visto que são 7 dias da semana, temos:
 28 == 0 (mod 7)
 30 == 2 (mod 7)
 31 == 3 (mod 7)

 Desta maneira, temos:
 3, 0, 3, 2, 3, 2, 3, 3, 2, 3, 2, 3

 Supondo que o primeiro dia 13 esteja em k, temos:
 k, k+3, k+3, k+6, k+8, k+11, k+13, k+16, k+19, k+21, k+24, k+26

 Analisando mod 7, temos:
 k, k+3, k+3, k+6, k+1, k+4, k+6, k+2, k+5, k, k+3, k+5

 Veja que temos todos os inteiros mod 7 somando com k.
 Desta maneira, sempre há uma sexta feira 13.

 abraços,
 Salhab



 2010/8/28 marcone augusto araújo borges marconeborge...@hotmail.com



 --
 From: marconeborge...@hotmail.com
 To: obm-l@mat.puc-rio.br
 Subject: Nosso calendario
 Date: Sun, 29 Aug 2010 02:12:05 +

 Mostre q em qualquer ano existe pelo menos uma sexta-feira 13.Eu acho q
 consegui resolver,mas gostaria de ver outra solução.
 Fiz assim:se o dia 13 de janeiro  é um domingo,entao o dia 13 de setembro é
 uma sexta pois,contando apenas o numero
 de dias q passam de 28 em cada mes,a partir de janeiro(até
 agosto),encontramos 19 dias(um multiplo de 7 mais 5),dai,conside-
 rando o domingo como dia 1,temos 1+5=6(sexta).Usei o mesmo raciocinio para
 o caso do dia 13 de janeiro ser segunda,terça,quarta,quinta ou sabado e
 encontrei para cada caso uma sexta feira 13 no mesmo ano.





[obm-l] Retirar nome da lista.

2009-12-26 Por tôpico José Fernando Malta
Gostaria que meu nome e e-mail fosse retirado da lista.

Grato.

Fernando.


Re: [obm-l] Vetores

2009-10-18 Por tôpico Hugo Fernando Marques Fernandes
1)

(1,0,1)x + (0,2,1)y + (1,-1,1)z = (2,1,3)

(I) x+z = 2
(II) 2y-z=1
(III) x+y+z = 3

(IV) = (I) em (III) = y + 2 = 3 = y = 1

(V) = (IV) em (II) = 2 - z = 1 = z = 1

(V) em (I) = x + 1 = 2 = x = 1

Então: (2,1,3) pertence ao subespaço vetorial do R3 gerado pelos vetores
(1,0,1) , (0,2,1), (1,-1,1).

2)

(-1,2,1)x + (1,0,2)y + (2,-2,1)z = (a,b,c)

-x+y+2z = a = x =y+2z-a

2x-2z=b = 2y+4z-2a-2z=b = 2y+2z-2a = b = y = (b+2a-2z)/2

x+2y+z=c = y+2z-a+2y+z=c = 3y+3z-a = c = 3/2(b+2a-2z) +3z - a = c = 3b/2
+ 3a - 3z + 3z - a = c = 3b/2 + 2a = c = 3b + 4a = 2c

R: 3b + 4a = 2c

2009/10/17 Bruna Carvalho bruna.carvalho.p...@gmail.com

 Poderiam me ajudar com essas duas questões?

 1) Verificar se o vetor α = (2,1,3) pertence ao subespaço vetorial do R3,
 gerado pelos vetores (1,0,1) , (0,2,1), (1,-1,1).

 2) Qual a relação entre a, b e c, para que o vetor (a,b,c) do R3 pertença
 ao subespaço vetorial gerado por (-1,2,1), (1,0,2) e (2,-2,1) ?


 --
 Bjos,
 Bruna



[obm-l] Re: [obm-l] Questão sobre poliinômios

2009-10-16 Por tôpico Hugo Fernando Marques Fernandes
R(x) tem grau  2.

R(x) = ax + b ( a,b possivelmente nulos)

P(x) = Q1(x).(x-2)(x-3) + R(x)
P(2) = R(2) = -1
P(3) = R(3) = 2

P(x) = Q(x)(x-2) + R1(x)
P(2) = R1(x) = -1
P(3) = Q(3)(3-2) + R1(x) = 3 + R1(x) = 3 - 1 = 2

R1(x) tem grau  1
R1(x) = a = -1

2a + b = -1
3a + b = 2

a = 2 - (-1) = 3

b = - 1 - 2a = -7

R(x) = 3x - 7

2009/10/16 João Paulo V. Bonifácio joaop.bonifa...@gmail.com

 Pessoal,

 Se alguém puder me ajudar nesta questão, eu ficarei muito grato. Já tentei
 fazer isso de todo jeito, mas até agora nada.

 *03. * Seja P(x) um polinômio tal que P (2) = – 1. Suponhamos que o
 quociente Q(x) da divisão de P(x)  por x – 2 seja tal que Q(3) = 3.
 Determine o resto R(x) da divisão de P(x) por (x – 2 ).(x – 3).

 Abraços!

 --
 João Paulo Vieira Bonifácio

 Universidade Federal de Uberlândia
 Faculdade de Engenharia Elétrica
 Programa de Educação Tutorial - PET/Eng. Elétrica
 Fone: (34) 9942 - 7427 / (34) 3239 - 4754



Re: [obm-l] Problema

2009-09-24 Por tôpico Hugo Fernando Marques Fernandes
Soma dos termos de uma P.A.

S = n*(a1+aN) / 2

n = 999 - 100 + 1 = 900

a1 = 100

aN = 999

S = 900*(100+999)/2 = 494550

Abraços.
Hugo.

2009/9/24 Paulo Barclay Ribeiro paulobarc...@yahoo.com.br

 Prezados,

 Peço uma ajuda (orientação)na resolução do seguinte problema:
 Qual o valor da soma de todos os numeros naturais de três algarismos?

 Desde  já agradeço a gentileza

 Paulo Barclay

 --
 Veja quais são os assuntos do momento no Yahoo! + Buscados: Top 
 10http://br.rd.yahoo.com/mail/taglines/mail/*http://br.maisbuscados.yahoo..com/-
 Celebridadeshttp://br.rd.yahoo.com/mail/taglines/mail/*http://br.maisbuscados.yahoo.com/celebridades/-
 Músicahttp://br.rd.yahoo.com/mail/taglines/mail/*http://br.maisbuscados.yahoo..com/m%C3%BAsica/-
 Esporteshttp://br.rd.yahoo.com/mail/taglines/mail/*http://br.maisbuscados.yahoo.com/esportes/



[obm-l] Re: [obm-l] Re: [obm-l] Re: [obm-l] Informação

2009-09-15 Por tôpico Hugo Fernando Marques Fernandes
Rita.

Veja este aqui, é muito bom.

http://www.ime.uerj.br/ensinoepesquisa/publicacoes.html

Abraços.
Hugo.

2009/9/13 RitaGomes rcggo...@terra.com.br

  Ok fico grata, mas alguns que encontrei estão todos em Ingles, ams vou
 procurar novamente.

 Rita Gomes

 - Original Message -
 *From:* Carlos Gomes cgomes...@uol.com.br
 *To:* obm-l@mat.puc-rio.br
 *Sent:* Sunday, September 13, 2009 11:32 AM
 *Subject:* [obm-l] Re: [obm-l] Informação

 Rita...vê em

 www.4shared.com

 tem milhares!

 - Original Message -
 *From:* RitaGomes rcggo...@terra.com.br
 *To:* OBM Lista obm-l@mat.puc-rio.br
 *Sent:* Sunday, September 13, 2009 10:34 AM
 *Subject:* [obm-l] Informação

 Caros Colegas,

 Alguem pode me informar algum endereço eletronico em que posso baixar algum
 livro de cálculo bom

 Rita Gomes

 --


 No virus found in this incoming message.
 Checked by AVG - www.avg.com
 Version: 8.5.409 / Virus Database: 270.13.94/2367 - Release Date: 09/13/09
 05:50:00

  --
 Esta mensagem foi verificada pelo E-mail Protegido Terra.
 Atualizado em 13/09/2009

  --


 No virus found in this incoming message.
 Checked by AVG - www.avg.com
 Version: 8.5.409 / Virus Database: 270.13.94/2367 - Release Date: 09/13/09
 05:50:00




[obm-l] Re: [obm-l] Teoria dos Números

2009-09-10 Por tôpico Hugo Fernando Marques Fernandes
Diogo.

Questão 01.

 (x - 1)(x² + x +1) = 2^n = x-1 = 2^k1 (I) e (x² + x +1) = 2^k2 (II) tal
que k1+k2 =  n.

somando (I) e (II)

x² + x +1 + x -1 = x² + 2x = x(x+2) =  2^k1 +  2^k2

Como
2^k1 é par
2^k2 é par

2^k1 + 2^k2 é par. = x(x+2) é par = x é par.

Porém de (I) x-1 é par!!!

x e x-1 pares = Absurdo.

Não existe x tal que (x - 1)(x² + x +1) = 2^n

Abraços.
Hugo.


2009/9/10 Diogo FN diog...@yahoo.com.br

  *Eu tava estudando e não consegui resolver, essas 3 questões.*

 01. Mostre que não existe x (natural) tal que (x - 1)(x² + x +1) = 2^n
 02. Determine todos os pares (x,n) (inteiros) tais que x² = 2^n + 1
 03. Fazer um estudo sobre as soluções da equação x^m = p^n + 1 , onde x,
 m,n, p são naturais e p é primo.

 Agradeço a todos.

  --
 Veja quais são os assuntos do momento no Yahoo! + Buscados: Top 
 10http://br.rd.yahoo.com/mail/taglines/mail/*http://br.maisbuscados.yahoo.com/-
 Celebridadeshttp://br.rd.yahoo.com/mail/taglines/mail/*http://br.maisbuscados.yahoo.com/celebridades/-
 Músicahttp://br.rd.yahoo.com/mail/taglines/mail/*http://br.maisbuscados.yahoo.com/m%C3%BAsica/-
 Esporteshttp://br.rd.yahoo.com/mail/taglines/mail/*http://br.maisbuscados.yahoo.com/esportes/



[obm-l] Re: [obm-l] Problema Prático

2009-09-04 Por tôpico Hugo Fernando Marques Fernandes
Coloca as duas camisinhas, uma sobre a outra e transa com a primeira, sem
risco.
Tira a de cima e deixa de lado, e transa com a segunda, sem risco.
Pega a que estava de lado e coloca de novo, pelo lado contrário e transa com
a terceira.

É isso? rsrsrsrsrs

Abraços e bom feriado a todos.

Hugo.

2009/9/4 luiz silva luizfelipec...@yahoo.com.br

 Pessoal,

 Me passaram este problema e achei bem interessante e instrutivo ::))

 Um homem contrata três prostitutas e quer fazer sexo com todas. Todos os
 envolvidos podem ter doenças sexualmente transmissíveis, e todos querem usar
 preservativos. Infelizmente, só há duas camisinhas. Pior ainda, estão no
 meio do nada e não podem comprar mais camisinhas. Poderá o homem fazer sexo
 com todas as três mulheres sem risco para qualquer um dos quatro

 --
 Veja quais são os assuntos do momento no Yahoo! + Buscados: Top 
 10http://br.rd.yahoo.com/mail/taglines/mail/*http://br.maisbuscados.yahoo.com/-
 Celebridadeshttp://br.rd.yahoo.com/mail/taglines/mail/*http://br.maisbuscados.yahoo.com/celebridades/-
 Músicahttp://br.rd.yahoo.com/mail/taglines/mail/*http://br.maisbuscados.yahoo.com/m%C3%BAsica/-
 Esporteshttp://br.rd.yahoo.com/mail/taglines/mail/*http://br.maisbuscados.yahoo.com/esportes/



[obm-l] Re: [obm-l] Re: [obm-l] RE: [obm-l] Problema Prátic o

2009-09-04 Por tôpico Hugo Fernando Marques Fernandes
Caro Paulo César.

Não estará se expondo ao risco ao realizar a inversão da camisinha
inicialmente deixada de lado porque neste momento ele ainda está usando a
primeira que teria colocado.

Tiago.

Um pouco mais de bom humor numa sexta véspera de feriado não lhe faria
nenhum mal.

Abraços.

Hugo.

2009/9/4 Paulo Cesar pcesa...@gmail.com

 Mas ao inverter a posição da camisinha inicialmente usada, não estará o
 homem expondo-se ao risco?
 Vamos supor que a primeira camisinha (a que ficou por cima da outra) tenha
 um lado A e um lado B. O lado B entra em contato com a primeira Prima, ao
 passo que o lado A fica limpo. Ao mudar de posição para usá-la com a
 terceira entrevistada do Superpop, o lado B entrará em contato direto com o
 homem.

 Ou será que estou errado?

 De qualquer forma, o problema é bem interessante. Mas tem sempre alguém
 (que provavelmente não conhece as nobres meretrizes) pra reclamar.

 Abraço

 PC

 2009/9/4 tiago lucas gouveia tiago-lucas-gouv...@hotmail.com

  Meu, vê se tem um pouco de respeito com as pessoas que participam dessa
 lista

 --
 Date: Fri, 4 Sep 2009 07:15:28 -0700
 From: luizfelipec...@yahoo.com.br
 Subject: [obm-l] Problema Prático
 To: obm-l@mat.puc-rio.br


   Pessoal,

 Me passaram este problema e achei bem interessante e instrutivo ::))

 Um homem contrata três prostitutas e quer fazer sexo com todas. Todos os
 envolvidos podem ter doenças sexualmente transmissíveis, e todos querem usar
 preservativos. Infelizmente, só há duas camisinhas. Pior ainda, estão no
 meio do nada e não podem comprar mais camisinhas. Poderá o homem fazer sexo
 com todas as três mulheres sem risco para qualquer um dos quatro

 --
 Veja quais são os assuntos do momento no Yahoo! + Buscados: Top 
 10http://br.rd.yahoo.com/mail/taglines/mail/*http://br.maisbuscados.yahoo.com/-
 Celebridadeshttp://br.rd.yahoo.com/mail/taglines/mail/*http://br.maisbuscados.yahoo.com/celebridades/-
 Músicahttp://br.rd.yahoo.com/mail/taglines/mail/*http://br.maisbuscados.yahoo.com/m%C3%BAsica/-
 Esporteshttp://br.rd.yahoo.com/mail/taglines/mail/*http://br.maisbuscados.yahoo.com/esportes/
 --
 Novo Internet Explorer 8: faça tudo com menos cliques. Baixe agora, é
 gratis!http://brasil.microsoft.com.br/IE8/mergulhe/?utm_source=MSN%3BHotmailutm_medium=Taglineutm_campaign=IE8





Re: [obm-l] Livro Disquisitiones Arithmeticae - Oportunidade

2009-09-01 Por tôpico Hugo Fernando Marques Fernandes
Marco.

Tb gostaria de receber o livro.
Obrigado.

Hugo.

2009/8/31 alexmay nunes soares alexmaynu...@yahoo.com.br

 Se você puder fazer a gentileza de enviar-me o livro eu ficaria muito
 grato, obrigado.

 --- Em *seg, 31/8/09, Marco Bivar marco.bi...@gmail.com* escreveu:


 De: Marco Bivar marco.bi...@gmail.com
 Assunto: [obm-l] Livro Disquisitiones Arithmeticae - Oportunidade
 Para: obm-l@mat.puc-rio.br
 Data: Segunda-feira, 31 de Agosto de 2009, 20:37


  Caros colegas, acredito que muitos de vocês conhecem pelo nome o livro de
 Gauss. Sabem também que até recentemente o único acesso que tinhamos a
 ele era através da tradução inglesa, e isso se o tivéssemos
 ou então o emprestássemos de algum amigo. Pois bem, aproveitem esta
 oportunidade. Uma tradução em espanhol foi feita em colaboração de países
 como a Costa Rica e a Argentina e está em formato digital pdf, pronto para
 baixar.

 O endereço é este: http://www.cimm.ucr.ac.cr/da/

 Façam bom uso teoristas, e caso tenham interesse, eu envio por e-mail o
 livro completo com todos os arquivos agrupados, pois no endereço acima eles
 estão separados por seção. Mande-me seu e-mail.

 --
 Marco Bivar


 start: -00-00 end: -00-00
 --
 Veja quais são os assuntos do momento no Yahoo! + Buscados: Top 
 10http://br.rd.yahoo.com/mail/taglines/mail/*http://br.maisbuscados.yahoo.com/-
 Celebridadeshttp://br.rd.yahoo.com/mail/taglines/mail/*http://br.maisbuscados.yahoo.com/celebridades/-
 Músicahttp://br.rd.yahoo.com/mail/taglines/mail/*http://br.maisbuscados.yahoo.com/m%C3%BAsica/-
 Esporteshttp://br.rd.yahoo.com/mail/taglines/mail/*http://br.maisbuscados.yahoo.com/esportes/



[obm-l] Re: [obm-l] Re: [obm-l] RE: [obm-l] Teoria dos núme ros (2 questões simples)

2009-08-21 Por tôpico Hugo Fernando Marques Fernandes
Todos os números são da forma 3k+1, 3k ou 3k-1. Como 3k não é primo, k1,
então todos os primos maiores que 3 são da forma 3k+1 ou 3k-1.

Abraços.

Hugo.

2009/8/21 luiz silva luizfelipec...@yahoo.com.br

   Ola Marcone,

 Pq vc trabalha com primos da forma 3k+1 ou 3k-1..creio que nem todos os
 primos podem ser representados nesta forma...O correto não seria 2k+1 ou
 2k-1 ??Ou então, se quiser representar primos maiores que 3, não seria
 correto trabalhar com primos da forma 3+2k?

 Abs
 Felipe

 --- Em *qui, 20/8/09, marcone augusto araújo borges 
 marconeborge...@hotmail.com* escreveu:


 De: marcone augusto araújo borges marconeborge...@hotmail.com
 Assunto: [obm-l] RE: [obm-l] Teoria dos números (2 questões simples)
 Para: obm-l@mat.puc-rio.br
 Data: Quinta-feira, 20 de Agosto de 2009, 21:10

 Olá,Diogo.Um comentário singelo:o único primo múltiplo de 3 é o próprio
 3.Se a é primo e diferente de 3, então a=3k+1 ou a=3k-1,dai a^2=3p+1 e a^2
 +2=3q, o que é uma contradição(pois a^2+2 é primo).Portanto a=3.Se eu
 estiver errado,certamente alguem irá corrigir.Um abraço.

 --
 Date: Thu, 20 Aug 2009 15:15:39 -0700
 From: diog...@yahoo.com.br
 Subject: [obm-l] Teoria dos números (2 questões simples)
 To: obm-l@mat.puc-rio.br

  Teoria dos números (2 questões simples)? 1. Mostre que se (a) e (a² + 2)
 são ambos primos então a=3
 2. Mostre que se (a² +ab +1) divide (b² +ab + 1) então a=b.


 Se puder ajudar, agradeço.

 --
 Veja quais são os assuntos do momento no Yahoo! + Buscados: Top 
 10http://br.rd.yahoo.com/mail/taglines/mail/*http://br.maisbuscados.yahoo.com/-
 Celebridadeshttp://br.rd.yahoo.com/mail/taglines/mail/*http://br.maisbuscados.yahoo.com/celebridades/-
 Músicahttp://br.rd.yahoo.com/mail/taglines/mail/*http://br.maisbuscados.yahoo.com/m%C3%BAsica/-
 Esporteshttp://br.rd.yahoo.com/mail/taglines/mail/*http://br.maisbuscados.yahoo.com/esportes/
 --
 Novo Internet Explorer 8: mais rápido e muito mais seguro. Baixe agora, é
 grátis!http://brasil.microsoft.com.br/IE8/mergulhe/?utm_source=MSN%3BHotmailutm_medium=Taglineutm_campaign=IE8


 --
 Veja quais são os assuntos do momento no Yahoo! + Buscados: Top 
 10http://br.rd.yahoo.com/mail/taglines/mail/*http://br.maisbuscados.yahoo.com/-
 Celebridadeshttp://br.rd.yahoo.com/mail/taglines/mail/*http://br.maisbuscados.yahoo.com/celebridades/-
 Músicahttp://br.rd.yahoo.com/mail/taglines/mail/*http://br.maisbuscados.yahoo.com/m%C3%BAsica/-
 Esporteshttp://br.rd.yahoo.com/mail/taglines/mail/*http://br.maisbuscados.yahoo.com/esportes/



Re: [obm-l] Ligas Metalicas

2009-07-02 Por tôpico Hugo Fernando Marques Fernandes
(...) nessa nova liga, a razão entre OURO e PRATA deve ser  5 : 11 (...)

Não me parece que o enunciado diga isso. Na verdade, ele pergunta isso:
(...) Qual a razao entre as massas de ouro e prata na nova liga?

Na verdade, o enunciado diz: *as massas *de X e Y, na razao 5:11

Ou seja, na nova liga a razão entre o total de ouro e prata proveniente de X
e o total de ouro e prata proveniente de Y é 5:11.

Daí: 1 parte de X contém 2/5x de ouro e 3/5x de prata
   1 parte de Y contém 3/10x de ouro e 7/10x de prata
(sendo x o peso de uma parte)

Na nova liga, há 5 partes de X para 11 partes de Y, portanto:

2x + 33/10x ouro para 3x + 77/10x de prata

ou seja, 53/10x de ouro para 107/10x de prata

ou seja, a nova razão é 53/107.

Hugo.

2009/7/1 Palmerim Soares palmerimsoa...@gmail.com

 Olá Jose Aurimenes

 Na liga X há 2 partes de ouro e 3 de prata, então se 5x for a massa total
 da liga X, teremos 2x de ouro e 3x de prata.Usando o mesmo raciocínio para
 a liga Y teremos 3y de ouro e 7y de prata (a massa total da liga Y é 10y).
 Fundindo as massas das duas ligas, a nova liga terá:
 OURO: 2x + 3y ( *i* )
 PRATA: 3x + 7y( *ii* )

 Mas, nessa nova liga, a razão entre OURO e PRATA deve ser  5 : 11, então
 podemos escrever:
 11(2x + 3y) = 5(3x + 7y), donde: x = 2y/7. Substitua agora este valor de x
 em ( *i* ) e em ( *ii* ) e você terá (na nova liga) as massas 27y/7 de
 OURO e 45y/7 de PRATA. Portanto, a razão (na nova liga) entre as massas de
 OURO e PRATA é:
 (27y/7) / (45y/7) que é igual a 5/9, a razão pedida.

 Abraços

 2009/7/1 Jose Aurimenes profa...@yahoo.com.br

  Pessoal, peco ajuda na solucao.

 Dispomos de 2 ligas de ouro e prata. A liga X contem os metais,
 respectivamente, na razao de 2:3, e a liga Y  os contem, respectivamente, na
 razao 3:7. Fundindo as massas de X e Y, na razao 5:11, obtemos uma nova
 liga. Qual a razao entre as massas de ouro e prata na nova liga?

 Antecipadamente agradeco.

 Aurimenes




 --
 Palmerim



[obm-l] Re: [obm-l] Re: [obm-l] Re: [obm-l] Re: [obm-l] Teor ema da Ordinalidade dos Números Primos

2009-07-02 Por tôpico Hugo Fernando Marques Fernandes
Henrique.

Poderia colocar aqui a tal demonstração da falsidade do argumento de
Euclides, para que possamos discuti-la de forma mais consistente?

Abraços.

Hugo.

2009/7/2 Henrique Rennó henrique.re...@gmail.com



 2009/7/2 Bernardo Freitas Paulo da Costa bernardo...@gmail.com

 Oi Henrique e obm-l,

 2009/7/2 Henrique Rennó henrique.re...@gmail.com:
  No começo do texto você cita que pelo teorema de Euclides existem
 infinitos
  primos, mas o teorema não é válido, pois supõe que exista um primo maior
 que
  todos e demonstra que existe um outro primo maior que o maior, gerando
 uma
  inconsistência e assim concluindo que não há um maior primo, ou seja,
 são
  infinitos.
 Isso se chama prova por (redução ao) absurdo, e consiste numa das
 ferramentas mais uteis em matemática (pois nem todas as demonstrações
 são construtivas. Ah, Euclides era (e para muitos, continua sendo) um
 dos grandes fundadores da logica, portanto, se você acha que uma das
 demonstrações dele está errada, pense bem forte, e verifique bem o que
 você vai dizer.


 No livro Os Problemas do Milênio do autor Keith Devlin (que o Marco Bivar
 colocou como uma das referências), ele coloca em apêndice a demonstração que
 Euclides fez e diz que essa demonstração não é verdadeira. Posso colocar a
 demonstração aqui caso necessário. Ela é lógica e simples de entender.

 Conheço muitos problemas que são demonstrados por absurdo (ou por
 contradição), mas a falha da demonstração de Euclides está onde ele diz que
 o novo primo gerado a partir do suposto maior primo é um novo número primo,
 o que pelo mencionado no livro é falso já que através de outras teorias ou
 listagens de primos geradas por computador esse novo número pode ser um
 composto.



  Mas quando se faz a suposição de que existe um maior primo, já é
  uma falha do teorema.
 Justamente, isso se chama a hipótese de absurdo. E é justamente por
 ela ser falsa que se chega a uma contradição, e o principio do
 terceiro excluído garante que na verdade ela é realmente falsa.
 Existem sistemas lógicos onde proposições não são necessariamente
 falsas ou verdadeiras, existindo uma terceira possibilidade, mas
 isso é bastante discutido em filosofia, não tanto assim em matemática
 (mesmo que talvez devesse sê-lo !)

  Acredito que uma prova válida de que existem infinitos
  primos é através do somatório 1/2 + 1/3 + 1/5 + ... que foi demonstrado
 por
  Euler e converge para infinito.
 Ah, se você olhar bem, esta também é uma prova por absurdo : se fossem
 finitos números primos, a tal seqüência convergiria, e por um
 raciocínio muito esperto, se chega à conclusão de que a série
 harmônica divergiria, o que não é o caso !

 Abraços lógicos,
 --
 Bernardo Freitas Paulo da Costa

 =
 Instruções para entrar na lista, sair da lista e usar a lista em
 http://www.mat.puc-rio.br/~obmlistas/obm-l.htmlhttp://www.mat.puc-rio.br/%7Eobmlistas/obm-l.html
 =




 --
 Henrique



[obm-l] Re: [obm-l] ANÁLISE COMBINATÓRIA!

2009-07-02 Por tôpico Hugo Fernando Marques Fernandes
*Um exame consta de 4 provas. Os graus em cada matéria variam de 0 a 10,
aproximados até décimos. Qual o número mínimo de candidatos que nos
permitirá afirmar a existência de dois que tenham obtido notas idênticas?
*
É uma aplicação do chamado Princípio da Casa de Pombos. Existem 101 graus
possíveis (incluindo o grau 0) em cada prova.
Logo, existem 101^4 graus possíveis nas quatro provas combinadas. Assim, o
número pedido é 101^4+1.

*Quantos milhares sem algarismos repetidos podem ser formados com 2
algarismos pares e 2 ímpares significativos?*

Escolher dois algarismos pares significativos distintos: C(4,2)
Escolher dois algarismos ímpares significativos distintos: C(5,2)

Formas de escolher os quatro algarimos: C(4,2)*C(5,2)

Para cada escolha anterior, há 4! formas de montar o milhar (permutações).

Então, a resposta será: 4! * C(4,2) * C(5,2).

Depois faço os outros.

Abraços.

Hugo.

2009/6/29 Jorge Luis Rodrigues e Silva Luis jorgelrs1...@hotmail.com

  Olá, Pessoal!

 Um exame consta de 4 provas. Os graus em cada matéria variam de 0 a 10,
 aproximados até décimos. Qual o número mínimo de candidatos que nos
 permitirá afirmar a existência de dois que tenham obtido notas idênticas?

 Quantos milhares sem algarismos repetidos podem ser formados com 2
 algarismos pares e 2 ímpares significativos?

 Em quantas permutações dos algarismos 1, 2, 3, 4, 5 e 6 os equidistantes
 dos extremos somam 7?

 Quantos diferentes colares usando 13 pedras distintas podem ser feitos se
 virar o colar ao invés de rodar?

 Qual o número de maneiras que podemos colocar quatro bolas indistingüíveis
 em seis compartimentos separados?

 A propósito, quantos números tem todos os seus dígitos de igual paridade?
 Afinal! Qual o maior número  de interseções de 5 circunferências?


 Abraços!

 --
 Novo Internet Explorer 8: mais rápido e muito mais seguro. Baixe agora, é
 grátis!http://brasil.microsoft.com.br/IE8/mergulhe/?utm_source=MSN%3BHotmailutm_medium=Taglineutm_campaign=IE8



Re: [obm-l] Mais sobre bolas e urnas

2009-06-01 Por tôpico Fernando Lima Gama Junior
Boa, gostei da solução. Mas então vamos dificultar para exigir uma resposta
mais formal. Qual a chance de eu tirar pelo menos 2 bolas pretas?

Fernando Gama

Sent from Brasilia, DF, Brazil


2009/6/1 Rafael Ando rafael.a...@gmail.com

 Olha só... A chance de vc tirar todas brancas é: (9/10)^10 = 0.3487
 (aproximadamente). Caso isso não aconteça, vc tirou a preta pelo menos uma
 vez. A chance disso ocorrer é, portanto, 1-(0.9)^10 = 0.6513 (aprox.), ou
 65.13%.

 2009/6/1 Fernando Lima Gama Junior fgam...@gmail.com

  Outra questão.

 Tenha uma urna com 10 bolas, sendo uma preta e 9 brancas. Eu faço dez
 retiradas, com reposição. Qual a chance de que eu tenha retirado a bola
 preta pelo menos uma vez?


 Fernando Gama

 Sent from Brasília, Brazilian Federal District, Brazil




 --
 Rafael



Re: [obm-l] Mais sobre bolas e urnas

2009-06-01 Por tôpico Fernando Lima Gama Junior
Boa, gostei da solução. Mas então vamos dificultar para exigir uma resposta
mais formal. Qual a chance de eu tirar pelo menos 2 bolas pretas?

Fernando Gama

Sent from Brasilia, DF, Brazil


2009/6/1 Rafael Ando rafael.a...@gmail.com

 Olha só... A chance de vc tirar todas brancas é: (9/10)^10 = 0.3487
 (aproximadamente). Caso isso não aconteça, vc tirou a preta pelo menos uma
 vez. A chance disso ocorrer é, portanto, 1-(0.9)^10 = 0.6513 (aprox.), ou
 65.13%.

 2009/6/1 Fernando Lima Gama Junior fgam...@gmail.com

  Outra questão.

 Tenha uma urna com 10 bolas, sendo uma preta e 9 brancas. Eu faço dez
 retiradas, com reposição. Qual a chance de que eu tenha retirado a bola
 preta pelo menos uma vez?


 Fernando Gama

 Sent from Brasília, Brazilian Federal District, Brazil




 --
 Rafael



[obm-l] Mais sobre bolas e urnas

2009-05-31 Por tôpico Fernando Lima Gama Junior
Outra questão.

Tenha uma urna com 10 bolas, sendo uma preta e 9 brancas. Eu faço dez
retiradas, com reposição. Qual a chance de que eu tenha retirado a bola
preta pelo menos uma vez?


Fernando Gama

Sent from Brasília, Brazilian Federal District, Brazil


Re: [obm-l] Ajuda em probabilidade

2009-05-30 Por tôpico Fernando Lima Gama Junior
Também não entendi...
Fernando Gama




2009/5/29 Rafael Ando rafael.a...@gmail.com

 ?

 Rita, não entendo como vc está pensando...

 2009/5/29 RitaGomes rcggo...@terra.com.br

  Como agora ela esta na terceira posição, fazemos a permutação de 3, que
 6 e descontamos 1 condição ficando com 5 possibilidades de sair na terceira
 posição.

 - Original Message -
  *From:* Fernando Lima Gama Junior fgam...@gmail.com
 *To:* obm-l@mat.puc-rio.br
  *Sent:* Friday, May 29, 2009 8:56 PM
 *Subject:* Re: [obm-l] Ajuda em probabilidade

 Qual seria a chance, então, de ela ser tirada até a terceira bola?

 Fernando Gama

 Sent from Brasilia, Distrito Federal, Brazil


 2009/5/29 RitaGomes rcggo...@terra.com.br

  Fernando,

 Como são 5 bolas e 1 sendo preta, fazemos permutação de 5 que é 240,
 porem a preta deve ser a última a ser retirada. Faz permutação de 4, que é
 24 , sendo a possibilidade da bola preta sair em ordem diferente da última.
 Desconta do totoal das condições, ou seja 240 - 24 = 216 possibilidades
 de ser a última a ser retirada.
 Espero não ter feito o cálculo errado, pois estou meio atorodoada aqui
 com outros estudos.

 Rita Gomes

   - Original Message -
  *From:* Fernando Lima Gama Junior fgam...@gmail.com
 *To:* obm-l@mat.puc-rio.br
 *Sent:* Friday, May 29, 2009 7:18 PM
 *Subject:* [obm-l] Ajuda em probabilidade

 Uma urna tem 5 bolas, sendo 1 preta e as outra 4 brancas. As bolas são
 retiradas da urna sem reposição. Qual a chance de até a bola preta ser a
 última a ser retirada?

 Fernando Gama


  --
 Esta mensagem foi verificada pelo E-mail Protegido Terra.
 Atualizado em 29/05/2009

  --


 No virus found in this incoming message.
 Checked by AVG - www.avg.com
 Version: 8.5.339 / Virus Database: 270.12.46/2142 - Release Date:
 05/29/09 17:53:00


  --
 Esta mensagem foi verificada pelo E-mail Protegido Terra.
 Atualizado em 29/05/2009

  --


 No virus found in this incoming message.
 Checked by AVG - www.avg.com
 Version: 8.5.339 / Virus Database: 270.12.46/2142 - Release Date: 05/29/09
 17:53:00




 --
 Rafael



[obm-l] Ajuda em probabilidade

2009-05-29 Por tôpico Fernando Lima Gama Junior
Uma urna tem 5 bolas, sendo 1 preta e as outra 4 brancas. As bolas são
retiradas da urna sem reposição. Qual a chance de até a bola preta ser a
última a ser retirada?

Fernando Gama


Re: [obm-l] Ajuda em probabilidade

2009-05-29 Por tôpico Fernando Lima Gama Junior
Qual seria a chance, então, de ela ser tirada até a terceira bola?

Fernando Gama

Sent from Brasilia, Distrito Federal, Brazil


2009/5/29 RitaGomes rcggo...@terra.com.br

  Fernando,

 Como são 5 bolas e 1 sendo preta, fazemos permutação de 5 que é 240, porem
 a preta deve ser a última a ser retirada. Faz permutação de 4, que é 24 ,
 sendo a possibilidade da bola preta sair em ordem diferente da última.
 Desconta do totoal das condições, ou seja 240 - 24 = 216 possibilidades de
 ser a última a ser retirada.
 Espero não ter feito o cálculo errado, pois estou meio atorodoada aqui com
 outros estudos.

 Rita Gomes

 - Original Message -
 *From:* Fernando Lima Gama Junior fgam...@gmail.com
 *To:* obm-l@mat.puc-rio.br
 *Sent:* Friday, May 29, 2009 7:18 PM
 *Subject:* [obm-l] Ajuda em probabilidade

 Uma urna tem 5 bolas, sendo 1 preta e as outra 4 brancas. As bolas são
 retiradas da urna sem reposição. Qual a chance de até a bola preta ser a
 última a ser retirada?

 Fernando Gama


  --
 Esta mensagem foi verificada pelo E-mail Protegido Terra.
 Atualizado em 29/05/2009

  --


 No virus found in this incoming message.
 Checked by AVG - www.avg.com
 Version: 8.5.339 / Virus Database: 270.12.46/2142 - Release Date: 05/29/09
 17:53:00




Re: [obm-l] fantasma da luciana

2009-05-26 Por tôpico Fernando Lima Gama Junior
Com certeza ela fez alguma coisa de errado ou a lista está configurada de
forma errada. Talvez o administrador da lista possa nos dar alguma luz. Ou
pelo menos bloquear os emails oriundos da Luciana.
Abraços,
Fernando Gama



2009/5/26 Jefferson Gomes jeffe...@gmail.com

 Pessoal, eu recebi a seguinte resposta da Luciana:
 O que acham?
 Jefferson Gomes


 - Forwarded message --
 From: lucianarodrigg...@uol.com.br
 Date: 2009/5/25
 Subject: Re: Luciana - voce configurou mal o seu email
 To: Jefferson jeffe...@gmail.com


 Jefferson

 Já faz uns dois ou três meses que saí da obm.

 Att, Luciana


 Em 24/05/2009 09:08, *Jefferson  jeffe...@gmail.com * escreveu:


 Luciana, você está respondendo de forma automática todos os emails da lista
 OBM, entupindo a lista online.
 Por favor, reveja suas configurações.
 Obrigado,
 Jefferson.





[obm-l] Re: [obm-l] Re: [obm-l] Geometria Plana - 3 problema s clássicos

2009-05-26 Por tôpico Fernando Lima Gama Junior
Começou...

Fernando Gama

Sent from Brasilia, DF, Brazil

2009/5/26 lucianarodrigg...@uol.com.br




 Em 25/05/2009 22:05, *Carlos Nehab  ne...@infolink.com.br * escreveu:


 Aos aficcionados:

 Três problemas clássicos e interessantes de geometria plana:

 1) Dado um triângulo ABC, identifique o triângulo de perímetro mínimo
 nele inscrito (cada vértice - P, Q e R, em um lado distinto de ABC).
 2) Determinar o centro de uma circunferência dada utilizando apenas
 compasso.
 3) Determinar o ponto médio de um segmento dado, utilizando apenas
 compasso (difícil).

 Nehab

 =
 Instruções para entrar na lista, sair da lista e usar a lista em
 http://www.mat.puc-rio.br/~obmlistas/obm-l.html
 =

 =
 Instruções para entrar na lista, sair da lista e usar a lista em
 http://www.mat.puc-rio.br/~obmlistas/obm-l.html=


Re: [obm-l] fantasma da luciana

2009-05-26 Por tôpico Fernando Lima Gama Junior
O problema é que o remetente não é a luciana, mas a lista
obm-l@mat.puc-rio.br, certo? E ae?
Fernando Gama

Sent from Brasilia, Distrito Federal, Brazil

2009/5/26 Bruno França dos Reis bfr...@gmail.com

 Pessoal, não precisa ser tão expert pra criar um filtro nos seus clients de
 email para eliminar automaticamente todo email recebido com a luciana como
 remetente...

 Bruno
 --
 Bruno FRANÇA DOS REIS

 msn: brunoreis...@hotmail.com
 skype: brunoreis666
 tel: +33 (0)6 28 43 42 16

 http://brunoreis.com
 http://blog.brunoreis.com

 GPG Key: http://brunoreis.com/bruno-public.key

 e^(pi*i)+1=0


 2009/5/26 Bernardo Freitas Paulo da Costa bernardo...@gmail.com

  2009/5/26 Carlos Nehab ne...@infolink.com.br:
  Oi, gente,
 Oi Jefferson, Fernando, Nehab,

  Acho que o Ponce (e outros) explicarão melhor esta chatice.
 Uma tentativa abaixo, mas bastante incompleta, devo reconhecer.

  A princípio tem algum moleque, ou como se diz mais modernamente, um
  babaca, possivelmente amolado com a Lista, mandando emails se fazendo
  passar por outros.
 Ao que parece, os mails brancos ou xxx da Luciana vêm
 dela mesmo... O Jefferson tem mais informação do qua a gente (porque
 tem o mail original da Luciana, vindo dela mesma, e não de um robô ou
 um brincalhão), mas ao que tudo indica, os mails que vêm com o nome
 dela pra lista passam por shark2.uol.com.br e relay5.uol.com.br, o que
 indica que eles vêm realmente de uma conta UOL. Por outro lado, os
 endereços do emissor são completamente desconhecidos
 (weasel3.srv.intranet, IP 172.26.14.32, não localizável, e
 possivelmente não no Brasil), o que complica a coisa; não é só alguém
 que finge que é a luciana, é alguém que tem acesso à conta dela...
 Só o cabeçalho que o Jefferson tem é que pode tirar a dúvida final,
 mas talvez o problema seja mais embaixo... por exemplo, um vírus no
 computador dela, que fica mandando esses spams pra gente. O mais
 estranho é ela ter dito já ter saído da lista, como é que ela faz pra
 fazer um reply ??

  Desafio pros experts da Lista.  Mãos 'a obra, gente.
 Algumas pistas acima... Jefferson, se você puder encontrar os
 cabeçalhos e confirmar/refutar minha interpretação, ótimo. Mas talvez
 a gente devesse discutir disso fora da lista (onde ?)

 Abraços,
 --
 Bernardo Freitas Paulo da Costa

 =
 Instruções para entrar na lista, sair da lista e usar a lista em
 http://www.mat.puc-rio.br/~obmlistas/obm-l.htmlhttp://www.mat.puc-rio.br/%7Eobmlistas/obm-l.html
 =





Re: [obm-l] Exponencial

2009-05-25 Por tôpico Fernando Lima Gama Junior
Luciana, seu email está com algum problema, não está?

Fernando Gama

Sent from Brasilia, Distrito Federal, Brazil

2009/5/25 lucianarodrigg...@uol.com.br




 Em 25/05/2009 11:23, *Carlos Nehab  ne...@infolink.com.br * escreveu:


  Ihhh, Luis,

 Este que você postou com certeza é do tempo do Ari Quintela e do Cecil
 Thire ;-) :-) .   Haja décadas...

 Abraços
 Nehab


 Luís Lopes escreveu:

 Sauda,c~oes,

 Vamos ver se esta chega tambem.

 O que conhecia eh

 4^x + 6^x = 9^x

 (Divida tudo por  e ... )

 []'s
 Luis


 Thu, 21 May 2009 20:58:01 -0300, fabrici...@usp.br 
 http://compose?to=fabricio  http://compose?to=fabricio escreveu:



  Acredito que seja:

 4^x + 6^x = 2.9^x

 Aí, a solução existe. (Divida tudo por 9^x e...)

 .

 On May 21, 2009, at 19:59 , Rhilbert Rivera wrote:



  A resposta do Luís bate com o valor obtido no Maple. Se quiser um
 valor inteiro, o único jeito é dar um jeitinho
 4^x+x^6=29^x  ...

 From: qed_te...@hotmail.com http://compose?to=qed_te...@hotmail.com
 To: obm-l@mat.puc-rio.br http://compose?to=obm
 Subject: RE: [obm-l] Exponencial
 Date: Thu, 21 May 2009 21:35:21 +

 Sauda,c~oes,

 Pelo excelente site aqui indicado há poucos dias
 encontrei

 x ~~ 0.3915575306295271

 []'s
 Luís




 Date: Thu, 21 May 2009 18:19:02 -0300
 Subject: Re: [obm-l] Exponencial
 From: saulo.nil...@gmail.com http://compose?to=saulo.nil...@gmail.com
 To: obm-l@mat.puc-rio.br http://compose?to=obm

 x=0,6355

 2009/5/20 Eduardo Wilner  http://compose?to=eduardowil...@yahoo.com.br

 Ops , estou me referindo a x natural (inteiro positivo).

 --- Em qua, 20/5/09, Walter Tadeu Nogueira da Silveira   
 http://compose?to=wtadeu
 escreveu:

 De: Walter Tadeu Nogueira da Silveira  http://compose?to=wtadeu

 Assunto: [obm-l] Exponencial
 Para: obm-l@mat.puc-rio.br http://compose?to=obm
 Data: Quarta-feira, 20 de Maio de 2009, 18:18

 Amigos,

 Deparei-me com a questão do livro do Euclides Roxo 190. e lá
 vai...

 4^x + 6^x = 29 ^x

 Tentei uma solução algébrica e não numérica. Não creio que haja um
 x inteiro. Alguma idéia?

 Abraços

 --
 Walter Tadeu Nogueira da Silveira



 Veja quais são os assuntos do momento no Yahoo! + Buscados: Top 10 -
 Celebridades - Música - Esportes


 Novo Internet Explorer 8: mais rápido e muito mais seguro. Baixe
 agora, é grátis!
 Quer uma internet mais segura? Baixe agora o novo Internet Explorer
 8. É grátis!


  =
 Instruções para entrar na lista, sair da lista e usar a lista 
 emhttp://www.mat.puc-rio.br/~obmlistas/obm-l.html
 =






  =
 Instruções para entrar na lista, sair da lista e usar a lista 
 emhttp://www.mat.puc-rio.br/~obmlistas/obm-l.html
 =




 =
 Instruções para entrar na lista, sair da lista e usar a lista em
 http://www.mat.puc-rio.br/~obmlistas/obm-l.html=

 =
 Instruções para entrar na lista, sair da lista e usar a lista em
 http://www.mat.puc-rio.br/~obmlistas/obm-l.html=


Re: [obm-l] Problema Bonito - Probabilidade

2009-05-20 Por tôpico Fernando Lima Gama Junior
Oi Pedro, obrigado pela ajuda.

Na verdade, estava com um problema parecido com este. A história era
praticamente a mesma, só que o período era de 6 minutos (claro é só corrigir
a equação) e do encontro, se ocorresse, sairia um duelo no estilo velho
oeste. O porém da sua solução, no entanto (embora muito boa), é que ela não
usa os conceitos de análise combinatória. A pergunta que eu deveria ter
feito é a seguinte: como resolver esse problema por meio de análise
combinatória (se é que é possível). Digo isso porque o problema estava
estampado em um livro de análise combinatória do lendário Victor Mirshawka.
Muito bom o livro de exercícios dele que encontrei num sebo, mas de todos os
problemas, esse para mim, parecia insolúvel por meio da análise
combinatória.

Fernando Gama



2009/5/20 Pedro Cardoso pedrolaz...@hotmail.com


 Oi, Fernando. Veja que os pontos pertencentes às retas são as
 'situações limites' - aquelas em que uma pessoa chega exatamente
 10min depois (ou antes, né?) da outra. Além disso, um par (a,b), para fazer
 parte do conjunto solução do problema, deve satisfazer três coisas:

 [1] a-b  -10 .:. b  a+10 (isto é, b está abaixo da reta y = a+10)
 [2] a-b  10 .:. b  a - 10 (isto é, b está acima da reta y = a-10)
 [3] 0 = a,b = 60

 Então, se (a,b) satisfaz as condições de encontro das duas pessoas,
 então (a,b) está entre essas retas E dentro do quadrado. O resto é conta.

 Abraços,

 Pedro Lazéra Cardoso

 --
 Date: Tue, 19 May 2009 22:59:55 -0300
 Subject: Re: [obm-l] Problema Bonito - Probabilidade
 From: fgam...@gmail.com
 To: obm-l@mat.puc-rio.br


 Não entendi porque destas retas:

 *Desenhar as retas y = x+10 e y = x-10.*
 *
 *
 *A região do quadrado entre as retas (região S) forma o conjunto de pares
 (a,b) tal que abs(a-b) = 10, isto é, os pares*
 *que representam tempos de chegada para os quais há encontro entre as duas
 pessoas. O quadrado todo representa o conjunto de todos os pares possíveis
 (tudo em minuto, claro). Assim, como os pares são equiprováveis...*

 2009/5/19 Pedro Cardoso pedrolaz...@hotmail.com

 Olá. Eu acho que é assim:
 Problema:

 luiz silva escreveu:
 Duas pessoas marcam um encontro em um determinado local. Combinam que ambos
 deverão chgegar a este local entre 12 e13 h. Porém, qdo o 1o. chegar ao
 local, irá esperar 10 min pelo outro. Caso o outro não chegeu ao local nete
 intervalo de tempo (10 min), o primeiro a chegar vai embora, e eles não
 conseguem se encontrar. Qual a probabilidade do encontro ocorrer ?

 Bom, seja (a,b) o par que representa os dois instantes em que as duas
 pessoas chegaram, onde 0 = a,b = 60.
 Como a chance de ocorrência dos pares é igual, vale desenhar um quadrado de
 lado 60 no plano cartesiano,
 cujos vértices ficam nos pontos (0,0), (60,0), (60,60), (0,60), e fazer o
 seguinte:

 Desenhar as retas y = x+10 e y = x-10.

 A região do quadrado entre as retas (região S) forma o conjunto de pares
 (a,b) tal que abs(a-b) = 10, isto é, os pares
 que representam tempos de chegada para os quais há encontro entre as duas
 pessoas. O quadrado todo representa o conjunto de todos os pares possíveis
 (tudo em minuto, claro). Assim, como os pares são equiprováveis...

 Basta calcular Área de S / Área do Quadrado = 11/36 para achar a
 resposta do problema.

 Eu também fiz usando integral, mas ficou bem mais feio, tendo que dividir
 em casos.

 Abraços,

 Pedro Lazéra Cardoso

 --
 Conheça os novos produtos Windows Live. Clique 
 aqui!http://www.windowslive.com.br/




 --
 Fernando Gama


 --
 Novo Internet Explorer 8: mais rápido e muito mais seguro. Baixe agora, é
 grátis!http://brasil.microsoft.com.br/IE8/mergulhe/?utm_source=MSN%3BHotmailutm_medium=Taglineutm_campaign=IE8



Re: [obm-l] Problema Bonito - Probabilidade

2009-05-19 Por tôpico Fernando Lima Gama Junior
Não entendi porque destas retas:

*Desenhar as retas y = x+10 e y = x-10.*
*
*
*A região do quadrado entre as retas (região S) forma o conjunto de pares
(a,b) tal que abs(a-b) = 10, isto é, os pares*
*que representam tempos de chegada para os quais há encontro entre as duas
pessoas. O quadrado todo representa o conjunto de todos os pares possíveis
(tudo em minuto, claro). Assim, como os pares são equiprováveis...*

2009/5/19 Pedro Cardoso pedrolaz...@hotmail.com

 Olá. Eu acho que é assim:
 Problema:

 luiz silva escreveu:
 Duas pessoas marcam um encontro em um determinado local. Combinam que ambos
 deverão chgegar a este local entre 12 e13 h. Porém, qdo o 1o. chegar ao
 local, irá esperar 10 min pelo outro. Caso o outro não chegeu ao local nete
 intervalo de tempo (10 min), o primeiro a chegar vai embora, e eles não
 conseguem se encontrar. Qual a probabilidade do encontro ocorrer ?

 Bom, seja (a,b) o par que representa os dois instantes em que as duas
 pessoas chegaram, onde 0 = a,b = 60.
 Como a chance de ocorrência dos pares é igual, vale desenhar um quadrado de
 lado 60 no plano cartesiano,
 cujos vértices ficam nos pontos (0,0), (60,0), (60,60), (0,60), e fazer o
 seguinte:

 Desenhar as retas y = x+10 e y = x-10.

 A região do quadrado entre as retas (região S) forma o conjunto de pares
 (a,b) tal que abs(a-b) = 10, isto é, os pares
 que representam tempos de chegada para os quais há encontro entre as duas
 pessoas. O quadrado todo representa o conjunto de todos os pares possíveis
 (tudo em minuto, claro). Assim, como os pares são equiprováveis...

 Basta calcular Área de S / Área do Quadrado = 11/36 para achar a
 resposta do problema.

 Eu também fiz usando integral, mas ficou bem mais feio, tendo que dividir
 em casos.

 Abraços,

 Pedro Lazéra Cardoso

 --
 Conheça os novos produtos Windows Live. Clique 
 aqui!http://www.windowslive.com.br/




-- 
Fernando Gama


Re: [obm-l] Problema Bonito - Probabilidade

2009-05-19 Por tôpico Fernando Lima Gama Junior
De fato, achei 12/36. Onde foi que eu errei?


integral_0^60(10-t) dt+ integral_0^60(10+t) dt



Fernando Gama



2009/5/19 Fernando Lima Gama Junior fgam...@gmail.com

 Não entendi porque destas retas:

 *Desenhar as retas y = x+10 e y = x-10.*
 *
 *
 *A região do quadrado entre as retas (região S) forma o conjunto de pares
 (a,b) tal que abs(a-b) = 10, isto é, os pares*
 *que representam tempos de chegada para os quais há encontro entre as duas
 pessoas. O quadrado todo representa o conjunto de todos os pares possíveis
 (tudo em minuto, claro). Assim, como os pares são equiprováveis...*

 2009/5/19 Pedro Cardoso pedrolaz...@hotmail.com

  Olá. Eu acho que é assim:
 Problema:

 luiz silva escreveu:
 Duas pessoas marcam um encontro em um determinado local. Combinam que
 ambos deverão chgegar a este local entre 12 e13 h. Porém, qdo o 1o. chegar
 ao local, irá esperar 10 min pelo outro. Caso o outro não chegeu ao local
 nete intervalo de tempo (10 min), o primeiro a chegar vai embora, e eles não
 conseguem se encontrar. Qual a probabilidade do encontro ocorrer ?

 Bom, seja (a,b) o par que representa os dois instantes em que as duas
 pessoas chegaram, onde 0 = a,b = 60.
 Como a chance de ocorrência dos pares é igual, vale desenhar um quadrado
 de lado 60 no plano cartesiano,
 cujos vértices ficam nos pontos (0,0), (60,0), (60,60), (0,60), e fazer o
 seguinte:

 Desenhar as retas y = x+10 e y = x-10.

 A região do quadrado entre as retas (região S) forma o conjunto de pares
 (a,b) tal que abs(a-b) = 10, isto é, os pares
 que representam tempos de chegada para os quais há encontro entre as duas
 pessoas. O quadrado todo representa o conjunto de todos os pares possíveis
 (tudo em minuto, claro). Assim, como os pares são equiprováveis...

 Basta calcular Área de S / Área do Quadrado = 11/36 para achar a
 resposta do problema.

 Eu também fiz usando integral, mas ficou bem mais feio, tendo que dividir
 em casos.

 Abraços,

 Pedro Lazéra Cardoso

 --
 Conheça os novos produtos Windows Live. Clique 
 aqui!http://www.windowslive.com.br/




 --
 Fernando Gama




Re: [obm-l] Problema Bonito - Probabilidade

2009-05-19 Por tôpico Fernando Lima Gama Junior
int 10 - t dt, t=0..60 + int 10 + t dt, t=0..60
Fernando Gama



2009/5/20 Fernando Lima Gama Junior fgam...@gmail.com

 De fato, achei 12/36. Onde foi que eu errei?


 integral_0^60(10-t) dt+ integral_0^60(10+t) dt



 Fernando Gama



 2009/5/19 Fernando Lima Gama Junior fgam...@gmail.com

 Não entendi porque destas retas:

 *Desenhar as retas y = x+10 e y = x-10.*
 *
 *
 *A região do quadrado entre as retas (região S) forma o conjunto de pares
 (a,b) tal que abs(a-b) = 10, isto é, os pares*
 *que representam tempos de chegada para os quais há encontro entre as
 duas pessoas. O quadrado todo representa o conjunto de todos os pares
 possíveis (tudo em minuto, claro). Assim, como os pares são
 equiprováveis...*

 2009/5/19 Pedro Cardoso pedrolaz...@hotmail.com

  Olá. Eu acho que é assim:
 Problema:

 luiz silva escreveu:
 Duas pessoas marcam um encontro em um determinado local. Combinam que
 ambos deverão chgegar a este local entre 12 e13 h. Porém, qdo o 1o. chegar
 ao local, irá esperar 10 min pelo outro. Caso o outro não chegeu ao local
 nete intervalo de tempo (10 min), o primeiro a chegar vai embora, e eles não
 conseguem se encontrar. Qual a probabilidade do encontro ocorrer ?

 Bom, seja (a,b) o par que representa os dois instantes em que as duas
 pessoas chegaram, onde 0 = a,b = 60.
 Como a chance de ocorrência dos pares é igual, vale desenhar um quadrado
 de lado 60 no plano cartesiano,
 cujos vértices ficam nos pontos (0,0), (60,0), (60,60), (0,60), e fazer o
 seguinte:

 Desenhar as retas y = x+10 e y = x-10.

 A região do quadrado entre as retas (região S) forma o conjunto de pares
 (a,b) tal que abs(a-b) = 10, isto é, os pares
 que representam tempos de chegada para os quais há encontro entre as duas
 pessoas. O quadrado todo representa o conjunto de todos os pares possíveis
 (tudo em minuto, claro). Assim, como os pares são equiprováveis...

 Basta calcular Área de S / Área do Quadrado = 11/36 para achar a
 resposta do problema.

 Eu também fiz usando integral, mas ficou bem mais feio, tendo que dividir
 em casos.

 Abraços,

 Pedro Lazéra Cardoso

 --
 Conheça os novos produtos Windows Live. Clique 
 aqui!http://www.windowslive.com.br/




 --
 Fernando Gama





Re: [obm-l] Gauss vs. Auto-valores

2009-04-21 Por tôpico Fernando Lima Gama Junior
Ele ainda não devolveu as provas corrigidas. Assim que entregar, eu posto
aqui. Também estou curioso. Aliás, por curiosidade até hoje ainda tento
resolver nas horas vagas aquele problema da matriz, sem solução. Estou
esperando que o professor saiba resolver. ahuahuahua. Acho que essa semana
ele entrega as provas.

Abcs,


Fernando Gama



2009/4/21 Bruno França dos Reis bfr...@gmail.com

 E aí, alguma novidade no assunto método de Gauss e cálculo de auto-valores?

 Ainda estou curioso para saber a resposta da questão da prova que nosso
 colega falou.

 Abraço!
 Bruno

 --
 Bruno FRANÇA DOS REIS

 msn: brunoreis...@hotmail.com
 skype: brunoreis666
 tel: +33 (0)6 28 43 42 16

 http://brunoreis.com
 http://blog.brunoreis.com

 GPG Key: http://brunoreis.com/bruno-public.key

 e^(pi*i)+1=0



Re: [obm-l] Pontos Fixos

2009-04-15 Por tôpico Fernando Lima Gama Junior
Eu também não sei explicar como, mas o professor meu, calcado no teorema SVD
disse que há como sair. Aliás, essa é prova do doutorado. Vou transcrevê-la
aqui:

Considere uma matriz quadrada n x n, A. Considere que você consiga
decompô-la, através do método de Gauss, em uma matriz UU (ou LL). Provar que
através do cálculo dos autovalores e autovetores de UU (ou LL) é possível
encontrar os autovalores e autovetores de A.

Meu esboço:

A = LL.UU

UU - decomposição em Gauss
A - dado do problema
LL calculável

autovalor de UU - linha diagonal
autovalor de LL - linha diagonal

Relação entre os autovetores de LL e UU (não sei ainda como estabelecer)

Bem, a prova parece ser tão fácil que ele deu uma semana para a gente
fazer, podendo consultar o que fosse. O prazo termina amanhã e ninguém ainda
conseguiu. Por isso joguei o problema na lista.

Abraços,
Fernando


Fernando Gama



2009/4/12 Bruno França dos Reis bfr...@gmail.com

 Fernando, não entendi direito ainda. Eu peguei a matriz que eu mandei no
 exemplo anterior, que tinha autovalores 1 2 e 3, e fiz a decomposição LU, e
 no final das contas U tem autovalores 1, 1 e 1, ao passo que L tem
 autovalores 4, 3 e 0.5, ou seja, não são os mesmos que da matriz A. Vc falou
 que a partir daí sai os autovalores de A, eu não consegui ver como :/
 Vc poderia explicar?

 Abraço
 Bruno

 --
 Bruno FRANÇA DOS REIS

 msn: brunoreis...@hotmail.com
 skype: brunoreis666
 tel: +33 (0)6 28 43 42 16

 http://brunoreis.com
 http://blog.brunoreis.com

 GPG Key: http://brunoreis.com/bruno-public.key

 e^(pi*i)+1=0


 2009/4/12 Fernando Lima Gama Junior fgam...@gmail.com

 O teorema da decomposição SVD, garante que os autovalores são os mesmos.
 SVD é a sigla do termo em inglês Singular Value Decomposition, decomposição
 em valores singulares, no caso, autovalores. Pode ser visto em Matrix
 Computation de Loan Golub, Numerical Analisys de R. L. Burden and J. D.
 Faires.

 Fernando Gama



 2009/4/12 Bruno França dos Reis bfr...@gmail.com

 Fernando, poderia explicar melhor seu método? Não entendi como funciona.
 Abraço
 Bruno

 --
 Bruno FRANÇA DOS REIS

 msn: brunoreis...@hotmail.com
 skype: brunoreis666
 tel: +33 (0)6 28 43 42 16

 http://brunoreis.com
 http://blog.brunoreis.com

 GPG Key: http://brunoreis.com/bruno-public.key

 e^(pi*i)+1=0


 2009/4/12 Fernando Lima Gama Júnior fgam...@gmail.com

  À despeito do que o Bruno pensa, é possível sim usar Gauss para
 calcular autovalores. Só não consegui ainda achar os autovetores.


 A = LL X UU

 UU - gauss
 LL=A*UU^(-1)

 Descobre-se os autovalores LL e UU e daí sai os autovalores de A.

 O problema é com os autovetores...

 Well, quem não acredita é só tentar em casa...

 Fernando


 silverra...@gmail.com escreveu:

  Caros colegas,
  Como posso usar o método de Gauss pra calcular autovalores?
  (...)
  Ok, brincadeirinhas à parte.. gostaria de outras opiniões sobre a
 minha resolução
 do seguinte problema.
   * Problema: Seja X um subconjunto não-vazio, limitado e fechado da
 reta.
 Considere uma função F: X - X contínua, não-decrescente.
 Prove que existe p pertencente a X tal que F( p ) = p, ou seja, F tem
 um ponto fixo.
   * Demonstração: Escolha y0 em X. Construa a sequência:
 y1 = f( y0 ), y2 = f( y1 ), ..., yn = f( y(n-1) ),...
  Como X é limitado, a sequência {yn} é limitada. Além disso, sendo F
 não-decrescente,
 a sequência {yn} é monótona. Logo {yn} é convergente.
 Como X é fechado, lim (yn) pertence a X.
  F contínua = F( lim (yn) ) = lim (F(yn)) = lim (y(n+1)) = lim (yn).
  Ou seja, lim (yn) é um ponto fixo para F.
   Cometi algum erro Crasso, ou é isso mesmo?
  Obrigado! :)
  - Leandro.



 =
 Instruções para entrar na lista, sair da lista e usar a lista em
 http://www.mat.puc-rio.br/~obmlistas/obm-l.htmlhttp://www.mat.puc-rio.br/%7Eobmlistas/obm-l.html

 =







Re: [obm-l] Pontos Fixos

2009-04-15 Por tôpico Fernando Lima Gama Junior
Ainda não. Estou supercurioso, pq cada um achou uma resposta diferente.
Ficou de entregar semana que vem.

Abcs,
Fernando Gama



2009/4/15 Bruno França dos Reis bfr...@gmail.com

 Oi, Fernando, parece que deu um pau ou no meu email ou na lista, esta sua
 mensagem (da sua prova) só chegou agora há alguns minutos (assim como umas
 30 mensagens da OBM-L desta última semana), então acho que já foi o prazo.

 E aí, alguém conseguiu resolver o problema? Seu professor comentou?


 Bruno


 --
 Bruno FRANÇA DOS REIS

 msn: brunoreis...@hotmail.com
 skype: brunoreis666
 tel: +33 (0)6 28 43 42 16

 http://brunoreis.com
 http://blog.brunoreis.com

 GPG Key: http://brunoreis.com/bruno-public.key

 e^(pi*i)+1=0


 2009/4/13 Fernando Lima Gama Junior fgam...@gmail.com

 Eu também não sei explicar como, mas o professor meu, calcado no teorema
 SVD disse que há como sair. Aliás, essa é prova do doutorado. Vou
 transcrevê-la aqui:

 Considere uma matriz quadrada n x n, A. Considere que você consiga
 decompô-la, através do método de Gauss, em uma matriz UU (ou LL). Provar que
 através do cálculo dos autovalores e autovetores de UU (ou LL) é possível
 encontrar os autovalores e autovetores de A.

 Meu esboço:

 A = LL.UU

 UU - decomposição em Gauss
 A - dado do problema
 LL calculável

 autovalor de UU - linha diagonal
 autovalor de LL - linha diagonal

 Relação entre os autovetores de LL e UU (não sei ainda como estabelecer)

 Bem, a prova parece ser tão fácil que ele deu uma semana para a gente
 fazer, podendo consultar o que fosse. O prazo termina amanhã e ninguém ainda
 conseguiu. Por isso joguei o problema na lista.

 Abraços,
  Fernando



 Fernando Gama



 2009/4/12 Bruno França dos Reis bfr...@gmail.com

 Fernando, não entendi direito ainda. Eu peguei a matriz que eu mandei no
 exemplo anterior, que tinha autovalores 1 2 e 3, e fiz a decomposição LU, e
 no final das contas U tem autovalores 1, 1 e 1, ao passo que L tem
 autovalores 4, 3 e 0.5, ou seja, não são os mesmos que da matriz A. Vc falou
 que a partir daí sai os autovalores de A, eu não consegui ver como :/
 Vc poderia explicar?

 Abraço
 Bruno

 --
 Bruno FRANÇA DOS REIS

 msn: brunoreis...@hotmail.com
 skype: brunoreis666
 tel: +33 (0)6 28 43 42 16

 http://brunoreis.com
 http://blog.brunoreis.com

 GPG Key: http://brunoreis.com/bruno-public.key

 e^(pi*i)+1=0


 2009/4/12 Fernando Lima Gama Junior fgam...@gmail.com

 O teorema da decomposição SVD, garante que os autovalores são os mesmos.
 SVD é a sigla do termo em inglês Singular Value Decomposition, decomposição
 em valores singulares, no caso, autovalores. Pode ser visto em Matrix
 Computation de Loan Golub, Numerical Analisys de R. L. Burden and J. D.
 Faires.

 Fernando Gama



 2009/4/12 Bruno França dos Reis bfr...@gmail.com

 Fernando, poderia explicar melhor seu método? Não entendi como funciona.
 Abraço
 Bruno

 --
 Bruno FRANÇA DOS REIS

 msn: brunoreis...@hotmail.com
 skype: brunoreis666
 tel: +33 (0)6 28 43 42 16

 http://brunoreis.com
 http://blog.brunoreis.com

 GPG Key: http://brunoreis.com/bruno-public.key

 e^(pi*i)+1=0


 2009/4/12 Fernando Lima Gama Júnior fgam...@gmail.com

  À despeito do que o Bruno pensa, é possível sim usar Gauss para
 calcular autovalores. Só não consegui ainda achar os autovetores.


 A = LL X UU

 UU - gauss
 LL=A*UU^(-1)

 Descobre-se os autovalores LL e UU e daí sai os autovalores de A.

 O problema é com os autovetores...

 Well, quem não acredita é só tentar em casa...

 Fernando


 silverra...@gmail.com escreveu:

  Caros colegas,
  Como posso usar o método de Gauss pra calcular autovalores?
  (...)
  Ok, brincadeirinhas à parte.. gostaria de outras opiniões sobre a
 minha resolução
 do seguinte problema.
   * Problema: Seja X um subconjunto não-vazio, limitado e fechado da
 reta.
 Considere uma função F: X - X contínua, não-decrescente.
 Prove que existe p pertencente a X tal que F( p ) = p, ou seja, F tem
 um ponto fixo.
   * Demonstração: Escolha y0 em X. Construa a sequência:
 y1 = f( y0 ), y2 = f( y1 ), ..., yn = f( y(n-1) ),...
  Como X é limitado, a sequência {yn} é limitada. Além disso, sendo F
 não-decrescente,
 a sequência {yn} é monótona. Logo {yn} é convergente.
 Como X é fechado, lim (yn) pertence a X.
  F contínua = F( lim (yn) ) = lim (F(yn)) = lim (y(n+1)) = lim (yn).
  Ou seja, lim (yn) é um ponto fixo para F.
   Cometi algum erro Crasso, ou é isso mesmo?
  Obrigado! :)
  - Leandro.



 =
 Instruções para entrar na lista, sair da lista e usar a lista em
 http://www.mat.puc-rio.br/~obmlistas/obm-l.htmlhttp://www.mat.puc-rio.br/%7Eobmlistas/obm-l.html

 =









Re: [obm-l] Pontos Fixos

2009-04-12 Por tôpico Fernando Lima Gama Junior
O teorema da decomposição SVD, garante que os autovalores são os mesmos. SVD
é a sigla do termo em inglês Singular Value Decomposition, decomposição em
valores singulares, no caso, autovalores. Pode ser visto em Matrix
Computation de Loan Golub, Numerical Analisys de R. L. Burden and J. D.
Faires.

Fernando Gama



2009/4/12 Bruno França dos Reis bfr...@gmail.com

 Fernando, poderia explicar melhor seu método? Não entendi como funciona.
 Abraço
 Bruno

 --
 Bruno FRANÇA DOS REIS

 msn: brunoreis...@hotmail.com
 skype: brunoreis666
 tel: +33 (0)6 28 43 42 16

 http://brunoreis.com
 http://blog.brunoreis.com

 GPG Key: http://brunoreis.com/bruno-public.key

 e^(pi*i)+1=0


 2009/4/12 Fernando Lima Gama Júnior fgam...@gmail.com

  À despeito do que o Bruno pensa, é possível sim usar Gauss para calcular
 autovalores. Só não consegui ainda achar os autovetores.


 A = LL X UU

 UU - gauss
 LL=A*UU^(-1)

 Descobre-se os autovalores LL e UU e daí sai os autovalores de A.

 O problema é com os autovetores...

 Well, quem não acredita é só tentar em casa...

 Fernando


 silverra...@gmail.com escreveu:

  Caros colegas,
  Como posso usar o método de Gauss pra calcular autovalores?
  (...)
  Ok, brincadeirinhas à parte.. gostaria de outras opiniões sobre a minha
 resolução
 do seguinte problema.
   * Problema: Seja X um subconjunto não-vazio, limitado e fechado da
 reta.
 Considere uma função F: X - X contínua, não-decrescente.
 Prove que existe p pertencente a X tal que F( p ) = p, ou seja, F tem um
 ponto fixo.
   * Demonstração: Escolha y0 em X. Construa a sequência:
 y1 = f( y0 ), y2 = f( y1 ), ..., yn = f( y(n-1) ),...
  Como X é limitado, a sequência {yn} é limitada. Além disso, sendo F
 não-decrescente,
 a sequência {yn} é monótona. Logo {yn} é convergente.
 Como X é fechado, lim (yn) pertence a X.
  F contínua = F( lim (yn) ) = lim (F(yn)) = lim (y(n+1)) = lim (yn).
  Ou seja, lim (yn) é um ponto fixo para F.
   Cometi algum erro Crasso, ou é isso mesmo?
  Obrigado! :)
  - Leandro.


 =
 Instruções para entrar na lista, sair da lista e usar a lista em
 http://www.mat.puc-rio.br/~obmlistas/obm-l.htmlhttp://www.mat.puc-rio.br/%7Eobmlistas/obm-l.html
 =





Re: [obm-l] Pontos Fixos

2009-04-11 Por tôpico Fernando Lima Gama Júnior
À despeito do que o Bruno pensa, é possível sim usar Gauss para calcular 
autovalores. Só não consegui ainda achar os autovetores.



A = LL X UU

UU - gauss
LL=A*UU^(-1)

Descobre-se os autovalores LL e UU e daí sai os autovalores de A.

O problema é com os autovetores...

Well, quem não acredita é só tentar em casa...

Fernando


silverra...@gmail.com escreveu:

Caros colegas,
 
Como posso usar o método de Gauss pra calcular autovalores?
 
(...)
 
Ok, brincadeirinhas à parte.. gostaria de outras opiniões sobre a 
minha resolução

do seguinte problema.
 
 
* Problema: Seja X um subconjunto não-vazio, limitado e fechado da reta.

Considere uma função F: X - X contínua, não-decrescente.
Prove que existe p pertencente a X tal que F( p ) = p, ou seja, F tem 
um ponto fixo.
 
 
* Demonstração: Escolha y0 em X. Construa a sequência:

y1 = f( y0 ), y2 = f( y1 ), ..., yn = f( y(n-1) ),...
 
Como X é limitado, a sequência {yn} é limitada. Além disso, sendo F 
não-decrescente,

a sequência {yn} é monótona. Logo {yn} é convergente.
Como X é fechado, lim (yn) pertence a X.
 
F contínua = F( lim (yn) ) = lim (F(yn)) = lim (y(n+1)) = lim (yn).
 
Ou seja, lim (yn) é um ponto fixo para F.
 
 
Cometi algum erro Crasso, ou é isso mesmo?
 
Obrigado! :)
 
- Leandro.


=
Instruções para entrar na lista, sair da lista e usar a lista em
http://www.mat.puc-rio.br/~obmlistas/obm-l.html
=


[obm-l] Matrizes

2009-04-10 Por tôpico Fernando Lima Gama Junior
Uma matriz C sofreu o processo de eliminação de Gauss, virando a matriz C*.
C e C* tem os mesmos autovelores e autovetores? (Note que C* é triangular
superior).


Fernando Gama


Re: [obm-l] Matrizes

2009-04-10 Por tôpico Fernando Lima Gama Junior
Bruno, antes que você fique nervoso (de novo) assim como ontem (ou
anteontem, para quem está no horário brasileiro), segue a resposta do meu
professor do Doutorado. Ele é Ph.D pela Unicamp, de modo que acredito, não
esteja falando besteira.
*

*

*Oi, Fernando!*

*Uma maneira de facilitar a determinação dos autovalores, é transformar a
matriz original numa matriz triangular superior (ou inferior), daí os
autovalores serão o elementos da diagonal principal.*

*Este processo pode ser feito pelo método de eliminação de Gauss, bem mais
simples que o processo de diagonalização, que necessita encontrar os
autovetores.*
*Uma observação, se a matriz possui autovalores complexos, a diagonalização
não é possível, no máximo o que você consegue é a diagonalização por blocos,
de matrizes 2x2.

Prof. Geraldo L. Diniz
Phones: +55(65)3615-8713 (office)
   +55(65)3615-8704 (fax)
Skype:   dinizgl *


Portanto, o que você fala, vai de encontro ao que ele, professor fala, por
isso a minha insistência no assunto. Ou você, ou ele, está errado. Ou eu não
sei ler.

Abraços,

Fernando Gama



2009/4/10 Bruno França dos Reis bfr...@gmail.com

 Fernando, vc está de brincadeira, não é mesmo?

 Antes de ontem (ou mesmo ontem, para quem está no horário brasileiro)
 EXATAMENTE essa questão foi bm discutida num tema lançado por você
 mesmo!

 Novamente: processo de eliminação de Gauss NÃO CONSERVA AUTOVALORES. Ponto.

 Pegue os mesmo exemplos e contra exemplo da discussão anterior, pois esta É
 a discussão anterior.

 Além disso, uma matriz triangular, assim como uma matriz diagonal, exibe
 seus autovalores na sua diagonal principal.

 Pra tentar te convencer que essa história de método de Gauss não serve pra
 nada na hora de diagonalizar matriz, entenda que o objetivo do método de
 Gauss é transformar uma matriz A em uma matriz diagonal com apenas 1's ou
 0's na diagonal principal, tanto para matrizes quadradas como para não
 quadradas.

 Se o método de Gauss conservasse os autovalores, como vc tanto insiste,
 então toda matriz só poderia ter 0 e 1 como autovalores, o que é um grande
 absurdo. Ainda mais, matrizes não quadradas teriam autovalores (?!?!?)


 A única coisa para a qual vc pode utilizar o método de Gauss é para estudar
 a independência linear das linhas/colunas de uma matriz. Lembrando-se do que
 eu disse no email anterior, operações elementares não alteram propriedades
 de dependência linear.

 Se vc então descobrir que a matriz não é de posto completo, isto é, que o
 conjunto das linhas/colunas não é linearmente independente, então significa
 que o núcleo não é vazio, o que nos diz que 0 é autovalor, ou seja, o
 polinômio característico vai ter a cara p(x) = x*q(x), que vc pode fatorar o
 x para te ajudar no cálculo.


 Ficou claro?

 Bruno

 --
 Bruno FRANÇA DOS REIS

 msn: brunoreis...@hotmail.com
 skype: brunoreis666
 tel: +33 (0)6 28 43 42 16

 http://brunoreis.com
 http://blog.brunoreis.com

 GPG Key: http://brunoreis.com/bruno-public.key

 e^(pi*i)+1=0


 2009/4/11 Fernando Lima Gama Junior fgam...@gmail.com



 Uma matriz C sofreu o processo de eliminação de Gauss, virando a matriz
 C*. C e C* tem os mesmos autovelores e autovetores? (Note que C* é
 triangular superior).


 Fernando Gama






Re: [obm-l] Como diagonalizar uma matriz?

2009-04-08 Por tôpico Fernando Lima Gama Junior
Então vou fazer a pergunta de outro jeito. Se eu ir simplificando a matriz
pelo método de Gaus, de modo a ter mais zeros, essa matriz transformada terá
os mesmos autovalores e autovetores da matriz inicial?

Fernando


2009/4/7 Ralph Teixeira ralp...@gmail.com

 Fiz de cabeça... :) :) :)

 Tá, usei o computador de novo (não o Excel, mas o tal do Scientific
 Workplace). Mas se eu fizesse o polinômio de 4o grau, ele seria
 divisível por x (daí o autovalor 0), e aí sobraria um polinômio que é
 fatorável como (x-3)^2.(x+4) (daí o autovalor duplo 3, e o -4).
 Então, se eu tivesse feito isso, neste caso teria funcionado (pois eu
 teria tido sorte) e eu teria achado as raízes.

 Em geral, concordo que achar os 4 autovalores de uma matriz 4x4 pode
 ser BEM complicado, se a equação que aparecer for nojenta.

 Abraço,
 Ralph

 2009/4/7 Fernando Lima Gama Junior fgam...@gmail.com:
  Oi Ralph, obrigado pelas respostas. Mas, não sendo diagonalizável, como
  conseguiu achar os autovalores? Fez no braço mesmo? Pq se fizer, vai
 gerar
  um polinomio de 4º grau de dificil solução algébrica...
 
  Abcs,
 
  2009/4/7 Ralph Teixeira ralp...@gmail.com
 
  Oi, Fernando.
 
  Esta matriz não é diagonalizável! Ela só tem 3 autovetores L.I., e não
  4. São eles:
  Autovalor 0: (-1,7,1,6)
  Autovalor -4: (-5,-2,8,4)
  Autovalor 3: (-2,2,-1,3)
  (3 é raiz dupla do pol. carac., mas não há outro autovetor asssociado ao
  3)
  Então o melhor que você consegue é colocá-la na forma de Jordan:
 
  0  0 0 0
  0 -4 0 0
  0  0 3 0
  0  0 1 3
 
  Note aquele 1 abaixo do primeiro 3 -- você não vai conseguir se livrar
  dele.
 
  Tanto quanto eu sei, não há diagonalização de matrizes no Excel, pelo
  menos não nativamente nas versões que eu conheço.
 
  Abraço,
  Ralph
 
  2009/4/7 Fernando Lima Gama Junior fgam...@gmail.com:
   Pessoal, sei que a pergunta parece fácil, mas não estou conseguindo
   diagonalizar a seguinte matriz:
  
2 -1 -3 1
   -2 -1 -1 1
4  0 -2 1
0 -2 -4 3
  
   Alguém poderia me ajudar? Há como diagonalizar matrizes no Excel?
   --
   Fernando Gama
 
 
 =
  Instruções para entrar na lista, sair da lista e usar a lista em
  http://www.mat.puc-rio.br/~obmlistas/obm-l.htmlhttp://www.mat.puc-rio.br/%7Eobmlistas/obm-l.html
 
 =
 
 
 
  --
  Fernando Gama
 
 

 =
 Instruções para entrar na lista, sair da lista e usar a lista em
 http://www.mat.puc-rio.br/~obmlistas/obm-l.htmlhttp://www.mat.puc-rio.br/%7Eobmlistas/obm-l.html
 =




-- 
Fernando Gama


  1   2   3   4   >